ExamQs Flashcards

1
Q

Write a short note on the biological behavior and histopathological features of Phyllodes tumour of the breast

A

Biological behaviour:
- Can grow slowly then rapid increase in size
Behaviour depends on grading:
o Benign: very low risk of recurrence (and recurrence may be benign as well), does not metastasize,
o Borderline: between benign and malignant
o Malignant – 30% risk of local recurrence, 10% risk of distant recurrence

Histopathological features:
- Leaf-like epithelial pattern
- Graded into benign, border, or malignant based on: stroma atypia/ cellularity/ overgrowth,
mitotic count, tumour border, malignant heterologous elements

How well did you know this?
1
Not at all
2
3
4
5
Perfectly
2
Q

A 35 year old female has a right mastectomy and axillar dissection for an invasive breast cancer.
a. List the factors in the pathology report that are required for you to make a decision on the recommendation for adjuvant radiation therapy. Indicate how each of these factors influences your decision to recommend adjuvant radiation therapy in this patient.

A

Especially in LN+ve patients, metanalysis demonstrates PMRT confers a locoregional recurrence and BCa-specific mortality reduction. In addition - The SUMREMO trial would consider CWRT in the following:
T3, close margin, T2N0grd 3, T2N0 with LVSI.
Therefore in report:
Tumour size >= T3 (50mm)
ALND: nodes positive, also nodes harvested (<=10 should consider nodal RT), ECE = RT, Nodes >=4 do RT, strongly consider>1, also micromets.
Hormone status: tripple negative
Grade = 3 and T2 = CWRT in SUPREMO
LVSI = consider RT is tumour T2
Margin status: <1mm
Multifocal disease.

How well did you know this?
1
Not at all
2
3
4
5
Perfectly
3
Q

The analyses of gene expression is being increasingly used in invasive breast cancer. Describe the 21 gene
recurrence score (Oncotype Dx) and the Amsterdam 70 gene profile (Mammaprint). Explain the utility of these (4m)

A

Oncotype:
Prognostic and predicitive (which non-locally advanced N0 pts benefit from chemo).
- Developed in the NSABP-B18 and B20 cohort
- Based on 21-gene DNA micro-array
- Used in women with ER+, N0 who have endocrine therapy
- Predict risk of distant metastases and benefits from adjuvant chemotherapy
- Stratified into 3 groups based on risk score – low (<18), intermediate (18-30), high (>30)

Mammaprint:
Based on 70-gene profile (DNA microarray) Prognostically validated in early breast (<T2=2-5cm) regardless of hormone status

(See also PAM50 and the EXPERT trial)

How well did you know this?
1
Not at all
2
3
4
5
Perfectly
4
Q

a. List the subtypes of non-Hodgkin lymphoma which occur within the hollow organs of the gastro-intestinal tract
(2m)

A

Follicular
Marginal - MALT
DLBCL
Mantle cell
Primary gastro-intestinal T cell lymphoma

How well did you know this?
1
Not at all
2
3
4
5
Perfectly
5
Q

A 40 year old man presents with a mass in the anterior mediastinum (4m)
i. What is your differential diagnosis for this mass?

A

Non-malignant:
Thymic cyst/congenital
Reactive nodes
Malignant Ddx:
Lymphoma: E.g mediastinal DLBCL, follicular lymphoma
Nodal met/or met: e.g from primary lung/skin (e.g melanoma)
Sarcoma
Seminoma/nonSem Germ cell tumour

How well did you know this?
1
Not at all
2
3
4
5
Perfectly
6
Q

A 40 year old man presents with a mass in the anterior mediastinum (4m)
ii. Describe the different approaches to obtaining a histological diagnosis for this mass, including specimen handling, plus advantages and disadvantages of each method described

A

CT-guided Bx - Advantages: Less resources (theatre time/staff), does not require GA. Disadvantages: Operator dependent, may be challenging, with high-risk anatomy (e.g SVC) nearby. No direct visualisation of mass.

EBUS - Advantage: Less chance of miss (compared to CT), can be done as a day day case, with less risk than operative Bx. Disadvantages: FNA therefore no architectural info, limited sample- insufficient for flow cytometry. Not all mediastinal lesions accessible by EBUS.

Medianoscopy and Bx (generally preferred), Adv: Direct visualisation, more tissue for full work up. Disadv: Operative risks and resources (GA, surgeon, theatre staff etc), where resources limited may delay Dx.

Open excision Biopsy: Adv: Highest chance of obtaining tissue, may be therapeutic. Disadv:Higherst risk, longest recovery and most resource intensive.

How well did you know this?
1
Not at all
2
3
4
5
Perfectly
7
Q

Compare and contrast the histological features of primary mediastinal B cell lymphoma and seminoma?

How could ancillary techniques be used to distinguish this entities?

A

Med BCL: Diffuse or clustered growing B-Cell neoplasm characterised by intermediate sized nuclei

Seminoma: Large round cells, “fried egg” nuclei
I
IHC B cell: +ve CD 19, 20, PAX8
IHC seminoma: +ve C-KIT, PLAP

How well did you know this?
1
Not at all
2
3
4
5
Perfectly
8
Q

A 70 year old man presents with a palpable cervical lymph node a. List the differential diagnosis (1m)

b. Regarding follicular lymphoma, describe the (2m)
i. Microscopic features
ii. Immunohistochemical profile

A

i. Microscopic features
B-cell neoplasm, forming follicles of centrocytes (smaller, cleaved nuclei) and centroblasts (larger, non cleaved lymphoid cells).

ii. Immunohistochemical profile
Pan B-cell marker: CD 19,20,21,79 and PAX8
Germinal centre markers: BCL-2, BCL-6, CD 10
Negative for T-cell marker: CD 3 and 5.

How well did you know this?
1
Not at all
2
3
4
5
Perfectly
9
Q

How is follicular lymphoma graded? (2m)

A

Grading from 1 to 3 based on density (#/HPF) of centroblasts: grd 1<5/hpf, grd 3>15/hpf. Grd 3 is incurable, grd 3B is centrocytes only (forming solid sheets)

How well did you know this?
1
Not at all
2
3
4
5
Perfectly
10
Q

Describe a prognostic scoring system used in follicular lymphoma (2m)

A

FLIP-2 (B-BLAH-N)*, similar to IPI for DLBCL, used to stratify into low (0-1) risk factor, intermediate and high (>2) RFs.

FLIPI-2: Bone Marrow Involvement, Beta2 Microglobulin> upper lim normal (ULN), Age>60, anaemia (Hb<120). And nodes>6cm. Don’t forget nodes!!!

Where RF are:
(B) Beta2 microglobuin >ULN,
(B)Bone marrow involvement,
(L)LDH>ULN,
(A)Age>60, [
(H) [Hb]<120.

10yr OS for low, int, and high risk are: 80, 50, 20 (%) respectively

How well did you know this?
1
Not at all
2
3
4
5
Perfectly
11
Q

The majority of follicular lymphoma have a translocation. Name the translocation. Describe 2 laboratory methods that can be used to assess whether the translocation is present (1m)

A

t(14,18) in 80-90%.

1) Cytogenetics via FISH for presence of translocation in chromosome (detected by flurescent probe)

2) IHC Over-expression of BCL2 protein from t(14:18) translocation
Localisation of protein antigen by fluorescent-tagged antibody through antigen- antibody interaction

How well did you know this?
1
Not at all
2
3
4
5
Perfectly
12
Q

What is the natural history of low grade follicular lymphoma (2m)

A

Generally indolent (within above prognostic Risk categories)
Excellent LC with RT (>90%) with distant failure likely
10-year PFS 50%; 10-year OS 50% 
30-50% risk of transformation to DLBCL

How well did you know this?
1
Not at all
2
3
4
5
Perfectly
13
Q

A 62 year old male presents with a six month history of increasing shortness of breath. Ten years ago he received chemotherapy and radiation therapy for a diffuse large B cell mediastinal lymphoma.
a. What are the possible causes for his shortness of breath? (3 marks)

A

Break into Disease related, treatment related, other causes:

Disease: Recurrence with mass effect (e.g/ SVC obstruction, trachial/bronchi compression potentially lobar collapse).

Treatment:
- CTx related (doxorubicin): Heart failure/ valvular disease
- RT-related
Lung: lung fibrosis, Secondary lung malignancies e.g. mesothelioma
Heart: valvular dysfunction, cardiac arrhythmia, heart failure

Other: name the systems: Resp: LRTI/COPD, CVS: Valv/failure/silent MI, Metabolic/Renal: Renal failure, oedma, Haem: anaemia.

How well did you know this?
1
Not at all
2
3
4
5
Perfectly
14
Q

In general, what strategies can be used to manage breathlessness in terminal cancer patients? (2 marks)

A

Pharmacology
- Opioid
- Sedative agents (midazolam/ benzodiazepam)
- Dexamethasone
- anti-secretory (hyoscine, atropine, glyco-pyronium bromide)

Non- pharmacology
- supplemental oxygen
- positioning (slight head elevation)
- manage secretion (suction)
- ascitic tap
- pastoral care/ single room

How well did you know this?
1
Not at all
2
3
4
5
Perfectly
15
Q

A 28 year old woman presents with a 6-month history of lethargy, progressive shortness of breath and non- productive cough. A chest CT scan demonstrates an 8cm mass in the anterior mediastinum. A core biopsy is performed. Histopathology demonstrates a diffuse large B-cell lymphoma

a. What further investigations would you organize (1m)

A

To complete IPI (LAKES) need LDH, determine stage and number of extra nodal sites. Also the below:
Blood test
FBE/ UEC/ CMP
Prognostic factors – LDH
B-HCG – exclude pregnancy
HIV
Hep B/ C – likely require rituximab, which may reactive HepB/C

Tissue Additional:
immunophenotyped from mediastinal biopsy
- C-MYC/ BCL2/ BCL6 for double/ triple hit phenotype
BMAT – ? marrow involvement

Imaging - CT and FDG PET scan
- Echo-cardiogram (baseline cardiac function prior to R-CHOP)

How well did you know this?
1
Not at all
2
3
4
5
Perfectly
16
Q

A 28 year old woman presents with a 6-month history of lethargy, progressive shortness of breath and non- productive cough. A chest CT scan demonstrates an 8cm mass in the anterior mediastinum. A core biopsy is performed. Histopathology demonstrates a diffuse large B-cell lymphoma

Staging investigations demonstrate no disease outside of the mediastinum.

What would you recommend as management for this patient now? Justify your answer (2m)

A

Assuming Stage I/II:

Bulky (>5cm or 7.5cm) should be treated with 6 cycles of R-CHOP (increased if activeated centre or dbl/trppl hit, e.g. x8 cycles). A PET should be perfomed post treatment. And adjRT ISRT 30-36Gy should be considered for increased event free survival (though not PFS/OS), I would not strongly recommend this, but would offer discussion.

UNFLODER RCT: Adjuvant radiotherapy for bulky disease after x6 R-CHOP, included sub analysis of primary mediastinal BCL for which EFS was only benefit. For partial responders on PET, adjRT may improve EFS to the degree of complete responders.

Interestingly RT may be associated with less secondary maligancy risk than other treatments (e.g. CAR T cell 4-6%).

How well did you know this?
1
Not at all
2
3
4
5
Perfectly
17
Q

A 28 year old woman presents with a 6-month history of lethargy, progressive shortness of breath and non- productive cough. A chest CT scan demonstrates an 8cm mass in the anterior mediastinum. A core biopsy is performed. Histopathology demonstrates a diffuse large B-cell lymphoma

Staging investigations demonstrate no disease outside of the mediastinum.
If her management was to include radiation therapy, describe a suitable technique and dose prescription for delivering this treatment (3m)

A

Adjuvant EBRT for improved EFS, to a total of 30gy/15#, 5#s/week, VMAT technique 6Mv photons.

Pre sim: pregnancy test, PPI/antemetics as needed

Sim: as usual.
2mm slice CT4, fusion with diagnostic PET
Planned on MIP and checked against all phases of breathing cycle.
Volumes: GTV = residual disease on CT and PET
CTV = GTV + extent of disease on pre-treatment CT/PET. Respecting anatomic boundaries except where invasion.
PTV = CTV + 5mm.

OARS -
MHD <4gy
Lung V20 Gy ≤30%
Breast mean dose <4Gy
Esophagus/Spine ALARA principle

Daily CBCT matched to bone with soft tissue review.

How well did you know this?
1
Not at all
2
3
4
5
Perfectly
18
Q

A 22 year old female presents with cough and shortness of breath. A chest x-ray shows a widened mediastinum.

Further investigation confirms a 10cm mediastinal mass with no evidence of disease elsewhere. i. What are the 4 most likely differential diagnoses in this patient? (1m)

ii. How will your proceed to obtain histological diagnosis (2m)

A

i. What are the 4 most likely differential diagnoses in this patient? (1m)
- Lymphoma (Hodgkin’s lymphoma, diffuse large B-cell lymphoma, primary mediastinal B-cell lymphoma)
- Germ cell tumour (teratoma)
- Thyroid malignancies
- Thymoma/ thymic cancers

Dx =
- CT-guided core biopsy
- Alternative with EBUS

How well did you know this?
1
Not at all
2
3
4
5
Perfectly
19
Q

A fit 46 year old female is referred for investigation of a 5cm lesion of the proximal humerus demonstrating full thickness cortical erosion in areas. Biopsy demonstrates a monoclonal plasma cell infiltrate.
a. What further investigation would you perform? Justify your answer (4m)

A

For a diagnosis IMWG requires all following criteria be met:
1)Bx-proven solitary lesion of bone or soft tissue with evidence of clonal plasma cells
2)Normal bone marrow with no evidence of clonal plasma cells
3)Normal skeletal survey and MRI (or CT) of the spine and pelvis, except for the primary solitary lesion
4) Absence of end-organ damage = CRAB (hypercalcemia, renal failure, anemia, and osteolytic bone lesions), that can be attributed to a lympho-plasma cell proliferative disorder

Other:
- Beta2-microglobulin, albumin, LDH (for staging)

How well did you know this?
1
Not at all
2
3
4
5
Perfectly
20
Q

In general, what is the role of surgery in the management of solitary bone plasmacytoma (SBP)? (1m)

A

May have role in surgical fixation for stabilisation, where there is risk of vertebral instability, or risk of pathological fracture. Assess by SINS (“greater than or equal to 8 you should fix8”) and Mirel’s scores respectivly.

How well did you know this?
1
Not at all
2
3
4
5
Perfectly
21
Q

What is the Mirel scoring system? Include in your answer its component and how it is used (3m)

A

Each of 4 components scored 1-3, where prophylactic Long bone fixation should be considered @8, and indicated at>=9.
A) Pain 1=mid 2 mod 3 severe
B) Location: Upper limb 1, LL 2, intertroch 3
C) Lesion: Blastic =1, Mixed 2, Lytic 3
D) Cortex involved <1/3=1, >2/3=3

How well did you know this?
1
Not at all
2
3
4
5
Perfectly
22
Q

What is the risk of this patient (plasmacytoma) progressing to multiple myeloma in the next 10 years?

A

Almost all will transform to MM – 50% in 5 years, 80-90% in 10 years

How well did you know this?
1
Not at all
2
3
4
5
Perfectly
23
Q

If the patient progressed to multiple myeloma, what would you discuss with this patient regarding the natural history of the disease with treatment and their prognosis

A

Incurable; alternate between remission and relapse Depending on stage
o MedianOS:Stage1~5yr,Stage2~4yr,Stage3~2yr

How well did you know this?
1
Not at all
2
3
4
5
Perfectly
24
Q

Compare multiple myeloma and plasmacytoma in terms of: (6m)
i. Pathological definition

A

MM:
Malignant proliferation of clonal plasma cells, requires 3 criteria:
- >10% abnormal clonal plasma cells on
bone marrow biopsy
- M-protein >3g/dL on SPEP/ UPEP
- Presence of CRAB end organ failure

Solitary plasmacytoma
- Presence of abnormal clonal plasma cells in one site (bone, or extramedullary)

How well did you know this?
1
Not at all
2
3
4
5
Perfectly
25
Q

Compare multiple myeloma and plasmacytoma in terms of: (6m)
ii. Clinical presentation

A

MM
CRAB end organ failure
- Hypercalcaemia
- Renal failure
- Anaemia
- Bone pain

PM
No CRAB features
Bone: bone pain, vertebral compression fracture Extramedullary (site dependent) e.g. nasal obstruction, epistaxis

How well did you know this?
1
Not at all
2
3
4
5
Perfectly
26
Q

Compare multiple myeloma and plasmacytoma in terms of: (6m)

iii. Biological behaviour

A

MM
OS 6 months without treatment
- Incurable; alternate between relapse/
remission
Survival based on stage: MedianOS Stage1~5yr, Stage2~4yr, Stage3~2yr

PM
- 90% local control with radiotherapy
- Risk of MM transformation
o Bone: 80% 10 year for bone
o Extramedullary: 20-30% at 10-year

How well did you know this?
1
Not at all
2
3
4
5
Perfectly
27
Q

c. Briefly describe the factors associated with poor prognosis for patients with multiple myeloma (2m)

A

Patient:
Increasing age
Poor performance status

Tumour
R-ISS factors
- high 2-microglobulin
- high LDH
- low albumin
- high risk cytogenetic abnormalities (t(4:14), del 17p, t(14:16) Other
- high CRP
- plasma-blastic morphology
- bone marrow infiltration

Treatment
fitness for systemic therapy/ bone marrow transplant

How well did you know this?
1
Not at all
2
3
4
5
Perfectly
28
Q

Regarding Hodgkin’s lymphoma:
a. Briefly describe the epidemiology (2m)

Describe the microscopic and immunohistochemical features (2m)

A
  • Relatively uncommon, ~10% of lymphoma
  • Male predominance
  • Bimodal: mid-20s, and 60s

Micro
- 1-2% of tumour volume are Reed-Sternberg cells (classic diagnostic cell, binucleated with prominent nucleoli, well-demarcated nuclear membrane, and eosinophilic cytoplasm with perinuclear halo)
- The rest are tumour infiltration of lymphocytes, eosinophils, and plasma cells

IHC CD15+, CD30+, PAX+

How well did you know this?
1
Not at all
2
3
4
5
Perfectly
29
Q

Regarding Hodgkin’s lymphoma:

List the factors associated with an adverse prognosis (2m)

A

Early stage (Stage I-II )
GHSG/ EORTC (slight differences) ‘A-MEEN’ Presence of any of the following factors = unfavourable
- Mediastinal mass : thoracic ratio >1/3
- Extranodal site
- ESR (>30 if B symptoms, >50 if no B symptoms)
- Nodal region >3 (GHSG), or >4 (EORTC)
- Age >50 (EORTC)

Advanced stage (Stage III-IV)
‘ALL-SHAM’
- Age >45
- Leucocytosis,
- Lymphocytopenia
- Stage IV,
- Hb<105
- Albumin<4
- Male

How well did you know this?
1
Not at all
2
3
4
5
Perfectly
30
Q

A 22 year old man presents with a painless mass in the right inguinofemoral region. He denies any other symptoms and examination is otherwise unremarkable. A core biopsy of the 3x5cm mass reveals nodular sclerosing Hodgkin’s disease. A staging CT and PET scan show no disease elsewhere.

a. Assuming that there are no adverse prognostic features, discuss the treatment options available for this man,
including the advantages and disadvantages of each approach (4m)

A

This man has Stage 1A (early favourable) right inguinal nodular HL

Option 1
2 x ABVD + INRT 20Gy (GHSG HD10)
Advantages
Similar outcomes (PFS/ OS) compared to 4xABVD and INRT 30Gy
- Less treatment and less toxicities
Disadvantages
Data mainly in pre-PET era

Option 2 (preferred)
PET-adapted approach
2xABVD then interim PET staging (EORTC H10)
a) - If PET negative, option of further 1xABVD, or RT
- Spare the late s/e of RT (e.g. 2nd malignancies),
- No difference in OS compared to
- In the event of progression, there are
good salvage options
Disadvantage
BUT! Lower PFS with omission of RT

b)If PET positive, - Improve PFS and OS compared to consider 2xBEACOPP + ISRT 30Gy

BUT
BEACOPP more toxicity than ABVD

How well did you know this?
1
Not at all
2
3
4
5
Perfectly
31
Q

A 22 year old man presents with a painless mass in the right inguinofemoral region. He denies any other symptoms and examination is otherwise unremarkable. A core biopsy of the 3x5cm mass reveals nodular sclerosing Hodgkin’s disease. A staging CT and PET scan show no disease elsewhere.

A decision is made to treat this patient with combined chemo-radiation therapy.
b. Describe in detail a suitable radiation therapy technique and dose prescription (3m)

A

I will offer him involved node radiotherapy to the right inguinal nodal mass a total dose of 20Gy in 10# with curative intent;

How well did you know this?
1
Not at all
2
3
4
5
Perfectly
32
Q

Name the histopathology and molecular subtypes of medulloblastoma

A

hISTO
- Desmoplastic/ nodular (15%)
- Extensive nodularity
- Classic (80%)
- Large cell/ anaplastic

Mol:
Group 1 (WNT)
- Group 2 (SHH)
- Group 3
- Group 4

How well did you know this?
1
Not at all
2
3
4
5
Perfectly
33
Q

For medulloblastoma - What is the clinical importance of molecular staging (1m)

A
  • Better risk stratification/ prognostication
  • Allow for development of biomarker-driven clinical trials, to allow de-intensification of treatment
How well did you know this?
1
Not at all
2
3
4
5
Perfectly
34
Q

Describe the microscopicof classical medulloblastoma
(1m)

A
  • Densely cellular and undifferentiated small round cells, (carrot-shaped hyperchromatic
    nuclei)
  • Classically a/w Homer-Wright rosettes (rings of neuroblasts surrounding eosinophilic neutrophils)
How well did you know this?
1
Not at all
2
3
4
5
Perfectly
35
Q

Describe the immunohistochemistry features of classical medulloblastoma
(1m)

A
  • Beta catenin associated with WNT
  • Neuron specific enolase (NSE) +ve
  • Synaptophysin +ve,
  • Chromogranin +ve
  • GFAP -ve (+ve in glioma)
How well did you know this?
1
Not at all
2
3
4
5
Perfectly
36
Q

Describe the cytogenetics of classical medulloblastoma
(1m)

A
  • WNT activated – Beta-catenin
  • SHH activated – PTCH1 mutation
  • MYC amplification/ gained
How well did you know this?
1
Not at all
2
3
4
5
Perfectly
37
Q

Discuss the pathological features of the desmoplastic variants of
medulloblastoma (1.5m)

A

Nodular architecture
2 important features giving desmoplastic appearance
* Reticulin fibres with densely packed undifferentiated cells with moderate pleomorphism
* Reticulin free zone (pale island)
Pathological
features
* Pathologic activation of SHH pathway, caused by SUFU and PTCH1 mutation in young
adult and PTCH1 and SMO mutation in adult

How well did you know this?
1
Not at all
2
3
4
5
Perfectly
38
Q

Discuss the biological behaviour of the desmoplastic variants of
medulloblastoma (1.5m)

A
  • Most common site = cerebellar hemisphere (cf other medulloblastoma which occur most
    commonly at midline e.g. vermis)
  • Can spread locally, or through CSF
  • Associated with good prognosis
How well did you know this?
1
Not at all
2
3
4
5
Perfectly
39
Q

An 8-year-old girl is referred following gross total resection of a classic, non-WNT (wingless type), non-SHH posterior fossa medulloblastoma. She has an uncomplicated post-operative course. There is no overt disease elsewhere and no malignant cells in the cerebrospinal fluid.
a. What risk category is this patient in? Outline a management plan for this risk category (2m)

A

I will treat this patient as per SJMB protocol
Risk category Average risk (age>3, no residual, no anaplastic variants, no mets, CSF no malignant cells)
Management Adjuvant RT + adjuvant chemo (4x VCP)

How well did you know this?
1
Not at all
2
3
4
5
Perfectly
40
Q

An 8-year-old girl is referred following gross total resection of a classic, non-WNT (wingless type), non-SHH posterior fossa medulloblastoma. She has an uncomplicated post-operative course. There is no overt disease elsewhere and no malignant cells in the cerebrospinal fluid.
b. Describe a suitable radiation therapy technique and dose fractionation regiment for her treatment. Dose constraints to organs at risk are not required to be given (5m)

A

I will offer this patient cranio-spinal irradiation to a dose of 23.4Gy in 13 fraction followed by tumour bed boost to a dose of 30.6Gy/17# (to total dose of 54Gy/30#) (1.8Gy/#, 5#/ week). VMAT technique with 6MV photon- 3 isocentres (cranial, upper spinal, lower spinal), junction at 4cm depth, no feathering.

Pre-SIM
* Ensure recovery from surgery
SIM
* Positioning: supine, arms on side, head towards gantry
* Immobilisation: on vac bag/ body cradle, thermoplastic mask
* CT simulation: 3mm slice planning CT from vertex to mid-thigh
* Fusion: with pre-op MRI, post-op MRI
Target volume:
* Phase 1: CSI 23.4Gy/13# (entire intracranial/ spinal subarachnoid volume)
o CTVcranial = Entire brain, covering cribriform plate, and optic nerve
o CTVspine = Thecal sac expand laterally to intervertebral foramina; inferior level must be determined on
MRI (usually ~ S1-2)
o PTVcranial = CTVcranial + 3mm
o PTVspine = CTVspine + 5mm lat + 10mm AP + 10mm SI
* Phase 2: Tumour bed boost 30.6Gy/17#
o GTV: surgical cavity + post-op residual disease
o CTV30.6: GTV + 5mm
o PTV30.6: CTV30.6 + 3mm

Plan evaluation:
* Ensure CTV D98>98%, and PTV D98>95%,
* Minimise hotspot: D2<105% of prescribed dose
* Homogenous 16Gy covering entire vertebra body
Treatment verification
* daily kV matched to bone, 2mm tolerance for cranial (optic nerve/ chiasm), 4mm tolerance elsewhere
Additional
* Weekly FBE monitoring (given entire vertebral marrow irradiation)

How well did you know this?
1
Not at all
2
3
4
5
Perfectly
41
Q

For medulloblastoma: What are the potential radiation therapy side effect of the treatment you have prescribed? (3m)

A

Acute:
* Neurological – headache, N+V,
* GI symptoms – N+V, diarrhea,
esophagitis,
* Bone marrow suppression – need
weekly blood
* Skin – alopecia, dermatitis
* General – fatigue

Late:
* Neurocognitive – ‘I M-ABLe’ – IQ, Memory, Attention, Behaviour,
Learning)
* Neuro-endocrine deficits (pituitary dysfunction) – GH deficiency,
hypothyroidism, gonadal dysfunction
* Hypoplasia of occipital bone (high dose boost region)
* Hearing loss (from RT and chemo)
* Eye – cataract
* Alopecia
* Cerebrovascular – aneurysm/ stroke
* 2
nd malignancy (meningioma)
Spinal RT
* Spinal cord damage
* Hypothyroidism
* Musculoskeletal – impaired soft tissue/ bone growth/ reduced siting
height
* Cardiovascular – MI, arrhythmia
* Infertility
* Second malignancy (bowel and bladder cancer)

How well did you know this?
1
Not at all
2
3
4
5
Perfectly
42
Q

For Merkel cell carcinoma, describe the epidemiology, risk factors and pathological features (3m)

A

Epi
Rare primary neuroendocrine cancer of the skin
Mean age ~70
M:F ~2:1

Risk factors
UV radiation
Increasing age
Light skin
Merkel cell polyoma virus (MCPyV)
Immunosuppression – organ transplant, CLL, myeloma

Histology
Macro: red fleshy firm nodules
Micro: small round blue cells, frequent mitoses
IHC: synaptophysin+, chromogranin+, CK20+, CK7-, EMA+

How well did you know this?
1
Not at all
2
3
4
5
Perfectly
43
Q

Describe the role of viral carcinogen in the pathogenesis of Merkel cell carcinoma (1.5m)

A
  • Merkel cell polyoma virus (MCV) is implicated
  • MCV is integrated into Merkel cell tumour, which then require continued MCV oncoprotein expression to survive
  • The integrated viral genomes have tumour-specific pattern of tumour antigen gene mutation that incapacitates viral DNA replication
How well did you know this?
1
Not at all
2
3
4
5
Perfectly
44
Q

A 58 year old man presents after local excision of a 3cm Merkel cell carcinoma from the upper right lateral arm
(over the deltoid).
There is a positive radial margin of 0.2mm and there is prominent lymphovascular space invasion.
a. What further information do you require to determine a management plan for this patient? (3m)

A

This is at least a T2 (3cm) Merkel cell carcinoma, with close surgical margin and LVI:

Hx -
Sx: tempo/ progress of primary lesion; any associated lymphoedema/ radiculopathy
- RFs: immunosuppression (organ transplant/ CLL), sun-damaged skin
- Fitness for RT: ECOG performance status, previous RT to the arm or contra-indications for
RT (which will dictate suitability for RT)

Ex - Other skin lesions, axillary/ supraclavicular lymphadenopathy (which will dictate treatment
volume)

Ix - CT C/A/P and FDG PET (routinely done, but not actually MBS funded) for assessment of
lymphadenopathy and distant mets, which will dictate treatment volume (nodal coverage and
dose) and intent (curative/ palliative)

Tx - Discussion with surgeon regarding feasibility for further resection to achieve good margin
(>3cm)

How well did you know this?
1
Not at all
2
3
4
5
Perfectly
45
Q

A 58 year old man presents after local excision of a 3cm Merkel cell carcinoma from the upper right lateral arm
(over the deltoid).
There is a positive radial margin of 0.2mm and there is prominent lymphovascular space invasion.

What management options are available for this patient? What would you recommend? Justify your answer
(4m)

A

Re-excision, node biopsy and adj RT to primary + nodal irradiation (axilla and
SCF)- Preferred recommendation
- Considered radio-sensitive disease
- Excellent local control/ distant control/ OS (>70%)
- Reduce risk of nodal recurrence (compared to no nodal irradiation)
- Avoid surgical morbidities

RT alone to primary (without elective
nodal irradiation)
- Will require SLNBx prior to RT, and if negative sentinel LN, can consider omitting nodes (in patients with T1 disease)
- In this case (T2 disease), higher risk of nodal recurrence with omission (~ 60% risk nodal recurrence)

RT + chemo - RT with concurrent weekly carboplatin
- TROG single arm study to improve LC/ PFS/ OS with concurrent chemo, compared to historical control w/o chemo (no actual evidence that outcomes superior to RT alone)
- However, associated with more chemo-related toxicity (30% febrile neutropenia)

Surgery (WLE +/-ND)
- Further re-excision, aiming for at least 3cm margin
- Morbid surgery (and will need graft)
- Avoid radiotherapy

How well did you know this?
1
Not at all
2
3
4
5
Perfectly
46
Q

A 42-year-old man, previously treated with chemotherapy for acute myeloid leukaemia, is referred following relapse
for total body irradiation prior to a bone marrow transplant.
a. What wold you explain to the patient about the rationale for total body irradiation and possible acute side effects that he may experience (3m)

A

Rationale:
1)Bone marrow ablation
(2) Immune suppression to prevent Grf vs Host
(3) Residual tumour ablation
(4) Sanctuary sites (e.g. testes/brain).

Acute SEs:
Nausea/vomitting (within 2-4 days), parotiditis (common 2-4 days post), dry eyes/xerostomia (up to a month), dysguisa, fatigue, erythema, epilation.

How well did you know this?
1
Not at all
2
3
4
5
Perfectly
47
Q

Describe a suitable radiation therapy technique and dose fractionation schedule for TBI. Justify your answer

A

DOSE PRESCRIPTION
* 12Gy in 6 fractions, 2 Gy per fraction, 2 fractions per day, over 3 days
* Prescribed to a single point at midline of the patient (usually umbilicus)
* (Other dose options: 2Gy/1#, 13.2Gy/11#, 4Gy/2#)

Pre-SIM
* MDM discussion
* Fertility preservation referral

SIMULATION
* Position: supine, upper arms on side resting on 4cm polystyrene (maximise lung shielding from lateral beam),
and hand resting on abdomen
* Planning CT: 3mm slice covering the entire body length (from vertex to mid-thigh)
TARGET VOLUME
* Entire body contour
TECHNIQUE
* 4-field equally weighted MV photons AP/PA and opposing lateral, with extended SSD (4m) and largest practical
field size
* 6/10/18 MV (avoid 18MV if possible)

Alternate APPA and oppose lateral for each fraction (e.g. APPA for fraction 1, 3, 5, lateral for fraction 2, 4, 6)

Lateral field Position:
* patient lie supine, small sponge under head, knee fix,
* Upper arm resting on 4cm polystyrene (to reduce lung dose)
* Hand resting on abdomen
* Trolley turn around for the opposing lateral field treatment
Compensator/ bolus (‘beam spoiler’) – (because of skin-sparing effect of photon)
* Super-flab – on lateral and anterior surface of neck + chest (to reduce dose to lung)
* Perspex – as head frame compensator, and from mid-thigh inferiorly (thicker from mid knee
inferiorly)

AP/PA field Position:
As above but:
* patient lie on side, head sponge under head, patient’s torso support with pillow (on beam exit side),
towel and pillow between legs
* Patient facing linac head for AP, and turned around, facing away from linac head for PA

Verification:
Physics QA prior to treatment
- Light field – TBI laser run along the incident surface of patient’s pelvis, coverage of whole body within the light
field
- TLD for dose verification on each body site of interest

How well did you know this?
1
Not at all
2
3
4
5
Perfectly
48
Q

What dose would you generally prescribe for TBI? How would you verify this? (2m)

A

Verification:
Physics QA prior to treatment
- Light field – TBI laser run along the incident surface of patient’s pelvis, coverage of whole body within the light
field
- TLD for dose verification on each body site of interest

How well did you know this?
1
Not at all
2
3
4
5
Perfectly
49
Q

Describe the technical difficulties that arise in TBI, and how these are overcome? (2m)

A

POTENTIAL TECHNICAL ISSUES
1) Large field
* Extended SSD (4m), may need special bunker set up/ trolley arrangement
2) Dose in-homogeneity
* Assumption that whole body is a block of homogenous density
* Varying body thickness and separation along the patient’s length – need ‘beam spoiler’ e.g. Perspex head compensator/ below thigh, and super-flab around anterior/ lateral neck
* Uneven tissue density in body (lung/ bone/ soft tissue) – take into account in treatment planning process

3)Treatment verification
* Only based on crude verification – that patient in line with the laser ligh

How well did you know this?
1
Not at all
2
3
4
5
Perfectly
50
Q

What physical and pharmacological measures should be employed when a patient attends for TBI treatment in radiation facility (1m)

A

Physical measures
- choose appropriate standing (up to 3omins) vs. lying technique.

Pharmacological measures
- Anti-emetic pre-treatment
- Dexamethasone
- Pre-med lorazepam

How well did you know this?
1
Not at all
2
3
4
5
Perfectly
51
Q

Intermediate and late side effects of TBI:

A

Intermediate
* Somnolence syndrome
* Pneumonitis (common – occurs in 50% of cases, and fatal in 50% of those with pneumonitis)
* Pancytopenia

Late
* Gland: Xerostomia, dysgeusia, sicca syndrome (dry eye/ dry mouth)
* Sterility/ need for hormone replacement (very common)
* Cataract (30-40%)
* Thyroid dysfunction (common) – hypothyroidism, thyroiditis
* Decreased bone mineral density
* 2nd malignancies (5% at 15 years)

How well did you know this?
1
Not at all
2
3
4
5
Perfectly
52
Q

A number of regimens have been developed in which TBI is replaced with additional chemotherapy agents.
What advantages and disadvantages of this approach? (2m)

A

Advantages of chemo
- Avoid RT toxicity and late effects of RT
- RT may be contraindication in some patients
o Connective tissue disorders, or radiosensitivity syndrome
o Previous high dose to lung
o Concern re: lung toxicity
- TBI not widely available

Disadvantages of chemo
- Depends on blood supply for delivery of chemotherapy
- influenced by variability in drug absorption, metabolism, biodistribution, or clearance
kinetics – may cause renal/ liver toxicity
- May not penetrate sanctuary sites e.g. testis and brain
- Chemo-resistant clones may be better treated with RT

How well did you know this?
1
Not at all
2
3
4
5
Perfectly
53
Q

a. A well 40-year-old female presents with intermenstrual bleeding. Clinical
examination reveals a 5 cm mass arising from the cervix. Biopsy confirms
squamous cell carcinoma.(1)
i. List and explain the components of an Examination under Anaesthetic (EUA).

A

Components of EUA to determine extent of disease:
Pt in lithotomoy position
1) External examination and palpation: of vulva, vagina, cervix, bimanual examination, DRE.
2) Internal examination:
Speculum examination and or hysteroscopy +/- biopsy.
Cystoscopy and/or proctoscopy may also be included.

How well did you know this?
1
Not at all
2
3
4
5
Perfectly
54
Q

a. A well 40-year-old female presents with intermenstrual bleeding. Clinical
examination reveals a 5 cm mass arising from the cervix. Biopsy confirms
squamous cell carcinoma.(1)
ii. Discuss the relative benefits of various imaging modalities

A

USS (not needed) benefits: Commonly available, results may help speed up further workup, no radiation (e.g. if pregnancy), can detect tumour and suspicious soft tissue changes (e.g. parametrium)
MRI: Best resolution of soft tissues allowing pre-op staging of tumour invasion, and nodal involvement.
CT staging: For systemic staging, local disease and hydronephrosis. More accessible than PET
PET CT – Sensitive for local and systemic disease, allows more accurate staging

How well did you know this?
1
Not at all
2
3
4
5
Perfectly
55
Q

In general, list the prognostic features that influence local control and survival in
cervical cancer.

A

Patient: Functional status, immune suppression, older age (>60), socioeconomic status, smoking status, Hb<110

Tumour: Nodal status and T stage (i.e. invasive higher risk recurrence and systemic spread), M stage (ie. Survival and approach to local control), LVI, DOI, tumour differentiation (poor vs well diff). Cell histology: in order worse to best prog small cell, adenosquarmous, adenoCa, SCC (basaloid SCC most aggressive SCC subtype).

Treatment: Overall treatment time (aim complete within 8 weeks, or <7 weeks), concurrent chemo (OS benefit), brachy boost available (OS benefit).

How well did you know this?
1
Not at all
2
3
4
5
Perfectly
56
Q

This patient is diagnosed with a squamous cell carcinoma of the cervix with a 2cm
para-aortic lymph node at the level of the third lumbar vertebrae (FIGO Stage
IIIC2). Definitive chemoradiation is recommended.
For the external beam component describe in detail
i. Target volumes.

A

TARGET volumes for EBRT phase
GTVp = primary on imaging (CT/PET/MRI), GTVn = Involved nodes on imaging
CTV55 = GTVn +1cm.
CTV45= (GTVp + 1cm), cervix, uterus, upper ½ vag, parametrium.
PTV45 = CTV45 +1.5cm to account for bladder/rectum varaiation.
CTVpelvis 45 = GTVn + Obturator, external, internal iliacs. Include inguinal nodes if lower 1/3 vag involved. Consider common if high risk. Extended field - 2cm above highest positive node.

How well did you know this?
1
Not at all
2
3
4
5
Perfectly
57
Q

This patient is diagnosed with a squamous cell carcinoma of the cervix with a 2cm
para-aortic lymph node at the level of the third lumbar vertebrae (FIGO Stage
IIIC2). Definitive chemoradiation is recommended.
For the external beam component describe in detail

ii. Dose fractionation schedule.

A

Dose: Curative intent EBRT VMAT technique, 6Mv photons, to primary and pelvis 45Gy/25#s, boost involved node to 55/25, 5#s/week. Prescribed to D50.

How well did you know this?
1
Not at all
2
3
4
5
Perfectly
58
Q

This patient is diagnosed with a squamous cell carcinoma of the cervix with a 2cm para-aortic lymph node at the level of the third lumbar vertebrae (FIGO Stage
IIIC2). Definitive chemoradiation is recommended.
For the external beam component describe in detail

ii. Organs at risk.

A

Rectum, small bowel, bladder, lumbar plexus, cauda equina, large bowel, femoral head, potentially kidney.

How well did you know this?
1
Not at all
2
3
4
5
Perfectly
59
Q

For soft tissue sarcoma:
a. Describe the pathogenic risk factors (2m)

A
  • Genetic predisposition (5%) – e.g. NF(NF-1), Familial Rb (Rb-1), Li-Fraumeni (p53, 17p13), Gorlins (PTC -> PTCH in Shh), the inbred Triad of Carneys (c-Kit - so GIST).
  • Prior radiotherapy e.g. undifferentiated pleomorphic sarcoma, angiosarcoma
  • Chronic iritation/inflammation
  • Carcinogen exposure
  • Viral infection HIV/ HHV8 for Kaposi sarcoma
How well did you know this?
1
Not at all
2
3
4
5
Perfectly
60
Q

For soft tissue sarcoma:
b. Describe the biological behaviour (2m)

A

Local
- Locally invasive
- Grow along path of least resistance initially (e.g. within muscle fascia/ compartment),
before eventually invade nerve/ vessels

Nodal Haematogenous Outcomes
- Nodal spread is rare (<5%) – higher in certain histology (CARE i.e. clear cell, angiosarcoma, rhabdomyosarcoma, epithelioid)

METS
- Most common route of spread is haematogenous
- Most common site lung> bone> bone marrow> liver> brain - depends on subtype (e.g. Myxoid Liposarc and angiosarc can go to skin)

Outcome: Depends on grade, nodal and met involvement, operable status and subtype (e.g. 30% 5yr OS for Leiomyosarcoma).

How well did you know this?
1
Not at all
2
3
4
5
Perfectly
61
Q

Clinical Features of Melanoma:

Epidemiology of Melanoma:

A

CLINICAL FEATURES (ABCDE)
* Asymmetry
* Border irregular
* Colour
* Diameter
* Evolve (mole evolve over time)

Mean age 50
2nd most common cancer in NZ
M>F

How well did you know this?
1
Not at all
2
3
4
5
Perfectly
62
Q

Regarding Lentigo Maligna or Hutchinson’s Melanotic Freckle, describe its (2.5):
i. Epidemiology and common locations.
Ii. Macroscopic appearance.
Iii. Biological behaviour.
iv Microscopic appearance

A

10% of all melanomas
Most common in elderly european population (age>70)
Most common sites are head and neck, and arms.

Ii. Macroscopic appearance.
At first, often resembles freckles or brown marks (lentigines). Becomes more distinctive and atypical in time, often growing to several centimetres over several years or decades. Like other flat forms of melanoma, can be recognised by ABCDE rule
Macro: freckle-like pigmented macule
Micro: lentiginous growth (continuous single cells at base of epidermis), often in 
atrophic epidermis, ill-defined borders 


Iii. Biological behaviour.
Can be slow growing over many years. 5-10% of cases develop invasive melanoma, which may result in itchiness, pain, thickening or ulceration of the lesion.

iv. Micro = Lentigenous spreading in sheet of cells at base of dermis, often occuring in atrophic skin. Not invading through basement layer.

How well did you know this?
1
Not at all
2
3
4
5
Perfectly
63
Q

Describe the BRAF signalling pathway and its significance in the development of cutaneous melanoma. Include in your answers the incidence of BRAF mutations and list its two most common variants.

A

BRAF is an intracellular part of the receptor tyrosine kinase signal transduction pathway. Where Growth factor (E.g. EGFR) binds the extracellular domain of RTK, leading to signal transduction via the following molecules: RAS (NRAs mutations 20% of melanomas) followed by RAF (BRAF mutations in 50% of melanomas) followed by MEK and ERK. Mutations in this pathway lead to increased cell growth/proliferation and apoptosis evasion, leading to increased potential for malignant transformation. Of the BRAF mutations BRAF 600E and 600K are most common (70 and 20% respectively) - can be target by BRAF inhibitors (Eg Debrafinib).

How well did you know this?
1
Not at all
2
3
4
5
Perfectly
64
Q

Regarding Superficial Spreading Melanoma, describe its (2.5):
i. Epidemiology and common locations.
Ii. Macroscopic appearance.
Iii. Biological behaviour.
iv Microscopic appearance

A

Most common subtype of melanoma - accounting for 70%
M>F
25% from existing dysplastic naevus, 75% de novo
a/w intermittent sun exposure of affected skin, light skin (Fitzpatrick skin type), younger group, a/w naevi

Macro: As per other superfical spreading can be identified by ABCDE

Micro: Radial (non-invasive) and pagetois growth components.
IHC+ve S100, SOX10, Melanin-A
IHC -ve EMA (i.e distinguish from Merkel)
Molecular: BRAF V600E or K, NRAS, kit unlikely for this subtype

Biobeh:
25% from existing dysplastic naevus, 75% de novo
Often long period of radial/superficial growth with fast appearance of nodules/invasive disease.
Less likely to spread via lymphatics/haem/skin than nodular melanoma.

How well did you know this?
1
Not at all
2
3
4
5
Perfectly
65
Q

For testicular germ cell tumours, describe:
a. The risk factors for their development (2.5m)

A

For testicular germ cell tumours, describe:
a. The risk factors for their development (2.5m)
* Cryptorchidism (abdominal cryptorchid testis higher risk than inguinal cryptorchid testis)
* Intratubular germ cell neoplasia (ITGCN)
* Testicular development disorders/ gonadal dysgenesis (e.g. Klinefelter syndrome)
* Family history
* Hypospadias
* Previous contralateral testicular cancer
* White race
* HIV

How well did you know this?
1
Not at all
2
3
4
5
Perfectly
66
Q

For testicular germ cell tumours, give the related serum tumour markers and their clinical application (5m)

A

General applications: Are not suited to screening (lack sens/spec), can help point to a Dx at initial work-up (e.g. AFP raised the not seminoma), along with disease extent, Monitoring for residual/progression.

AFP: If elevated not a seminoma, often elevated in embryonal (60%) and yolk sac (almost always)
BetaHCG - both seminomas ~10% (execpt spermatocytic) and NSGCTs may have raised levels - especially choriocarcinomas.

How well did you know this?
1
Not at all
2
3
4
5
Perfectly
67
Q

A 30 year old man undergoes an orchidectomy. Histology demonstrates pure seminoma.
a. What factors would you take into consideration when deciding on a management plan for this patient? (2m)

A

Patient
Fitness for treatment
- Radiotherapy (connective tissue disorders/ inflammatory bowel disease, horse-shoe kidney) Compliance with surveillance if opt for no adjuvant treatment
Fertility/ family planning
Patient’s preference

Tumour
Stage of disease – nodes involvement and size of node (Stage II)
Treatment
Previous abdominal/ pelvic RT

How well did you know this?
1
Not at all
2
3
4
5
Perfectly
68
Q

Outline the evidence for Mx approach to stage 1 (LVI negative) seminoma:

A

Surviellance: Retrospective data (relapse rate ~ 20%) Mortality 0.2% (due to effective salvage therapies)

TE19: 1-2 cycles Cyclophos vs Par-aortic RT (20/10). Both decrease recurrence to 5%, Chemo may be less toxic.

How well did you know this?
1
Not at all
2
3
4
5
Perfectly
69
Q

1)Describe a suitable technique and dose prescription for stage 1 seminoma post surgery:

2) Justify the target volume described above (1m)

A

Adj Para-aortic strip EBRT to reduce risk recurrence from 20% to 5%.
20Gy in 10 fractions, 2Gy per fractions, 5#s/wk VMAT technique, 6MV photons prescribed to D50
Pre-SIM
- Fertility counselling/ sperm banking
SIM
- Position: supine, arms up, head towards gantry
- Immobilisation: vac bag, knee fix, ankle support
- Additional: Clamshell for testicular shielding
- Planning CT: 3mm slice CT from mid thorax to mid-thigh Target volumes
IVC and aorta from 2cm below sup pole of kidney to bifurcation, expanded by 2cm (trimmed to boundaries), PTV 0.5cm.
- No hotspot of >107% in the field
- Review OAR DVH
o Kidney: Dmean<8Gy (=V8Gy<50%)
o Testicle: ALARA < 0.5Gy Treatment verification:
- Daily kV matched to bone, with 3mm tolerance

Justify vols:
MRC study showed that in patients with Stage 1 seminoma, there is no differences in relapse and OS between patients who had PA strip RT vs. dog-field RT

How well did you know this?
1
Not at all
2
3
4
5
Perfectly
70
Q

A 66-year-old man presents with a 3-week history of mid epigastric pain, 5% weight loss and fatigue. A GP orders an ultrasound which demonstrates a 4cm mass in the head of pancreas. History and examination is otherwise unremarkable.
a. How would you further evaluate this patient and justify your answer (2m)

A

Hx:
Symptoms: duration and tempo of pain, LOW, fatigue, nausea/ vomiting
- Address any acute symptoms e.g. analgesia for pain, anti-emetic for n+v
Risk factors: history of pancreatitis, cigarette smoking, obesity, diabetes, alcohol, family history or any genetic mutations e.g. BRCA
Fitness for treatment:
- General: ECOG, comorbidities
- Surgery: cardio/respiratory function
- Radiotherapy: connective tissue disorders/ previous chest/ abdo RT

Ex:
Jaundice
Abdominal examination – any palpable hepatosplenomegaly
Lymphadenopathy

Tests:
Bloods: LFTs (bili obstruction, mets, suitable for chemo), Lipase (pancreatitis), CA19.9, protein/alb (malnutrition), Baseline pretreatment FBC/U&E
Imaging:
- CT pancreatic protocol
- MRI (abdomen/ pelvis)/ MRCP
- FDG PET
Tissue diagnosis:
- EUS (preferred method) – better yield, safe, lower risk of peritoneal seeding (but risk of false negative)
- ERCP – may be useful if symptomatic obstructive jaundice requiring stenting

How well did you know this?
1
Not at all
2
3
4
5
Perfectly
71
Q

In general, what factors might determine whether a patient with pancreatic cancer is suitable for resection? (2m)

A

The critical things are arterial contact (none with coliac axis, sma or common hepatic), and venous (at most limited contact with SMV or portal vein).

Patient factors
Tumour factors:
- No distant mets
- No arterial tumour contact with coeliac axis, SMA, and common hepatic artery
- No radiographic evidence of SMV or portal vein contact, or <180degree contact without vein contour irregularity

How well did you know this?
1
Not at all
2
3
4
5
Perfectly
72
Q

This woman (left thigh HG myxoid) is discussed in a multi-disciplinary sarcoma meeting and a recommendation of preoperative radiation therapy is made.
c. Describe a suitable radiation treatment technique and dose prescription for this patient. (2m)

A

I will recommend pre-operative radiotherapy to the left thigh lesion to a total dose of 50.4Gy in 28 fractions, 1.8Gy per fractions, 5 fractions per week to Left thigh sarcoma. RT technique
- VMAT with 6MV photon with partial arc, prescribed to D50.

Pre-SIM
- MDM discussion
SIM:
- Position: supine, slight frog leg, arms on chest, head towards gantry
- Immobilisation: vac bag
- Planning CT: 3mm slice CT from mid abdomen to below knee
Fusion: MRI and FDG PET
Target volume:
- GTV = visible disease based on planning CT, and fused MRI/ FDG PET (T1 sequence + contrast)
- CTV = GTV + 1.5cm radial expansion + 4cm sup-inf expansion (include all oedema on T2 sequence)
- PTV = CTV + 1cm expansion

Plan evaluation
- Tumour coverage: PTV D98>95%
- Minimise hotspot: PTV D2< 105%, ensure no hotspot outside of PTV
- Ensure sparing of a strip of skin/ subcutaneous tissues for lymphatic drainage
- Review OAR DVH
o Weight bearing bone: Dmax< TD, <50% of circumferences of bone< TD, avoid dose across joint
Treatment verification
- daily CBCT, matched to bone with soft tissue review, ensuring tumour covered within PTV

How well did you know this?
1
Not at all
2
3
4
5
Perfectly
73
Q

Compare Ewings and Osteosarc in terms of:
X-ray/ CT
features (1.5m)

A

Ewings:
Lytic destructive lesions
Located at diaphysis
‘moth-eaten’, ‘onion-skinning’
Codman’s triangle

Osteosarc:
Sclerotic lesions
Located at metaphysis
‘sun-burst’ pattern (new bone formation)
Codman’s triangle

How well did you know this?
1
Not at all
2
3
4
5
Perfectly
74
Q

Compare Ewings and Osteosarc in terms of:
Aetiology/ risk
factors (1m)

A

Ewings: no known risk

Osteo: Rb mutation (next most common event after the eye). Li-Fraumeni, previous RT, Paget’s disease.

How well did you know this?
1
Not at all
2
3
4
5
Perfectly
75
Q

Compare Ewings and Osteosarc in terms of:
Microscopic
histopathology
(3m)

A

Ewings: Small round blue cells
Homer-Wright pseudo-rosettes

Osteosarc: Malignant osteoid
High grade spindle cells
No small round blue cells

How well did you know this?
1
Not at all
2
3
4
5
Perfectly
76
Q

Compare Ewings and Osteosarc in terms of:
IHC markers (1m)

A

Ewings: EWS FLI1+, CD99+,

Osteosarc: Histo Dx. CD99+ (like Ewings)

How well did you know this?
1
Not at all
2
3
4
5
Perfectly
77
Q

Compare Ewings and Osteosarc in terms of:
How is response to neo-adjuvant chemotherapy assessed histologically in both tumours (1.5m)

A

Ewings:
* Assessed by extent of % of tumour necrosis (% of viable tumour cells)

Osteo:
* Huvos grading (based on degree of necrosis (G1-2 = poor response; G3-4 = good response)
o Grade 1: no treatment effect (<50% necrosis)
o Grade 2: 50-89% necrosis
o Grade 3: 90-99% necrosis
o Grade 4: 100% necrosis, no viable tumour

Clinical implications
o Predict LC and survival, independent of complete resection
o May guide decision on adjuvant radiotherapy
o No evidence for intensification of adjuvant chemotherapy in poor responders

How well did you know this?
1
Not at all
2
3
4
5
Perfectly
78
Q

For malignant mesothelioma:
i. What is the main risk factor and briefly how does that cause carcinogenesis?

ii. Name a few other risk factors

A

> 90%:
- Asbestos exposure
- Carcinogenesis:
2 Pathways:
1. Asbestos causes ROS mediated DNA strand breaks and mutation leading to malignant transformation via increased mutation rate.

  1. Pleural macrophages
    - Further mesothelia DNA damage through release of ROS and TNF alpha.
    - Chronic inflammation providing tumour promoting environment.

Other RFs:
- Previous thoracic radiotherapy
- Viral oncogene – Simian virus (SV-40)
- Genetic susceptibility – loss of expression of BAP1, homozygous deletion of CDKN2A like grd4 Astro
- Other mineral fibres e.g. erionite

How well did you know this?
1
Not at all
2
3
4
5
Perfectly
79
Q

Describe the main histological subtypes and immunohistochemistry of mesothelioma (2m)

A
  • Epithelioid – cuboidal columnar cells, with psammoma body, favourable prognosis
  • Sarcomatoid – spindle cells, poor prognosis
  • Mixed/ biphasic – mixed of epithelioid and sarcomatoid features (at least 10% of each)

CK5/6 +
- Calretinin +
- WT1+
- TTF1-
- Napsin-A-

How well did you know this?
1
Not at all
2
3
4
5
Perfectly
80
Q

Describe the options that are available for the management of pleural effusions in the setting of malignant
mesothelioma. What are the advantages and disadvantages of each approach (4m)

A

From least to most invasive:
1) Observation - Reasonable option if
asymptomatic/ depends on
severity of effusion BUT
- Non-invasive
BUT, may be symptomatic down the
track
2) Pleural tap
(thoracentesis)
- Invasive (relatively)
- U/S guided tap BUT
- More likely to re-accumulation
- Risk of infection
- Risk of pneumothorax
3) Indwelling pleural
catheter (pig-tail)
- Patient can self-drain at regular
interval e.g. every 2nd day or
when symptomatic BUT:
- Risk of infection
- Dislodgement
- Risk of pneumothorax
4) Talc pleurodesis - Reduce risk of re-accumulation
of pleural fluid BUT:
- More invasive
- Risk of infection
- Risk of pneumothorax
- Inflammatory like symptom
5) Pleurectomy + decortication

How well did you know this?
1
Not at all
2
3
4
5
Perfectly
81
Q

Discuss the role of the following in the systemic therapy of GBM. What are the main toxicities of these agents:
(3m)
i. Temozolomide
ii. Bevacizumab

A

TMZ:
- Improve OS when given concurrent/ adjuvant
with definitive RT (Stupp)
- Can be used instead of RT in elderly patients
with MGMT methylation (as per NOA-08 and
Nordic study)!!!!!!
Tox: Myelosuppression - especially thromobocytopenia, also neutropenia.

Bevacizumab:
PFS benefit in order of 3mths
NO role in definitive setting when combined with STUPP.
Tox:
GI bleeds, bowel perf, wound break down, kidney injury.

How well did you know this?
1
Not at all
2
3
4
5
Perfectly
82
Q

What are the most important molecular and/ or genetic markers in high grade glioma and outline their
prognostic and predictive significance:

A

1) IDH:
- Presence of IDH more likely to re-present secondary HG glioma i.e. progression from LG
glioma
- Prognosis – IDH mutation = more favourable outcomes - now defines GBM (if any of: necrosis, TERTmut, EGFRmut, microvasc prolif).
- Predictive – IDH mutation = better response to chemoRT
2) MGMT meth: epigenetic silencing of DNA repair protein (esp due to alkylating agents).
- Prognostic - better outcomes indep of treatment
- Predictive - Response to Tx esp/ TMZ
3) H3K27M: associated with midline glioma.
Prognostic: assoc poor prog.
4) CDKNA2 - upgrades a grd III astro to IV
5) TERT mutation - Prognostic worse outcome in IDHwt
6) EGFRmut - worse outcomes

How well did you know this?
1
Not at all
2
3
4
5
Perfectly
83
Q

A patient with GBM has standard definitive Mx.

What is the expected survival outcomes of your recommended treatment (1m)

A

Expected median OS ~ 14 months (as per EORTC/ Stupp study).

Nat Hx = 4-6months from Dx
Stupp: 14 months
Sub-analysis and other studies suggest 26months for methylated.

How well did you know this?
1
Not at all
2
3
4
5
Perfectly
84
Q

d. In general, how may radiation necrosis be distinguished from progressive high grade glioma? (2m)

A

Needs to be discussed in MDM, or ideally specialised neuroradiologist.
MRI:
Progression (and pseudo) looks like ring-enhancement, whereas Necrosis tends to have “Cut Pepper” appreance.
DWI: Prog has restriction, High FA, lower ADC
Nec has: no restriction, Lower FA, Higher ADC
Spect/PET: Prog has increased metabolic activity.

How well did you know this?
1
Not at all
2
3
4
5
Perfectly
85
Q

a. A 57-year-old female presents with seizures. She is otherwise well, ECOG0, with unremarkable examination. MRI brain shows periventricular enhancing posterior cerebral mass without significant mass effect and biopsy confirms primary CNS lymphoma.
List the investigations you would request.(1.5)

A

How would they know its PCNSL from a Bx only? Assume an eye exam is an investigation (and not an exam). Bloods: LDH, FBC, U&E, HepB/C, HIV (Aids defining). Tissue: BMAT to confirm PCNSL. Lumbar puncture/CSF protein. Imaging: Spinal MRI, PET/CT staging (8%have systemic disease)

How well did you know this?
1
Not at all
2
3
4
5
Perfectly
86
Q

a. A 57-year-old female presents with seizures. She is otherwise well, ECOG0, with unremarkable examination. MRI brain shows periventricular enhancing posterior cerebral mass without significant mass effect and biopsy confirms primary CNS lymphoma.

b.What are the key patient and tumour related prognostic factors in this disease? (1)

A

Patient: Age >60, KPS, co-morbidties that may limit aggressive therapy (including stem cell transplant), morbidity/neurology associated with tumour
Tumour: Response to induction chemo/pre-transplant therapy, systemic spread.
There is no staging just a prognostic score (Poor DEAL): Protein in CSF, Deep brain involved, ECOG>1, age>60, LDH>ULN

How well did you know this?
1
Not at all
2
3
4
5
Perfectly
87
Q

Fit well 57F with primary CNS lymphoma.

c. Outline the preferred initial treatment for this patient. What is the disadvantage of (initial radiation therapy? (2)

A

Is this fit and well patient not considered “elderly” (age <60).
Induction: High-Dose Methotrexate + Rituxumab + Alkylating agent (e.g. TMZ)
Initial radiotherapy prevents selecting a dose based on chemo response (thereby over treating some patients) (ie. Complete response dose 23.4/13, vs 39/26)

How well did you know this?
1
Not at all
2
3
4
5
Perfectly
88
Q

Fit well 57F with primary CNS lymphoma.
d. Following initial induction chemotherapy, a complete response is achieved. What are the options for consolidation therapy? Summarise the highest evidence for
each consolidation strategy. (3.5)

A

CR (age<60)
After above (and if feasible) +/- further chemo (HD-ARAC) then Autologuos Stem Cell Transplant (ASCT) - highest chance of cure. Phase 2 PRECIS trial a complex chemo regime then ASCT is better than Ritux HDmtx and some other stuff then 23.4/13WBRT. Better = improved event free survival, and less neurotoxic.

Or

After above (and if feasible) +/- further chemo (HD-ARAC) then WBRT 23.4Gy/13#. Chemo + WBRT improves PFS (phase 3 RCT) compared to chemo alone

If Elderly: WBRT 23.4/13, alcylating agent maintenance or wait and watch are all potential options,

How well did you know this?
1
Not at all
2
3
4
5
Perfectly
89
Q

Fit well 57F with primary CNS lymphoma.
d. Following initial induction chemotherapy, a PARTIAL response is achieved. What are the options for consolidation therapy? Summarise approaches (3.5)

A

High dose Ifosphamide
Or
WBRT 23.4/13 then boost residual to 45/25

How well did you know this?
1
Not at all
2
3
4
5
Perfectly
90
Q

Broadly, with treatment what are the expected outcomes for PCNSL?

A

Outcomes based on risk factors (PoorDEAL):
Protein elevated CSF
Deep brain structures involved
ECOG>1
Age>60
LDH elevated

0-1 RFs: 2yr 80%
2-3 50%
4 or more 20%

How well did you know this?
1
Not at all
2
3
4
5
Perfectly
91
Q

A 39 year old woman present with PR bleeding and a 4cm mass in the anal canal
a. How you would further evaluate this patient (Hx and Ex)? Justify your answer

A

Hx:
Symptoms: duration and tempo of PR bleeding, pain, urinary symptoms, incontinence,
symptoms of anaemia
- Risk factors: history of other HPV related cancers (vulva/ vagina/ cervical cancer), sexual
history, history of HIV, smoking
- Fitness for treatment: ECOG performance status, comorbidities, previous pelvic RT,
inflammatory bowel disease, contraindication for RT
- Fertility/ family planning

Ex: Peri-anal/ perineal/ vulva skin changes (risk of field cancerisation)
- PR examination – determine extent of tumour and sphincter function
- PV examination – to exclude vaginal involvement
- Abdominal examination for lymphadenopathy (inguinal LN)

How well did you know this?
1
Not at all
2
3
4
5
Perfectly
92
Q

A 39 year old woman present with PR bleeding and a 4cm mass in the anal canal
a. How you would further evaluate this patient - Investigations only! ? Justify your answer

A

Blood test: FBE/ UEC/ CMP/ LFT, HIV (CD4 if HIV+), Beta-HCG
- Imaging:
o MRI pelvis allow better evaluation of local extent of primary tumour
o CT A/P to evaluate extent of local disease and nodal disease
o FDG PET scan to evaluate distant disease
- Tissue diagnosis:
o Examination under anaesthesia + colonoscopy + biopsy of primary
o FNA/ excisional biopsy of palpable/ abnormal inguinal node
o Cystoscopy if bladder involvement

How well did you know this?
1
Not at all
2
3
4
5
Perfectly
93
Q

The patients has a biopsy proven squamous cell carcinoma of the distal anal canal. Investigations reveal a 2cm
right internal iliac node. The decision is made to treat with concurrent chemotherapy and radiation therapy
b. Describe dose fractionation schedule (technique next slide)

A

This is a T2N1a anal SCC, and assuming this patient is fit for curative intent chemoradiotherapy, I will offer:

Definitive chemo to a total dose of 54Gy in 30 fractions with concurrent 5FU/ MMC

Chemo: 5FU/ MMC (MMC IV D1 12mg/m2 + 5FU infusion 1000mg/m2/day D1-4 week 1 and 5)

How well did you know this?
1
Not at all
2
3
4
5
Perfectly
94
Q

A patient with a 2cm anal mass has a biopsy proven squamous cell carcinoma of the distal anal canal. Investigations reveal a 2cm
right internal iliac node. The decision is made to treat with concurrent chemotherapy and radiation therapy
b. Technique (dose ect other slide)

A

Node positive treat like bulky (>4cm). 54/30, 45/30 to nodes

T2N2 (some would not give higher dose to nodes<3cm).
Pre-SIM
- Fertility referral
- Medical oncology referral
- Full bladder/ empty rectum
SIM
- Position: supine, heads towards gantry, arms on chest
- Immobilisation: vacbag, knee fix, ankle support
- Addition: anal marker
- Planning CT: 3mm slice planning CT from iliac crest to mid-thigh
Fuse: staging MRI and PET
Target volume delineation
- GTVp and GTVn based on clinical examination/ MRI/ PET
- CTV54 = GTVp + 2cm + entire anal canal - - CTV54n = GTVn + 1cm isotropic
- CTV45 = CTV54 + elective nodal area (mesorectum + pre-sacral space + ischio-rectal fossa + internal iliac LN +
external iliac LN + inguinal) - !!! I would strongly consider treating the next echelon (common ilacs).

Verification daily CBCT w/soft tissue rev
OARS see below.

How well did you know this?
1
Not at all
2
3
4
5
Perfectly
95
Q

OARS for pelvis (anal, cervix)

A

Small bowel loop V30Gy<200c, Dmax<50Gy
Large Bowel loops = same as small bowel.
Rectum V40Gy <60%(e.g. cervix)
Bladder: V40Gy< 30% (or 35%), V50Gy<5%
External genitalia: V40Gy< 5%
Femoral head/neck: V30Gy< 50%
Kidneys: Mean dose <15Gy

How well did you know this?
1
Not at all
2
3
4
5
Perfectly
96
Q

Anal SCC: Outline your follow-up program for this patient (1m)

A

Clinical review ~ 4 weeks post RT to ensure resolution of acute toxicity

3Months:
- Clinical review (DRE + endoscopic surveillance)
- MRI/ PET 3 month post completion of treatment to ensure CMR and tumour shrinkage, if still residual metabolic
activity, repeat PET in another 3 months

Ongoing clinic review (DRE/exam/Hx, protoscopy).
3 monthly 1st year,
6 monthly 2nd year, annually until 5 years

How well did you know this?
1
Not at all
2
3
4
5
Perfectly
97
Q

Post anal SCC defiintive ChemoRT:
If there is residual disease palpable on digital rectal examination at first post-treatment review, how would
you manage this? (2m)

A
  • Reassure patient that complete response can take 6momnths (or more)
  • Surgical review for endoscopic visualisation +/- biopsy if concern for clinical progression
  • If clinically no progression, will review again at 3 month post-completion of treatment with staging MRI and FDG
    PET scan + clinical examination
How well did you know this?
1
Not at all
2
3
4
5
Perfectly
98
Q

List the malignant tumour arising from the tissues of the endometrium (3m)

A
  • Epithelial
    o Endometrial adenocarcinoma
    o Uterine serous papillary carcinoma o Clear cell carcinoma
    o Undifferentiated adenocarcinoma o Small cell carcinoma
  • Mesenchymal
    o Uterine sarcoma – carcinosarcoma, leiomyosarcoma (LMS), stromal sarcoma
  • Uterine melanoma
  • Uterine lymphoma
  • Trophoblastic carcinoma
  • Germ cell tumour - ? teratoma
How well did you know this?
1
Not at all
2
3
4
5
Perfectly
99
Q

Discuss the evaluation of pathological information of various types in relation to the management of adenocarcinoma of the uterus. (4m)

A

Mangement (ideally after TAH+BSO+LN sampling) is stage directed:
With key factors:
Invasion (myometrial, stroma, pelvis), favourable (endometriod endometrial adeno grd I-II) vs aggressive subtype (grd III, papilary serous, clear cell, small cell), grade (FIGO Grade=%solid non squarm), LVSI (focal vs extensive, extensive is now stage 2), Node status (+ve = at least stg III), molecular - POLEmut downstages Stg1any to 1A. P53abn any II to IIC.

Stg1A = <50%myo, grd I-II non agressive, no/focal LVI = observation
Stg1B = >50%myo and otherwise 1A = VBT (PORTEC2)
Stg1C = Pelvic RT alone (or consider VBT) PORTEC2

II-IV: ChemoRT + adjuvant chemo (PORTEC3) + VBT if cervix stroma involvement

How well did you know this?
1
Not at all
2
3
4
5
Perfectly
100
Q

Briefly describe the macroscopic and microscopic features of uterine serous carcinoma

A

Macro: Arise on the surface of polyps on background of atrophic endometrium.
Micro: Features of necrosis, with Psamomma bodies!! Typically myo and vasc invasive. Complex architecture - papillary, glandular, solid. IHC: ER/PR-, Ck 7+ CK20-

How well did you know this?
1
Not at all
2
3
4
5
Perfectly
101
Q

How does uterine serous carcinoma differ in its mode of spread from endometroid carcinoma (1m)

A

Endo:
Less aggressive
Spread to lymph nodes and ovaries

Serous:
More aggressive
Spread to peritoneum (need peritoneal Bx) and nodes

How well did you know this?
1
Not at all
2
3
4
5
Perfectly
102
Q

Describe the FIGO histological grading system for endometroid adenocarcinoma (2m)

A

Based on the degree of glandular differentiation, which is described as % non-squamous solid growth pattern
G1 5% non-squamous solid growth pattern
G2 6-50% non-squamous solid growth pattern
G3 >50% non-squamous solid growth pattern (i.e. more sold, less glandular)

How well did you know this?
1
Not at all
2
3
4
5
Perfectly
103
Q

What is Lynch syndrome?

A

Hereditary non-polyposis colorectal cancer (HNPCC):
Autosomal dominant
Caused by germline mutation in any one of the mismatch repair gene s(MSH2, MLH1, MSH6, PSM2), or deletion of the last few exons of the gene EPCAM that results in epigenetic silencing of MSH2
Associated with 20-80% risk CRC, >30% risk of endometrial cancer, 20%Ovarian, 10% urinary tract, 1-5% pancreas, gastric.

How well did you know this?
1
Not at all
2
3
4
5
Perfectly
104
Q

How do you test for Lynch syndrome in a hysterectomy specimen?

A

If specimen is deficient for MMR protein then perfom PCR: MLH1, MSH2, MSH6, PMS2, and EPCAM

How well did you know this?
1
Not at all
2
3
4
5
Perfectly
105
Q

A fit 70 year old woman is referred to you following TAH+BSO for a FIGO Stage IB Grade 3 endometroid carcinoma of the uterus (the tumour is confined to the uterus and invades through more than half of the myometrium). Pelvic node dissection has not been performed. - What further treatment would you recommend? Justify your answer (2m)
Give target volumes.

A

Grd 3 now (new staging) makes stg 1C (assume POLEmut -)

As per PORTEC 3 and GOG 249: Pelvic EBRT alone for LCR benefit 3yr (reduce recurrence from ~15-20% to 5% or less).
45/25 to:
CTVp = upper half vaginal + parametrium + para-vaginal soft tissue.
ITVp = Consider ITV defined as the combined volume of CTVp in both the fused empty and full bladder CT scans
CTVnodes: external (and obturator) and internal and presacral - just like cervix dont do common iliacs.
PTV=CTVs+ 1cm.

How well did you know this?
1
Not at all
2
3
4
5
Perfectly
106
Q

Isolated vaginal recurrence of endometriod endometrial adenoC with no metastatic disease
- Describe a suitable radiation technique and dose prescription for this woman’s treatment (4m)

A

Observation data supports:
Pelvic RT to a dose of 45Gy in 25#s, 1.8Gy/#, 5#/wk (concurrent weekly cisplatin 40mg/m2) followed by adjuvant vaginal vault brachytherapy 28Gy/4.

  • Immobilisation: vac bag, knee fix, ankle support
  • Planning CT: 2mm slice mid-abdo to mid-thigh, w/full and empty bladder. Fusion: with all available imaging (MRI and PET)
    Target volume:
  • GTV = gross recurrence on vaginal apex based on imaging and clinical examination
  • CTV45 = GTV + 1cm + parametrium + paravaginal soft tissue + at risk nodal group (i.e. obturator, internal iliac,
    external iliac, pre-sacral LN)
  • ITV45 = CTV45 contoured on full and empty bladder scan
  • PVT45 = ITV45 +1cm
How well did you know this?
1
Not at all
2
3
4
5
Perfectly
107
Q

This patients has Stage IB Grade 2 endometroid adenocarcinoma.

Assuming there are no other adverse features, what would you estimate her risk of vaginal and nodal recurrence to be?

A

Vaginal recurrence ~15%
Nodal recurrence ~10%

(for own notes: PORTEC-2 – 5-yr VR 2% and 5-r LRR 5%)

How well did you know this?
1
Not at all
2
3
4
5
Perfectly
108
Q

This patients has Stage IB Grade 2 endometroid adenocarcinoma.

Outline the potential further management option for this patient. Indicate your preferred option and justify your answer. Outline the potential toxicities of your treatment.

A

Intermediate Risk:
Vaginal vault brachytherapy (VBT) (preferred option)
- Preferred option as per PORTEC 2
- No differences in vaginal recurrence, disease free survival and compared to pelvic
external beam RT but associated with lower toxicity
Toxicity:
- Acute: procedural discomfort, vaginal mucositis
- Late: vaginal atrophy/ dryness/ stenosis, cystitis, proctitis, 2nd malignancy

Adjuvant pelvic external beam RT: Would use if other risk fctors - i.e stage Ic = RT alone (PORTEC3)

Observation: Same OS, but higher %15 LR (name numbers for ball cancer)

How well did you know this?
1
Not at all
2
3
4
5
Perfectly
109
Q

Mx options for endometrial recurrence post adj RT:

A

MDM discussion

Referral to medical oncology
- Medroxyprogesterone (Provera®) if ER/PR+ve
- Palliative chemotherapy if fit for chemo
- Pembro - PFS benefit for stg IV

Palliative care
- for psychosocial support and ongoing symptom management

How well did you know this?
1
Not at all
2
3
4
5
Perfectly
110
Q

A fit 70 year old woman presents with post-menopausal bleeding. She is diagnosed with endometrial cancer and undergoes a total abdominal hysterectomy and bilateral salpingo-oophrectomy. Pathology demonstrates a Grade 3 endometroid adenocarcinoma with 10 of 12mm myometrial invasion and lymphovascular invasion. Cervical stromal involvement was noted. 9 pelvic nodes were negative for disease (FIGO 2021 Stage II disease).

a. What are the possible treatment options for the management of this patient? Provide the rationale for the preferred option

A

Concurrent chemoRT (cisplatin) 48.6Gy/27# + vaginal brachytherapy (BT) 10Gy/2# (given cervical stromal involvement) + adjuvant chemotherapy (carboplatin/ paclitaxel) (based on PORTEC-3) – my preferred option
- improves overall survival, failure free survival, and reduce distant metastases
- Pt is in lower risk group than PORTEC-3, but should have discussion regarding the risk/benefits of concurrent and adj chemo.

Adj chemoRT + VBT, no adj chemo - inferior outcomes compared to full PORTEC-3, but spare some toxicities of
adj chemo.

Adj pelvicRT alone (no chemo)
- inferior oncological outcomes compared to full PORTEC-3 regimen; however reasonable option to reduce the risk of LR

Adj VBT alone 21Gy/3#
- not inferior to pelvic RT alone (with lower toxicity as per PORTEC-2);

If contraindicated for RT
- refer to med onc for discussion re: adjuvant chemo alone (GOG-258)
- pt actually slightly lower risk than GOG-258

Observation, with close clinical surveillance –reasonable risk of LR (?30%)

How well did you know this?
1
Not at all
2
3
4
5
Perfectly
111
Q

Endometrial Stg II.
A decision is made to deliver pelvic radiation therapy with a vaginal brachytherapy boost. Describe a suitable technique and dose fractionation schedule (4m)

A

I will offer this patient pelvic external beam radiotherapy to a total dose of 45/25 1.8Gy/#, 5#/ week and vaginal vault brachytherapy boost 10Gy/2#, 2#/ week.

CTVp = upper half vaginal + parametrium + para-vaginal soft tissue
* ITVp = combined volume of CTVP as contoured on full bladder and empty bladder scan, taking into account
bladder/ rectal position
* CTVn = at risk nodal group (obturator, external iliac, internal iliac and pre-sacral node)
* PTV = ITVp and CTVn +1cm
Technique: VMAT technique with 6MV photon Concurrent chemotherapy: cisplatin 50mg/m2 3-weekly.

Vaginal vault brachytherapy component
* Position: supine, use largest vaginal cylinder that will fit comfortably to minimise air pocket
* Target volume: upper half of vagina
* Dose/fractionation: 10Gy/ 2#, 5Gy/#, 2#/ week
* Technique: HDR brachytherapy with Iridium-192 radioactive source, prescribed to 5mm from surface.

How well did you know this?
1
Not at all
2
3
4
5
Perfectly
112
Q

Vaginal stenosis is a common complication following treatment for gynaecological cancer. Describe potential prevention and treatment strategies (2m)

A
  • Use of topical oestrogen cream – may promote vaginal mucosa epithelial regeneration
  • Use of vaginal dilator, start 2-4 weeks post completion of treatment, 2-3 times/ week, for 5-10 minutes each time
    – break down adhesion and stretch vaginal wall to maintain patency
  • Maintain regular sexual intercourse in women who are still sexually active
How well did you know this?
1
Not at all
2
3
4
5
Perfectly
113
Q

A 40yro is referred following incomplete resection of a function pit adenoma (lactotroph) involving the right cavernous sinus. What would be critical in your history and examination?

A

Cranial examination: detailed focus on CNIII, CNIV, CNVI and V1 and V2.
Also optic chiasm
Sx:
Prolacinaemia: impotence/decreased sex drive (Galactorrhoea extremly rare in males)
Diplopia (e.g abducens palsy or CNIII palsy)
Bumping into things (may suggest bitemopral hemanopia = chiasm)
Scalp/forhead/facial numbness.

Ex:
Fields: e.g temporal hemanopia
Eyemovements: CNIII - Down and out, CNIV, Abducens = diplopia.
Sensation: V1 and V2

How well did you know this?
1
Not at all
2
3
4
5
Perfectly
114
Q

What is the role of external beam radiation therapy in heterotopic ossification of the hip? Include in your answer: (3m)
The indications for treatment
The optimal timing of treatment
The target volumes and
Dose fractionation schedules
What are the alternatives to radiation therapy?

A

Indication - Recurrent HO
Timing - <24 hour pre-op, or <72 hour post-op
Target volume - HO bone island + 1-1.5cm margin to PTV
Dose fractionation - 7Gy/1# , AP/PA dose to midline, spare a strip of soft tissue
Alternative - Surgery followed by NSAID (indomethacin 75-100mg/day) for 7-14 days post-op

How well did you know this?
1
Not at all
2
3
4
5
Perfectly
115
Q

List tumours that arises in the sellar region (1.5m)

A

Benign:
- Pituitary adenoma,
- Rathke cleft cyst
- Craniopharyngioma
- arachnoid cyst
- cavernous sinus cyst

Malig
- Pituitary carcinoma
- Glioma
- Malignant Craniopharyngioma
- Germ cell tumour
- Chordoma
- Metastases

How well did you know this?
1
Not at all
2
3
4
5
Perfectly
116
Q

Compare pituitary tumour arising from somatotroph and corticotroph in regards to:
i. Clinical features (1.5m)
ii. Laboratory findings (1m)

A

Somatotroph:
Headache/ bilateral hemianopia
- Acromegaly, frontal bossing,
macroglossia, prognathism, spade- like hand, LV hypertrophy, enlarged visceral, hypertension, diabetes/ insulin resistance

Corticotroph:
- Headache/ bilateral hemianopia
- Cushing’s disease (hypertension, central obesity,
moon face, buffalo hump, hirsutism, weak muscle strength, skin striae/ stretch marks) osteoporosis, impotence, oligomenorrhea

Labs Somato:
- Elevated GH/ IGF-1

labs Cortico:
- Elevated 24 hour urine cortisol
- Elevated serum cortisol on dexamethasone suppression test
- Elevated cortisol on late salivary test

How well did you know this?
1
Not at all
2
3
4
5
Perfectly
117
Q

c. What is the pituitary ‘stalk effect’?

A

Hyperprolactinaemia from tumour mass at the pituitary gland/ stalk that block delivery of dopamine from hypothalamus to prolactin secreting cells in the pituitary

How well did you know this?
1
Not at all
2
3
4
5
Perfectly
118
Q

Describe the pharmacological management of the three most common functional pituitary tumours (2m)

A

Prolactinoma - Dopamine agonist (bromocriptine, carbergoline)
- >50% will show reduction in prolactin, and 80% will show >25% reduction in volume

GH-oma - Somatostatin analogue e.g. Octreotide (while awaiting surgery) - GH receptor antagonist (pegivsomant)

ACTH-oma - Ketoconazole

How well did you know this?
1
Not at all
2
3
4
5
Perfectly
119
Q

A 30-year-old female presents with headache and diplopia. A CT brain shows a 2.5cm sellar mass with involvement of the cavernous sinus.
a. What further information from her physical examination

A
  • Visual field testing – bilateral hemianopia
  • Cranial nerve examination (for assessment of CN involvement from cavernous sinus
    extension III, IV V1, V2, VI)
  • Signs of hormone hypersecretion
    o Lactotroph – hypogonadism, galactorrhea,
    o Somatotroph – acromegaly (frontal bossing, macroglossia, prognathism (projecting
    lower jaw), large hand and feet, coarse skin, enlarged viseral
    o Corticotroph – Cushing’s syndrome (weight gain, truncal obesity, moon face, buffalo
    hump, striae), hypertension, glucose intolerance, osteoporosis o Gonadotroph – precocious puberty
    o Thyrotroph – tremor, palpitation, fine brittle hair
How well did you know this?
1
Not at all
2
3
4
5
Perfectly
120
Q

What would you tell the patient about the expected outcome of radiation to a partially resected non-function pit adenoma? What potential treatment toxicities would you discuss with her? (2m)

A

Local control Radiographic response based on size ~90-100%

Acute:
- Fatigue
- Skin dermatitis, alopecia
- CNS: headache, nausea, vomiting, worsening neurology
- Eye: corneal irritation, dry eye Late:
- pan-hypo-pituitarism (most important) – up to 80% at 10 years
- optic chiasm injury
- cerebrovascular event
- 2nd malignancy (e.g. meningioma)

How well did you know this?
1
Not at all
2
3
4
5
Perfectly
121
Q

A decision is made to give adjXRTpartially resected non-function pit adenoma. Describe a suitable radiation therapy technique and dose fractionation schedule. (4m)

Make it SRS

A

Stereotactic radiosurgery of 16Gy (20Gy functional, 25/5 SRT) in 1# to improve LC
Pre-SIM
- Baseline hormonal profile
- Baseline ophthalmology + neurology + endocrinology review
- Post-op MRI SIM
- Position: supine, arms down, head towards gantry
- Immobilisation: thermoplastic mask/ stereotactic frame, vac bag, knee fix, ankle support
- Planning CT: 1mm slice planning CT with IV contrast Fusion: diagnostic and post-op MRI
Target volume:
- GTV = residual tumour on post-op MRI
- CTV = GTV
PTV = GTV + 1mm (3mm if SRT)

How well did you know this?
1
Not at all
2
3
4
5
Perfectly
122
Q

Oars for treating something in the middle of the chest (e.g. esophagus):

A

Spinal cord Dmax 45Gy,
Lungs V20Gy<20%,
Heart Kidney V40Gy<20%, consider split renal function if mean dose to one kidney >15Gy,
Liver V30Gy<30%,
Large bowel V15Gy<120cc.

How well did you know this?
1
Not at all
2
3
4
5
Perfectly
123
Q

a. With regards to Barrett’s oesophagus, describe:
i. Macroscopic and microscopic appearance (1m)

A

Macro:
red velvet-like “salmon flesh”

Microscopic
- Metaplasia: Replacement of non-keratinized stratified squamous epithelium with ‘intestinal- type’ columnar epithelium with goblet cells
- Dysplasia: nuclear polychromasia, irregular nuclear size and shape, prominent nucleoli, high N: C ratio, increased mitotic activity

How well did you know this?
1
Not at all
2
3
4
5
Perfectly
124
Q

a. List common posterior fossa tumours in children (1m)

A

BEAM (Brain stem glioma, Ependymoma, Astrocytoma, Medulloblastoma)

How well did you know this?
1
Not at all
2
3
4
5
Perfectly
125
Q

b. What are the microscopic features of classic ependymoma (2m)

A
  • Cellular neoplasm with sheet like growth pattern
  • Presence of perivascular pseudo-rosette (ependymal cell surrounding vascular lumen, which creates a nucleus
    free zone around the lumen)
  • Ependymal rosette (present in ~ 10% of cases)

IHC: GFAP+ (i.e.glial origin), EMA+(i.e. endothelial differentiation), S100+, vimentin +

How well did you know this?
1
Not at all
2
3
4
5
Perfectly
126
Q

a. With intracranial germ cell tumour (ICGCT) in adults, what are the most common symptoms and physical
examination findings? (2m)

A

Usually arise from proximal 3rd ventricle more commonly than pineal:
Pineal - Compression of 3rd ventricle -> hydrocephalus, Nearby structures include optic pathway (LGN, radiations) causing visual Sx.
Suprasellar:
Classic triads: diabetes insipidus + precocious puberty + visual deficit (bitemporal hemianopia)
Hypothalamic/ pituitary dysfunction
* Diabetes insipidus due to reduced ADH
* GH abnormalities: delayed/ precocious puberty
* Isolated GH deficiency
* Hypothyroidism
* Adrenal insufficiency Ophthalmologic abnormalities

How well did you know this?
1
Not at all
2
3
4
5
Perfectly
127
Q

Regarding ependymoma of the spine. Outline: (2.5m)
i. Epidemiology
ii. Clinical presentation

A

i. Epidemiology – uncommon, bimodal age distribution (<5y/o, and 30-40 y/o)
ii. Clinical presentation – chronic back/ sacral/ lower leg pain; 1⁄4 may present with leg weakness or
sphincter dysfunction

How well did you know this?
1
Not at all
2
3
4
5
Perfectly
128
Q

Regarding ependymoma of the spine. Outlines: (2.5m)

iii. Histological subtypes and their common locations

A

iii. Histological subtypes and their common locations

Cervical/thoracic: spinal ependymoma, myc-amplified spinal ependymoma, subependymona, tancytic ependymoma.

Lower cord: myxopapillary ependymoma.

  • G1: Myxopapillary ependymoma – more commonly in spinal cord/ filum terminale
  • G1: Sub-ependymoma – most common in 4th ventricle
  • G2: Classic ependymoma - intracranial
  • G2: Tancytic subtype – usually in spinal cord
  • G3: Clear cell subtype – usually supratentorial
  • G3: Anaplastic ependymoma – intracranial
  • G4: Ependymo-blastoma - treat like glioblastoma
How well did you know this?
1
Not at all
2
3
4
5
Perfectly
129
Q

Briefly discuss the prognostic factors in ependymoma (2m)

A

Pt: Age - see agreesive subtypes and spine location more common in adults (e.g. spinal myxopapillary ependymoma).
age <3 may limit options - but in some cases GTR + chemo equivalent to GTR+immeadiate RT
Males worse outcome
Tumour: Grade/aggressive subtype - anaplastic or worse ependymal-blastoma. Genetic: 1q gain, unfavourable molecular sub type.
Treatment: STR or worse unresectable.

How well did you know this?
1
Not at all
2
3
4
5
Perfectly
130
Q

A 36-year-old man presents with a 4 week history of progressive headaches and nausea. MRI brain demonstrates a mass arising from the 4th ventricle typical of an ependymoma causing early obstructive hydrocephalus.
a. Describe how you would further investigate and manage the patient. Justify your answer (2m)

A

Ix:
* Complete whole spine MRI for staging
* Lumbar puncture to ensure no spinal/ CSF mets (when safe i.e. when no increased ICP) !!!!!!! Can do 2 weeks post surg.

Mx:
Manage acute symptoms – analgesia, dexamethasone, ondansetron for headache/ nausea/ vomiting
Neurosurgical opinion:
- Shunting if required/ concern re: increased ICP
- Gross total resection of mass for tissue diagnosis and treatment

How well did you know this?
1
Not at all
2
3
4
5
Perfectly
131
Q

The patients undergoes surgical resection. Histology shows anaplastic ependymoma WHO Grade III. Post-op MRI shows minimal residual disease in the posterior fossa. Staging is otherwise clear
b. What treatment would you recommend for this patient? Include in your answer a suitable radiation therapy
technique and dose fractionation schedule including normal tissue dose constraints (3m)

A

Adj RT to the tumour bed alone to a total dose of 59.4Gy in 33 (if grd II then 54/30), 1.8Gy/#, 5#/week to reduce LRR. Aim to start RT within 6-week post-op. VMAT technique, 6MV photons prescribed to D50. Aim PTV D98% > 95%PD.

  • CTV59.4 = surgical bed + post-op residual disease + 1cm, clipped at anatomical boundaries
  • PTV = CTV + 5mm expansion
How well did you know this?
1
Not at all
2
3
4
5
Perfectly
132
Q

What investigation would you perform on a patient with a suspected Intracranial GCT? Justify your answer (3m)

A

Blood
AFP/ B-HCG (germinoma usually normal AFP/ HCG; NGGCT usually elevated)

Imaging
MRI whole brain/ spine: Spine mets are common in some subtypes (e.g. yolk sac).

Tissue diagnosis
Surgery for tissue diagnosis. Resection extent (if any), Subtype and grade guide management/dose.

How well did you know this?
1
Not at all
2
3
4
5
Perfectly
133
Q

What radiation therapy volumes you recommend for adult intra-cranial germinoma vs. non-germinoma tumour (3m)

A

Germinoma (1.5Gy/#)= WVRT+Boost, unless mets then CSI + GTV boost

NGGCT (1.5Gy/#)= CR/Pr = WVRT+Boost, unless mets or no response then CSI + GTV boost

WVRT = entire ventricle (include lateral, 3rd and 4th ventricle) and supra-sellar cistern + pineal cistern + pre-pontine cistern

CSI = CSI = entire brain parenchyma + thecal sac (down to ~ S2 level based on MRI)

Tumour bed boost =
GTV + 1cm = CTV CTV + 3mm = PTV

How well did you know this?
1
Not at all
2
3
4
5
Perfectly
134
Q

What radiation irradiation doses would you recommend for adult intra-cranial germinoma vs. non-germinoma tumour (3m)

A

Dose depends on response to chemo and presence of Mets.
Germinoma always 1.5Gy/#, and boost always 12Gy:
CR: 30/20 (18Gy whole vent, 12 boost)
PR: 36/30 (24 whole vent, 12Gy boost)
Met: CSI 24/16 Boost 45/25 (GTV+met)

NGGCT 1.8/Gy:
CR/PR: 30.4Gy/17 whole vent.

If non-response after chemo: CSI 36Gy/20#, 1,8Gy/#
Tumour bed boost to total 54Gy/30#

How well did you know this?
1
Not at all
2
3
4
5
Perfectly
135
Q

Bladder cancer: List the findings that should be included in the cystoscopy and histopathology reports.

A

Cystoscopy findings: Site, size, description of lesion (e.g. seated, flat), number of sites involved, tis/Cis, UO involved, Extent of Macroscopic resection, Overt muscle invasion/perforation, tissue taken.
Histo: Whether sample includes mucosa and muscle, macroscopic description of tissue and number samples collected, Histo: Tumour description and type and grade, extent of muscle/mucosal invasion, IHC as below.

How well did you know this?
1
Not at all
2
3
4
5
Perfectly
136
Q

ii. List the common immunohistochemical stains used to identify urothelial
carcinoma.

A

IHC: HMWCK+ (marker of urothelial origin), CK7+ Ck5/6 -ve

How well did you know this?
1
Not at all
2
3
4
5
Perfectly
137
Q

b. List the risk factors for the development of carcinoma of the bladder. (2)

A

Older age (>70),
smoking,
chronic inflammation (e.g stones, recurrent UTI, permanent catheters)
Immunsupression,
previous bladder cancer,
radiation exposure,
chemical exposure (e.g. cyclophosphamide),
congenital urogenital malformations.

138
Q

Describe 2 potential pathogenesis pathways for the development of multifocal bladder cancers. In what other tumour sites can this phenomenon occur? (3)

A

Monoclonal cell pathway: Transformation of a single cell, with further genetic alterations/mutations with replication progressing through hyperplasia or in-situ disease to invasive cancer with potential for INTRA-EPITHELIAL SPREAD and INTRA-LUMINAL SEEDING to multiple sites with ongoing growth.

Field-cancerization: Repeat cell injury/cytoxic exposure (e.g. recurrent UTI, chronic inflammation from IDC, cyclophosphamide), leading to genetic (e.g chromosome 9 deletion in bladder Ca) and epigenetic changes in cells at multiple sites (progressing through hyperplasia or dysplasia). Each such cell with increased probability of malignant transformation to invasive disease and therefore multiple sites (multifocal) of cancer. Besides bladder, Head and Neck, and oesophagus are other examples of field cancerisation.

139
Q

d. Regarding squamous cell carcinoma of the bladder, describe the: (2.5)
i. Potential pathogenesis.

A

SCC common pathogenesis: chronic inflammation/IRRITATION (e.g. permanent IDC), leading to increased proliferation and abnormal maturation of transitional cells with squamous metaplasia, the accumulation of mutations including p53 and gain of function mutations leading to increased growth, cell death evasion, further mutations and malignant transformation.

140
Q

d. Regarding squamous cell carcinoma of the bladder, describe the: (2.5)
ii. Biological behaviour

A

In comparison to urothelial carcinoma.
Biological Beh: faster growing and more aggressive than UC, more invasive, nodal spread common. Less responsive to systemic therapies and radiation

141
Q

A fit 72-year-old man presents with unsteadiness and headache, after surgery alone for localised lung adenocarcinoma. On imaging, he is found to have a solitary brain metastasis in the right cerebellum measuring 3cm in maximal dimension. His symptoms have mostly resolved with oral steroids.
What further information do you require in order to recommend a management
plan?(3)

A

Patient: Fitness for biopsy/resection, ECOG/KPS, symptoms suggesting metastatic disease (weight loss, neoplastic syndrome Sx).

Tumour: Details of previous cancer including actionable mutations (e.g. ALK/EGFR status), Biopsy or excision to define tumour, staging with PET to determine extent of disease. Other investigations: LDH, baseline (for systemic options) renal/liver/electrolytes.

Treatment: Options available in health system (e.g. gamma knife)

142
Q

A fit 72-year-old man presents with unsteadiness and headache, after surgery alone for localised lung adenocarcinoma. On imaging, he is found to have a solitary brain metastasis in the right cerebellum measuring 3cm in maximal dimension. His symptoms have mostly resolved with oral steroids.

There is no evidence of disease elsewhere. The patient undergoes macroscopically complete resection of the lesion, and histopathology is consistent with metastatic lung adenocarcinoma, ALK/EGFR/ROS-1 negative, PD-L1 TPS
<1%. Compare and contrast the management options following surgery. Include in your answer relevant information you would use to counsel the patient in making an informed decision.

A

This patient’s tumour lacks actionable mutations. With limited systemic options the argument for improving local control may be stronger.

Preferred: Adjuvant SRT to cavity 30/5. Broadly equivalent OS and LCR (around 70-80%) to WBRT (both WBRT and SRT do not improve OS), but RCT data suggests less/or delayed neurocognitive decline. Conformal therapy allows further SRT at any new sites. Disadvantage: Potential toxicity (generally very well tolerated) for no OS benefit, Resource intensive and may be unpleasant for patient.

WBRT (e.g 30Gy/10#): Adv: Improves local control following surgery, may treat disease not detected on imaging, less resource intensive (though more fractions for patient). Disadvantages: less dose at highest risk area,

Observation: Adv: avoid toxicities of radiation, progression may be caught early on surveillance, and XRT/further surgery. Dis: Significantly higher chance or recurrence (approx 40%LCR w/surgery), patient anxiety RE under treatment.

For asymptomatic disease the SOC is Osimertinib, for symptomatic surg/adjRT then Osi. Unaware of any data supporting adj systemic therapy alone following GTR. Not an option here as presumably lacks exon 19 or 21 EGFR mutations.

143
Q

The surgical cavity (post resection ofsolitary brain met) is estimated to measure 3.5cm on postoperative MRI. The
decision is made to treat the surgical cavity alone using a linear accelerator.
Describe a radiation therapy technique and dose fractionation schedule to treat
the surgical cavity alone, including on-treatment localisation.(3)

A

Adj SRT to the surgical cavity, VMAT technique, 6mV photons, 30/5 (<2cm do 27/3, >3cm 30/5), Prescribed to 100% isodose line. Aim Max within CTV 125-140% prescribed dose (130% typical IMRT).
Sim: patient supine, head towards gantry, arms by side, knee rests, comfort measures. Imo: thermoplastic mask.
Contrat CT 1mm fine slice - vertex to C4. Planning MRI T1+C.
HighRiskTumourVolume=HRTV=Surgical cavity
CTV = HRCTV + any GTV + 2mm (may include tract and/or dural margin).
PTV: =GTV +2mm
OARS: in 2Gy/# (I would convert for SRT): Brain stem <45Gy, Brain Dmax<60Gy, V40Gy<5cc.
Verification: daily CBCT matched to bone

144
Q

i. Discuss the effects of ionising radiation on the foetus in the first trimester, and how these are known.

A

During the organogenesis phase of foetal development the foetus is at most risk from radiation. Studies of atomic bomb survivors, and other disaster exposures, combined with animal studies demonstrate:
A threshold dose of 0.1Gy
W/general features of Interuterine growth restriction, Cognitive impairment, and future malignancy risk being proportional to dose and also the phase of development at which exposure occurred:
- Exposure during pre-implantation phase (day 0-10) - often fatal/failed implantation.
- Organogenesis phase (up to 6 weeks): Highest risk of congenital abnormalities, especially CNS (microencephaly). Severe intrauterine growth restriction (due to cellular depletion).
- Foetal period 6 weeks to 9 months: 1) Mental retardation - 25 IQ points per Gy, threshold 0.1Gy, due to failure of neurons to migrate peripherally. 2) Microcephaly 3) Eye, skeletal, genital abnormalities 4) Intrauterine growth restriction

145
Q

How would you evaluate the dose received by the foetus in this patient?

A

1 - Calculate length/duration of exposure (e.g 18 fractions)
2 - Dose to foetus: Will be a combination of field size and proximity to field - with sources of radiation from leakage from collimator head, external and internal scatter. Can be estimated roughly using phantom and dosimeters.

146
Q

iv. What exposure levels are relevant when advising the patient on how to proceed with the pregnancy?

A

Not just levels but exposure within early 1st trimester. A above for every Gy 6% increased risk of malignancy, severe mental retardation over 2Gy, with significant growth disturbance over 0.2Gy.

147
Q

b. A 36-year-old woman presents with a mobile, 3 cm breast mass at 22 weeks gestation. Biopsy of the breast lesion confirms malignancy. Discuss how does pregnancy impacts staging and treatment decisions.(5)

A

Staging: Avoid significant radiation exposure from PET/CT/Bones scan. CXR with shielding ok. Alternative imaging: MRI bilateral breast/Axilla/bone, USS liver, CXR chest. (NB Mammogram with shielding considered safe)
Decisions for each modality are effected:
- Surgery (safe after 12 weeks): Not yet in 3rd trimester (when could give post part XRT), so mastectomy and ALND preferred over BCS. Some nodal tracers from SLNBx are teratogenic.
- Chemo (after 14 weeks): Significantly lower risk than 1st trimester. Avoid taxanes. Systemic chemotherapy with 5-fluorouracil, doxorubicin, cyclophosphamide (FAC) has been used in pregnancy
- Hormone therapy: Avoid Herceptin during pregnancy, delay hormone therapy/ovarian suppression till postpartum.
- Radiation: Perform post partum (i.e. delay) - e.g.If after BCS perform or indicated by mastectomy findings.

148
Q

A fit pt and has widespread bone metastases. She is symptomatic at the T10 site. There is no spinal instability, spinal cord compression or previous
radiation at the site.
What are her options for radiation therapy treatment? Justify your answer and provide likely response rate for each option. (3)

A

Preferred option if single symptomatic site and no risk cord compression at other sites: 20Gy/5#s to T11-T12, EBRT 3D-planned for durable (>1 year) control of progression and symptoms. Large meta-analysis supports 8Gy/1# as equally effective at 1 year, however that analysis shows higher retreatment rates w/single #s. In this case, where multiple lesions may need treatment in the future I would like to avoid retreatment if possible.

Other options:
Commence systemic therapy, and treat as above if symptomatic.
SABR to single symptomatic met not appropriate in setting of extensive disease elsewhere. Mixed data as to benefit over multifraction EBRT.
Multi-modal analgesia alone or in combination with systemic therapy.

149
Q

Breast cancer.
C. In the setting of bone only metastatic disease, discuss the different classes of systemic therapy available for use.
Include in your discussion of each (2):

  • The rationale/considerations for use of this class.
  • Give one example of a drug within the class.
A

Hormone Therapy: Indicated if receptor status positive - If pre menopausal then anti-estrogen (e.g Tamoxifen) or GnRH agonist, or post menopausal Aromatase inhibitor (e.g. anastrozole). In this setting RCT data support delay progression, with increased OS and provide symptom relief/less bone pain. If pre-menopausal then include ovarian suppression (GnRH agonist)
Palliative Chemotherapy - To reduce systemic tumour burden for progression free survival and OS benefit. E.g AC chemotherapy (Doxorubicin + Cyclophosphamide). Consideration should be given to toxicities, pt fitness and previous chemo therapy
Monoclonal antibody therapy - If tumour expresses target (E.g HER2), e.g Herceptin and pertuzumab. OS and PFS benefit in metastatic setting.
Molecular therapies – Targeting growth/signaling pathways – E.g Palbocilib for ER+ metastatic cancer. OS benefit uncertain, PFS and arguably QOL benefit.

150
Q

In general, what factors do you consider when advising the medical oncologist whether there needs to be a break in the systemic treatment when palliative radiation is delivered (2)?

A

Patient factors: Frailty leading to increased toxicity/poor toleration of treatment, pt wishes (e.g. impact of symptoms on life)
Tumour factors: growing tumour insensitivity, alternatively if tumour control is better with systemic therapy, potentially should not compromise it.
Treatment: Overall treatment time, timing with chemotherapy. Consider risk of tumour lysis syndrome with concurrent treatments. Radiosensitizing chemotherapy. Where risk of concurrent treatment not well defined (e.g. most immunotherapies)

151
Q

List the differential diagnoses for a lesion arising in the sellar region. (1)

A

Sellar lesions:
Pituitary adenoma (anterior/posterior),
meningioma,
benign/malig nerve sheath (e.g optic) tumour, craniopharyngioma,
glioma,
lymphoma,
Bengin: Congential cyst, AVM. Rathke cleft cyst.
Mets.

152
Q

Compare pituitary tumours arising from lactotrophs and corticotrophs in regard to:
Clinical Sx & Laboratory Findings

A

Both can present with similar sx of mass effect: E.g. bilateral hemanopia (or homonymous or quadrantinopia) or , head ache, cranial nerve symptoms (e.g. from Mekels cave invasion). Very Rare: CSF rhinnorhoea, pituitary apoplexy.

Lactotrophs: Women: galactorhoea, oligo/amenorhoea, infertility.
Men: Hypoganadism, infertility, erectile dysfunction, galctorehoea rare.
Bloods: demonstrate increased serum prolactin.

Corticotrophs: Anterior pit stimulate corticosteroid release (esp cortisol) – Can present with cushinoid symptoms: buffalo hump, moon face, hypertension, central weight gain, hypereglycaemia/insulin intolerance, Impotence, amen/oligomenorrhoea.
Bloods: Serum ACTH, 24hr urine free cortisol, Dexamethasone suppression test +ve (higher serum cortisol post dex).

153
Q

Regarding pituitary adenoma, what pathologic features may predict more aggressive behaviour following surgical resection? (1)

A

Invasion (boney sella, Mekels),
Significant residual in surgically inaccessible region (e.g. cavernous sinus),
Tumour features: high mitotic rate, higher grade/cellular atypia. AGGRESSIVE SUBTYPE – e.g. TSH-oma.
Not responsive to medical therapy – e.g. Prolactinoma not responsive to cabergoline.
Fast growing residual post surgery.

154
Q

What is the pituitary ‘stalk effect’ and what is the relevance to pituitary adenoma subtype? (1.5)

A

Unlike other ant pituitary –trophs, Hypothalamic innervation is inhibitory, whereby interruption of the hypothalamic-pituitary axis causes disinhibition of lactotrophs and subsequent increased release of prolactin (in contrast disruption this causes inhibition of release of other pit hormones).

Compression of pituitary stalk → interrupt the dopaminergic fibres (from hypothalamus to pituitary) that inhibit prolactin secretion → increased secretion of prolactin (hyperprolactinaemia)

155
Q

For both MEN1 and MEN2 outline (2):

1) Genetic mutation

2)Associated cancers

A

MEN1 – Autosomal dominant Mutation of MEN1 a tumour suppressor gene encoding the protein Menin.
Associated with: Medullary thyroid cancer, neuroendocrine pancreatic tumours and pituitary tumours . (mnemonic: 3P = pituitary, parathyroid, pancreatic islet cells)

MEN2 – Typically a mutation (gain of function) of RET proto-oncogene.
Associated with: AMYLOID PRODUCING Medullary thyroid cancer, pheocytocytoma.

156
Q

For medullary carcinoma of the thyroid gland, briefly describe the: (2.5)
Cell of origin

Microscopic and immunohistochemical features.

Clinical and pathological features that may indicate an underlying germline mutation.

A

Cell of origin:
Arises from Parafollicular C-Cells
Microscopic = Salt and peper chromatin, significant LVI, amyloid common.
IHC = Calcitonin +ve, TTF1 and PAX8 weak/mod pos. Thyroglobulin -.

Clinical/Path features suggestive of germline: production of amyloid, Younger age of onset (e.g. in 30s), multifocal and bilateral at diagnosis, other malignancy (e.g. MEN1 related pheochromocytoma) or hemangioblastomas (VHL).

157
Q

Components of EUA on cervical cancerworkup

A

Components of EUA to determine extent of disease – Pt in lithotomoy position: bimanual examination to fornices, digital rectal examination, Speculum examination +/- biopsy. Hysteroscopy and cystoscopy may also be included.

158
Q

For cervix cancer: Discuss the relative benefits of various imaging modalities.

A

USS (not needed) benefits: Commonly available, results may help speed up further workup, no radiation (e.g. if pregnancy), can detect tumour and suspicious soft tissue changes (e.g. parametrium)
MRI: Best resolution of soft tissues allowing pre-op staging of tumour invasion, and nodal involvement.
CT staging: For systemic staging, local disease and hydronephrosis. More accessible than PET
PET CT – Sensitive for local and systemic disease, allows more accurate staging

159
Q

In general, list the prognostic features that influence local control and survival in cervical cancer.

A

Patient: Functional status, immune suppression, older age (>60), socioeconomic status, smoking status, Hb<110

Tumour: Nodal status and T stage (i.e. invasive higher risk recurrence and systemic spread), M stage (ie. Survival and approach to local control), LVI, DOI, tumour differentiation (poor vs well diff). P16 IS NOT PROGNOSTIC IN CERVIX YOU TWAT, Cell histology: in order worse to best prog small cell, adenosquarmous, adenoCa, SCC

Treatment: Overall treatment time (aim complete within 8 weeks), concurrent chemo, brachy boost available.

160
Q

This patient is diagnosed with a squamous cell carcinoma of the cervix with a 2cm
para-aortic lymph node at the level of the third lumbar vertebrae (FIGO Stage
IIIC2). Definitive chemoradiation is recommended.
For the external beam component describe in detail (4)
i. Target volumes.
ii. Dose fractionation schedule.
ii. Organs at risk.

A

Curative intent EBRT VMAT, SIB technique, 6Mv photons, to primary and pelvis 45Gy/25#s, boost involved node to 54/25, 5#s/week. Prescribed to D50. Brachy boost in final weeks of treatment 24/3 (or 28/4) HighRiskCTV>90% PD.

EBRT:
GTVp = primary on imaging (CT/PET/MRI), GTVn = Involved nodes on imaging
CTV55 = GTVn +1cm.
CTV45= (GTVp + 1cm), cervix, uterus, upper ½ vag, parametrium. Combine contours from full and empty bladder to form ITV.
PTV45 = CTV45 +1.5cm to account for bladder/rectum varaiation.
CTVpelvis 45 = GTVn + Obturator, external, internal iliacs. If node + include eschelon above (up to para-aortic). Include inguinal nodes if lower 1/3 vag involved.

161
Q

Epidemiology of
Pleomorphic adenoma,
Muco-epidermoid cancer,
Adenoid cystic carcinoma

A

Pleo
* Most common benign salivary gland tumour
* M:F 1:2 (female in 40s)
Muco-epidermoid cancer
* Most common malignant tumour of parotid gland
* Most common RT-induced neoplasm
* Wide age range
* No sex predominance
* Age ~40
ACC
* Most common malignant tumour of
submandibular gland
* M:F 1:1
* Age 60-70

162
Q

A 71 year old man presents with a mass in the right parotid gland. Discuss the differential diagnosis and the relative frequency of each potential diagnosis (3m)

A

Benign
Pleomorphic adenoma (80% of all benign salivary gland tumour)
- Slow growing, risk of recurrence ~ 50%
- ~ 5% Risk of malignant transformation to carcinoma ex-pleomorphic Oncocytoma (1%)
Warthin’s tumour (6%)
Basal cell adenoma (2%)

Malignant
Mucoepidermoid carcinoma
* most common parotid gland malignant tumour
* most common RT-induced neoplasm (known dose-response relationship) Adenoid cystic (most common submandibular gland tumour)
* slow growing, long natural history, locally infiltrative, tendency for PNI, and distant mets (lung
mets)
Adenocarcinoma (5-10%)
Acinic cell carcinoma
* low grade, slow growing
Carcinoma ex-pleomorphic adenoma (4%) * high grade, aggressive
Primary SCC
Metastases (5%)
* mainly SCC, followed by melanoma Other: lymphoma, sarcoma

163
Q

Epidemiology of:
i. Muco-epidermoid cancer (2m)
ii. Adenoid cystic carcinoma (2m)
iii. Pleomorphic adenoma (3m)

A

Pleo:
* Most common benign salivary gland tumour
* M:F 1:2 (female in 40s)

Muco-epi:
* Most common malignant tumour of parotid gland
* Most common RT-induced neoplasm
* Wide age range
* No sex predominance
* Age ~40

ACC
* Most common malignant tumour of
submandibular gland
* M:F 1:1
* Age 60-70

164
Q

Micro and IHC of:
i. Muco-epidermoid cancer (2m)
ii. Adenoid cystic carcinoma (2m)
iii. Pleomorphic adenoma (3m)

What IHC marker is common to all?

A

CK7 for all
Any with a myo- component S100, SOX10, SMA.
Pleo:
* Triphasic tumour with mixed of epithelial (ductal), myoepithelial, stroma (chondromyxoid/ myxoid) component

Muco:
* Mixture of mucin- producing columnar cells and epidermoid (squamous), intermediate and clear cells
* more epidermoid = more aggressive

ACC
* Biphasic tumour with ductal and myoepithelial component
* 3 different architectural pattern: tubular (most favourable), cribriform, solid (most aggressive)
* Grading based on solid component (G1= no, G2<30%, G3 >30%)

165
Q

For primary adenoid cystic carcinoma of the salivary gland, describe the microscopic features (2m)

A
  • Biphasic tumour with ductal and myoepithelial component
  • 3 different architectural pattern: tubular (most favourable), cribriform, solid (most aggressive)
  • Grading based on solid component (G1= no, G2<30%, G3 >30%)
166
Q

e. What are the components of the Spinal Instability Neoplastic Score (SINS) score
and how is it interpreted? (2)

A

6 items scored 1 to 3. Score 7-12=potentially unstable, >=12 unstable. Score of 7 or more warrants surgical consultto assess for instability prior to any RT. SINS>12 is a contraindication to SABR.

1) Pain
2) Alignment:
3) Location: rigid=0, semi rigid, mobile, junctional
4) Vert body collapse
5) Posterior elements involved
6) Blastic, mixed, lytic

167
Q

What are the classes of immunotherapy and how do they work? Give an example of each type. (3)

A

1) Monoclonal antibodies:
Targeted to: immune checkpoints (e.g PDL-1, growth receptors to disrupt function or deliver a targeted cytoxic agent.
MABs are therapies targeted to an antigen (e.g Programed death Ligand). Targeting can:
Prevent normal function of target proteins such as immune checkpoints (e.g. inhibition of T-cell activity Pembroluzimab binds PD-1), or promoting growth signals (e.g. Cetuxumab binds EGFR).
Antigens can target a cell expressing high levels of an antibody to deliver a cytotoxic agent either chemo therapy or radiation (e.g. PSMA-Lutetium-177).
Can directly mediate cellular cytoxicity (ADCC) flagging a cell for immune destruction. E.g. Rituxmab binds CD20 activates compliment (amongst other mechanisms). Similar to Rituxumab Herceptin has multiple actions both impeding receptor (HER) function and antibody-dependent cellular cytotoxicity (ADCC).
2) Cytokines: Interferon - stimulates immune system (e.g. NK cells)
3) Vaccines: Prime immune system to target cancer proteins.
4) CAR T-Cell - Trained T cells re-introduced to target tumour cells

168
Q

Describe the side effects commonly associated with immunotherapy and how they are managed. (2).

A

Severe reactions are a life threading emergency and often require monitoring at 1st dose with acute reaction protocols often including:
High dose IV steroids (e.g. 120mg pred), immediate cessation of therapy (e.g. IV Nib), and acute Mx including fluids for distributive shock, or respiratory support, inpatient monitoring.

Limited reactions may be symptomatically managed w/little or ideally no steroids:
Skin - monitoring and topical therapies including (if needed low dose topical steroids),
Resp - broncho dilators, monitoring.
GIT - anti-emetics, loperamide.

All immunotherapies should have defined thresholds for cessation in the setting of symptom burden.

169
Q

Discuss at a cellular level, how the radiation induces the abscopal effect.

A

Tumour cell injury from radiation causes an increase in cellular debris both during necrosis and apoptosis. This leads to:

-Release (or loss) of various cytokines and hormones, which exert an endocrine effect on other parts of the body.

-Induce an immune response, sensitising the immune system to self antigens. These immune cells may then mount an autoimmune response against tissues elsewhere in the body.

-The pseudo-abscopal effect is seen in lymphomas, and is thought to be due to cells entering the field during treatment from other parts of the body.

170
Q

Using immunohistochemistry, how can you differentiate between adenocarcinoma, mesothelioma and small cell lung cancer? (3)

A

There are no single sensitive/specific marker for mesothelial differentiation: Tend to be WT1+, make keratin CK 6/7 +ve and TTF1 negative

Whereas both SCLC and Adeno are TTF1 + (well 90% of SCLC):
Small Cell: TTF1, synaptophysin and chromogranin-A positive
AdenoCa: TTF1 +ve (essential feature, opposite of SCC) and WT1 negative, both meso and adeno can be ck7+

171
Q

Define “paraneoplastic syndrome”.

Ii. How are these syndromes mediated?

A

Syndromes that arise from the products of tumours (e.g. antigens, hormones/growth factors), OR IMMUNE RESPONSE TO TUMOURS, causing a pathological behaviour in normal tissues (e.g Increased bone resportption and Hypercalcaemia) to produce a clinically detectable range of symptoms. They are not the result of mass effect from tumour growth.

Specifically mediated by either by chemical signalling molecules produced by tumours (E.g PTl) or by immune reaction to tumours.

172
Q

Briefly describe the methods used for molecular testing for HER2. (2)

A

Initial testing by IHC: Antibody stain for presence of HER2. If abundant (>50%) then 3+ and no further testing, if marginal (++) or less (+) then cytogenetic confirmation via FISH for HER2 gene (ERBB2), if on average 4 copies present then HER2+.

173
Q

How is microsatellite instability assessed? (1)

A

1) IHC for presence of MMR protein. If deficient then cytogenetic test (next gen sequencing technique) for MLH1, MSH2, MSH6, PMS2 and EPCAM silencing of MLH1.

174
Q

Bad prognostic cytogenetics/gene fusions for sarcomas:

A general non-prognostic cytogenetic traint of liposarcomas:

A
  • EWS-FL11 fusion transcript for Ewing sarcoma
  • SS18-SSX fusion transcript for synovial sarcoma (=worse)
  • FOXO1 translocation for alveolar rhabdomyosarcoma

Liposarcoma (e.g 40% of sarcomas are well diff liposarc)
MDM2 (12q15) is consistently amplified/ over-expressed

175
Q

Which sarcoma is particulary radiosensitive?

Which have a higher likliehood of recurrence?

A

Histopathological subtypes differ in their risk for local recurrence and responsiveness to radiotherapy.
Myxoid liposarcomas are relatively sensitive to radiotherapy and may have a significant reduction in size with preoperative radiotherapy.

Myxofibrosarcomas have a high propensity for local recurrence as they are highly infiltrative and have higher rates of positive margins after surgery. Given this increased risk of local recurrence associated with myxofibrosarcomas, radiotherapy is recommended to improve local control.

176
Q

35F patient. Preoperative radiation therapy treatment for thigh sarcoma. Describe a suitable radiation therapy treatment technique and dose fractionation schedule for this patient.

A

EBRT for improved local control: 50.4Gy/28#, 1.8Gy/#, 5#s/week. VMAT technique, >=6MV photons. Prescribed to D50.
Pre-sim: analgesia as needed. Preg test.
Sim: Supine, frog-leg position, legs towards gantry, comfort measures. Imo: vac bag, Knee brace, ankle support.
Planning imaging - 2mm CT non-con below knee to L5. Fused with planning MRI T1+C
Volumes: GTV on imaging, including bx tract and all oedema on T2MRI. CTV=GTV+1.5cm axial, and expanded 3cm cranial caudal (trimmed to anatomic boundaries). PTV=CTV+1cm
OARS: ALARA principles for pelvic organs. Avoid >50% Head/neck of femur receiving >50gy. Aim 50% femur<50% of prescribed dose.
Verif: Daily CBCT, with soft tissue review.
Aim: PTV D98 > 95%% prescribed dose. Aim DMAX within PTV and <107%, avoid hotspots over skin and bone.

177
Q

a. Breast cancers can be classified into 4 main molecular subtypes:
* Luminal A
* Luminal B
* HER-2 enriched
* Triple Negative (ER negative)
For each of the 4 main molecular subtypes:
i. What is the frequency in which they occur?

A

i. What is the frequency in which they occur?
Luminal A 50-60%
Luminal B 15%
Basal (tripple -) 15%
HER2 en 10%% (the rarest)

178
Q

a. Breast cancers can be classified into 4 main molecular subtypes:
* Luminal A
* Luminal B
* HER-2 enriched
* Triple Negative (ER negative)
For each of the 4 main molecular subtypes:
ii. What is the typical grade and immunohistochemical profile, including the proliferation index?

A

ii. What is the typical grade and immunohistochemical profile, including the proliferation index?

A = low ki-67 (<14%), Grd I-II, ER and or PR +, Her 2 neg

B = high ki-67 (>=20%), Grd II-III, Tripple (ER, PR, HER-2) +ve

HER-2 = high ki-67, often ER/PR-, HER2 +++

Trip Neg/basal = high ki-67, grd III, Triple - ve.

179
Q

a. Breast cancers can be classified into 4 main molecular subtypes:
* Luminal A
* Luminal B
* HER-2 enriched
* Triple Negative (ER negative)
For each of the 4 main molecular subtypes:
iii. What is the prognosis and typical biological behaviour?

A

Prognosis and typical biological behaviour:

A = Best prognosis, recurrence, nodal or distant spread unlikely. 5yr OS>90%.

B = Worse prog than A (though better than other types), occur in younger patient, higher chance of nodal spread.

HER-2 enriched = Poor prognosis. Aggressive growth and earlier spread to nodes (than luminal types), distant nodal spread, and haematogenous spread to brain, bone, lung.

Trip -ve/basal = Poorest prognosis. More aggressive growth Recurrence early, spread at earlier stage, heamatogenous spread more likely (preferred sites brain and lung). Higher response to anthracycline-based neoadjuvant chemo than luminal subtypes.

180
Q

Breast cancer molecular subtypes, What are the implications for systemic treatment?

A

What are the implications for systemic treatment?
All ER and or PR positive benefit from hormone therapy.Which affords a significant OS benefit (even at 2 years, especially if taken for 10yrs).

Postmenopausal women with small (<2cm, node-) luminal A tumours may not need RT if taking an AI (EXPERT Trial ongoing).

Basal subtype has higher benefit to anthracycline chemo

HER-2 positivity should be considered for HER-2 targeted therapy (Herceptin and pertuzumab)

181
Q

b. In carcinogenesis, what are the typical characteristics of a driver mutation? (1)

A

Mutations that promote proliferation and survival (clonal selection) over other cells in a population.
Can be in proto-oncogenes (e.g TK pathway RAS, RAF ect):
- A gain of function mutation increases the activity of the the pathway
- A loss of function mutation in a tumour suppressor gene which promotes inappropriate progression through cell cycle check points (e.g. p53 mutation).

For example driver mutation can occur in growth/tyrosine kinase pathway (e.g. KRAS) promoting proliferation, growth and apotosis evasion. Increasing the probability of further mutations (immune evasion, cdc activation/disinhibition) and potential for malignant transformation.

182
Q

c. For lung adenocarcinoma: (1.5)
i. What is the clinical significance of detecting an Epidermal Growth Factor Receptor (EGFR) mutation?

A

Predictive value: Can allow EGFR targeted therapies - e.g. Ozmetinib if Exon 19, or 21 mutation in NSCLC

Prognostic - The prognostic value of EGFR mutations is debated. (unlike in GBM for example where a poor prognaostic sign).

183
Q

2 examples of driver mutations? Include their frequency of occurrence.

A
  • BRAFV600E mutation common in melanoma increases increases activity of the tyrosine kinase pathway (e.g. EGFR) promoting growth and proliferation. Occur 50%melanomas
  • MYC amplification - seen in ~5% of all cancers.
184
Q

For NSCLC, besides EGFR, What are two other examples of driver mutations? Include their frequency of occurrence.

A

-KRAS (KRAS proto-oncogene) point mutation 20-30% adenoCa, most commonly in mucinous. Prognostic = poor, predictive of poor response to EGFR TKI.

ALK mutations (5%) Mutually exclusive with EGFR, e.g target with critotinib.

Others:
ROS-1 (ROS proto-oncogene 1) gene rearrangement (~1%). crizotinib

BRAFV600E 1-2%

185
Q

How does an ALK mutation promote oncogenesis?

A

ALK fusion proteins have a strong oncogenic potential, and it ultimately promotes tumor cell progression. ALK fusion happens when the ALK gene fuses (joins) with another gene, such as EML4.

ALK fusion protein activates downstream signaling pathways, including: MAPK/ERK, the (JAK-STAT), and PI3K– Akt.

Ultimately more growth signaling leading to more prliferation and survival.

186
Q

ii. Discuss the use of chemotherapy in uterine cancer including in your answer the rationale for its use. (2)

A

RCT Data (PORTEC3) demonstrates at 5years concurrent chemo (weekly cisplatin) followed by adjuvant x4 cycles (Carboplatin Paclitaxel) can provide LC, FFS and OS benefit when concurrent with radiotherapy. It may be considered from grd 3 stg1.
Sub-analysis of PORTEC3 shows the benefit of chemo is primarily in any of the following: Stage III (serosa or nodes involved), Papillary Serous or p53abn.

187
Q

WHO sub-type lung AdenoCa into:

A

Subtyping of adenocarcinoma into 5 main histological patterns
o Low grade: lepidic
o Intermediate grade: Acinar, Papillary
o High grade: Micropapillary, Solid

188
Q

Endometriod endometrial cancer, grd 2, 60%muscle invasion, not poleEdm. Adjuvant high dose rate vaginal brachytherapy in this patient. Describe a suitable radiation technique and dose fractionation schedule. (3 marks)

A

Assume stage 1B: Adjuvant vaginal vault Brachytherapy (Ir-192 source, remote after loader technique), 21Gy/3# to the upper 1/2 of vaginal vault prescribed to 5mm from cylinder surface.

Pre sim: Full discussion of procedure for consent. Analgesia, comfort measures as appropriate. Dilator education with educator (to begin 4 weeks post).

Sim: Patient in lithotomy position. Manual examination and selection of appropriate vaginal cylinder/cylinder segments. Aim largest size that is comfortable to minimise air gaps.
Plan selected from Plan library for applicator selected. Planning system to provide dwell times for afterloader.

OARS: Rectum, Bladder.

Verification: External measurement from labia.

Dose Delivery (in specialised bunker with video/audio monitoring, interlocks) via remote after loader.

189
Q

Endometrial Cancer
i. What are the indications for vaginal brachytherapy in the post-operative setting? (2 marks) .

A

Curative intent stage 1A: Brachytherapy alone (not favoured approach) for inoperative/declined operative intervention. 27Gy/3#
As part (adjuvant post surgery) curative intent treatment.
1) Stage 1B (>50% myometrial invasion, no LVSI, limited to uterus), without pelvic EBRT. E.g. 21Gy/3. (PORTEC2)
2) Stage II (extension beyond uterus where cervical stromal invasion has occurred) in addition (brachy boost) to pelvic EBRT. 10Gy/2#
3) Stage III and IVa. In addition (brachy boost) to pelvic EBRT. e.g. 10Gy/2# (PORTEC3).

190
Q

What immunohistochemical features allow you to distinguish between non small cell lung cancer (NSCLC), small cell lung cancer (SCLC) and mesothelioma. (1 mark)

A

NSCLC:
Aden IHC= TTF-1, Ck7, Napsin-A.
SCC = CK5/6, p63+, TTF-1 -ve.

SCLC: synaptophysin and chromgranin+, CK7-ve but 90% TTF-1+ve.

Mesothelioma: WT1+ve, ALSO CK5/6+, TTF1-ve, Ck7 negative.

191
Q

c. For squamous cell carcinoma and adenocarcinoma subtypes of NSCLC compare the following; (3 marks)
i. epidemiology & risk factors,

A

NSCLC=60% of lung cancers:
Adneno = 40% of lung cancers, more common in women and younger patients (compared to SCC), less association with smoking.
RFs: Smoking, occupational exposures (e.g. asbestos), previous lungCa, radiation exposure. Pulmonary fibrosis. Family history.
SCC: Epi - 20% of lung cancers, more common in males, strong association with smoking, most common Lung Ca in outside of western countries. Relatively older Pts.
RFs: Smoking, COPD, pulmonary fibrosis

192
Q

c. For squamous cell carcinoma and adenocarcinoma subtypes of NSCLC compare the following; (3 marks)
ii. macroscopic features

A

ii. macroscopic features
Adeno: Often arise more peripherally (than SCC/NSCLC), Solid tan-white, can have necrotic core, well-defined BUT not encapsulated.
SCC: Can be central or peripheral. Central tumours often invade into mediastinum structures/central airways. Peripheral often have nodular growth with central necrosis,

193
Q

c. For squamous cell carcinoma and adenocarcinoma subtypes of NSCLC compare the following; (3 marks)
iii. microscopic features.

A

iii. microscopic features.
Adeno: Glandular malignant cells with higher grades having more solid architecture. 5 histological subtypes across this spectrum: LAMPS = Lepidic (I.e. lepdic growth), Acinar (malignant gland invading stroma), papillary (papillary architecture w/fibrous core), Micropapilliary, solid (High-Grade).
There can me mucinous and non-mucinous subtypes.
SCC: Epithelial malignant/atypical cells with intracellular bridging and keratin (including keratin pearls). Further morphology described by 4 subtypes.

194
Q

d. List the targetable mutations in NSCLC. (1 mark)

Of these, describe 4 commonly encountered molecular mutations in adenocarcinoma of the lung and their clinical significance.(2)

A

Targetable mutations in NSCLC:
EGFR, ALK, BRAF V600E, ROS-1.

4 commonly encountered molecular mutations and their clinical significance:
EGFR mutations in: Exon 19 Exon 21. Predictive of response to EGFR targeted molecular therapies. Extent to which prognostic is debated.
ALK (found in 5%) - Commonly re-arrangement mutation leading to oncogenic fusion protein - > promotion of growth and cell survival. Targetable with Crisotinib. Both predictive and prognostic (worse prog). Mutually exclusive with EGFR mutations.
ROS-1 gain of function (growth signal pathway), rare (1% of patients), predictive (can target with crisotinib) and prognostic (worse prog).

195
Q

What are the key microscopic & immunohistochemical features of nodular sclerosing Hodgkin’s lymphoma? (2)

A

Micro: Reed-Sternberg Cells if present indicate Hodgkin’s lymphoma (except lymphocyte predominant nodular type) = binucleate with 2 prominent nucleoli, well remarked nuclear membrane, perinuclear halo, eosinophilic cytoplasm.
For nodular sclerosis type:
atypical mononuclear cells, RS cells, lymphocytes and eosinophils contained within bifringent sclerosis collagen,

IHC: CD15 and CD30, PAX 8+ve, (PAX5 weakly +ve) T-cell marker -ve.

196
Q

List the criteria that define favourable early stage Hodgkin’s lymphoma. (1)

A

Age<50, Mediastinal nodes not bulky (Not>1/3 mediastinal mass ratio), ESR not elevated (<ULN), No extra-nodal extension or >3 nodes. (A score of 1 or more = HR).

197
Q

In advanced stage Hodgkin’s lymphoma (Stage III – IV), the International Prognostic Score (IPS) may be used to stratify risk. (1)

i. List the criteria which constitute the International Prognostic Score.

ii. Briefly comment on how the total score may impact on overall survival.

A

The International Prognostic Score for advanced (Stg III/IV):
ALL-SHAM: Age>1, Leucocytosis, Lymphocytopenia, Stage IV, Hb<110, Low alb, Male

Score 0-1 90% 5yr OS, 2-3 = 80%, 4=60%, 5 or more 40% or less.

198
Q

Briefly describe the clinical, microscopic & immunohistochemical features of:
i.primary mediastinal B-cell lymphoma (PMBCL) (2)


A

Primary mediastinal B-cell lymphoma (PMBCL)
Clinical: Younger pts (mean 35), F>M. Can present with cough/shortness of breath/compressive Sx (e.g. SVC obstruction), dysphagia, and or B symptoms. Nodal spread uncommon at time of Dx.

Micro: Intermediate size B cell neoplasm with clustered or diffuse pattern.Clear cell change (abundant pale cytoplasm) and prominent sclerosis may be present - May mimic classic HL or DLBCL.
Immuno: CD19 CD20, CD79, PAX8, germinal centre markers may be present BCL2 and BCL6.

199
Q

Briefly describe the clinical, microscopic & immunohistochemical features of:
ii.nodular lymphocyte predominant Hodgkin’s lymphoma (2)

A

Nodular lymphocyte predominant Hodgkin’s lymphoma:
Clinical: Can present with B symptoms.

Micro: Popcorn (very lobulated nuclei) cells are characteristic. T-cells surround (“Cell rimming”).

Immuno: CD19, CD20, PAX8. (CD15 and 30 neg)

200
Q

A 44-year-old man presents with headache and imaging demonstrates a lesion consistent with a meningioma. What are the risk factors for the development of meningioma? (1)

A

Age - advancing age>50
Radiation exposure
Genetic syndrome: NF2 - lifetime risk 75% (But not NF1!!!), Gorlins, Li fraumi
Possible: skull trauma, hormone exposure.

201
Q

Regarding benign meningioma: (3)
i. What are the common intracranial locations?

ii. Describe the macroscopic appearance.

iii. Describe the microscopic appearance.

A

Tend to arise from arachnid villi (which contain high densities of arachnid cap cells) and therefore more common where these are highest density: cerebral convexities, parasagittal, parafalcine and sphenoid wing.

Macro: Well circumscribed, attached to dura, can be easily separated from brain, has dural tail. More aggressive disease may demonstrate invasion in to brain or bone.

Micro:
Synctal cells with indistinct membranes. Whorl type architecture. Psanomma bodies (concentric calcifications) are characteristic.
Grade based on Mitosis, Atypia, Invasion, Necrosis. (MAIN)

202
Q

c. Describe the WHO classification system for meningioma. (3.5)

A

Grade I to III. Is based on the presence or extent of Mitosis, Atypia, Invasion and Necrosis.
Grd I: (80% of mengiomas): NO Mitosis, no necrosis, No brain invasions. These are hypocellular. 13 Subtypes: Fibroblastic most common.

Grd II: (10%): Brain invasion OR frequent mitoses (>4/10xHPF) OR 3 or more atypical cellular features: Hyper-cellularity, high N:C ration, prominent nucleoli, pattern-less growth of focal necrosis.
There are 3 subtypes (ACC): Atypical (most common), Clear Cell, Chordoid

Grd III (<=5%): Very frequent mitosis (>20),OR melanoma, carcinoma, sarcoma features. Also 3 sub-types (RAP): Rhabdoid Anaplastic Papillary.
Molecular classification: TERT promotor mutation or CDKN2A/B may be grade III.

203
Q

d. On imaging, what features may indicate a higher grade of meningioma? (1)

i. Name the system used to grade the extent of resection in meningioma.

ii. What are the categories of resection described by this system?

A

Loss of well demarcated tumour borders. Adjacent cerebral oedema (may denote fast growth), brain invasion, multiple lesions. Adjacent bone lytic erosion.

Simpson grade.
Based on GTR (grades 1-3), along with dural tail Mx (resection or coagulation or none) and STR (4-5)
I - GTR with dural tail excision.
II - GTR with dural tail coagulation
III- GTR no dural tail Mx
IV - STR debulking
V - Bx only.

204
Q

Role/evidence of RT in oligmetastatic prostate cancer, give dose:

A

1) RT to the prostate in new Dx. 55/20 (STAMPEDE dose).
STAMPEDE found for <=3 sites treating the prostate improved OS
STOPCAT M1 meta-analysis (STAMPEDE +HORRAD) found OS benfit for <=4).

2) SABR-COMET (5 or less mets of most cancers, but not breast) Phase II - Very signif improved PFS and likely OS
Oriole and STOMP pooled analysis of phase II - SABR improves PFS

205
Q

Discuss the role of systemic therapy in the management of metastatic prostate cancer. Include in your answer the class of agents used, duration of therapy and justification for use.

A
  • Anti-androgen medications - Typically non steroidal anti-androgens (e.g. Bicalutamide): In hormone naive pts: part of initial therapy + GnRH agonist +/- abiraterone (e.g.STAMPEDE) increases progression free survival, and OS. Alternatively can be used for 3-4 weeks as part of commencement of GnRH agonist - to prevent testosterone spike from initial LH surge. Can also be used when progression despite GnRH agonist (“total androgen blockade) to prolong PFS. ADT+ Apalutamide superior DMFS (SPARTAN). Used life-long or until significant progression.
  • GnRH agonist (e.g Goserelin), provides an PFS, OS and symptomatic benefit in patients with newly Dx metastatic prostate cancer, and is part of multimodal therapy as above. Duration life-long, or until progression, or in the setting of stable disease intermittent (restart when >10ng/ml).
    Androgen synthesis inhibitor - Abiraterone: Improves OS and PFS (STOPCAT RCT)- used in the following contexts - as initial therapy for newly Dx metastatic disease (in combination with ADT but not ADT+Enzalutimide), or when progression on ADT.

Chemo - Typically Taxane chemotherapy - Docetaxel, improves PFS and OS (CHAARTED phase III RCT OS increase from 48 to 58months) when part of initial Mx of metastatic disease. Also increases OS for N+ and locally advanced (STAMPEDE). 75mg/m.sqr (65mg for Asians) until response (up to 10 cycles).

206
Q

Describe the FIGO histological grading system for endometrioid adenocarcinoma. (2)
t.

A

Prognosis is based on prognosis and TNM stage.
A Grd 3 endometrioid adenocarcinoma has approximately the same prognosis as an endometrial serous carcinoma.

Graded 1-3 Based on % of solid NON-squarmous component (vs glandular). <5% = I, <50%= II, >50% = III.

Grade !!!!!INCREASED by 1 if severe nuclear atypia is present.!!!!!!!

207
Q

A 44-year-old woman presents with vaginal bleeding. Clinical examination reveals a 5 cm ulcerative mass in the lower third of the vagina. Biopsy confirms a primary vaginal squamous cell carcinoma with no evidence of nodal or distant spread. The decision is made to treat the patient with curative intent. The first phase involves external beam radiotherapy with concurrent chemotherapy. Describe Volumes and OARS

A

Volumes:
For non-nodal volume contour on full and empty bladder and merge CTVs for ITV.
GTV = Primary tumour on imaging and examination.
iCTVp= GTV+5mm, Entire vaginal canal, and Cervix
PTVp = iCTVp+7mm
CTVn = Inguinal, external and internal iliacs (to bifurcation), obturator and presacral nodes (Pre sacral nodes if cervix involved).
PTVn = CTVn+7mm.

OARS:
Bladder V40Gy<40%
Rectum V40Gy<60%
Hips V40Gy<40%

208
Q

Vag Cancer:
b. Discuss the different radiation therapy techniques available to be used as a boost in vaginal cancer. Include in your answer the advantages and disadvantages of each technique. (2)

A

EBRT: Boost SIB: Advantages are reduced OTT (reduced treatment time for Pt, decreased re-population, less machine resoureces), Higher dose/fraction may suit some tumours. Disadvantages: Hypofractionation may have increased late toxicity due to reduced sublethal damage repair. Compared to brachy – much more integral dose, and dose to normal tissues, no dose escalation proximal to target.
Conventional boost: Advantage: less dose/fraction. Disadvantage longer OTT.
Brachy (initerstitial, cylinder, e.g. 12Gy/2#): Adv: Highly conformal , allows dose escalation adjacent tumour. Disv: invasive (especially interstial), more resource intensive, with risk procedural complications. Benefit over VMAT boost unclear.

209
Q

c. The Vag Ca patient is reviewed 6 months after her treatment is complete. She is disease free and presents with sexual dysfunction. List the possible causes and your management. (2)

A
  • Vaginal stenosis due to fibrosis: Mx beginning 2-4 weeks post treatment use dilators 2-3x/week for 5mins, maintain regular intercourse.
  • Dryness +/- opportunistic infection (e.g. yeast). Topical estrogen preparation, treat infection – e.g. fluconazole.
210
Q

d. The patient presents 2 years following her treatment with a faeculent discharge from the vagina. Outline your likely diagnosis and management of this.(2)

A

Likely recto-vaginal fistula. Requires examination +/EUA (consider recurrence), pelvic MRI and referral to gynaecology service:
May consider initial non-invasive Mx (bowel rest, hyperbaric therapy, pentoxyphilline) for very small fistula though much more common is surgical Mx – faecal diversion most common (spontaneous closure rare), other approaches (e.g. flaps) less established.

211
Q

n regards to secretory pituitary adenomas, explain how laboratory tests may help you to establish a subtype? (2 marks)

A

Lactotrophs: serum prolactin elevated.

Corticotrophs: 24hr urine free cortison. Serum ACTH, Dexamethasone suppression test.

Gonadotroph: LH/FSH elevation, with serum testosterone and Estrodiol elevation.

TSH-onma: Thyroid function test w/T3 and T4 - with elevated TSH despite high T3/T4

Somatotrophs: Serum growth hormone, insulin-like growth factor (IGF1) and fasting BSL - elevation suggestive of GH-onoma.

212
Q

Discuss the advantages and disadvantages of the different treatment strategies available for locoregionally advanced laryngeal carcinoma (Stages III and IV). (5 marks)

A

Surgery:
1) For stage III (T3 where thyroid cart not fully penetrated) - Larynx sparing surgery w/followed by PORT (no chemo). Advantage: Avoids chemo, equivalent to definitive ChemoRT (i.e VA study). Histology on resection provides more fine grained prognostic information/risk qualification and decisions on adjuvant therapies (e.g. the addition of chemo). Disadvantages: Worse reported QOL outcomes than definitive chemoRT, operative risks including failed organ preservation and dyspnonia/nerve injury. Does not avoid radiation.
2) For stage IV: Total laryngectomy and advent PORT +/- Chemo. Advantage: Definitive operation, with en bloc resection and detailed pathological information, preferred option for advanced disease. Disadvantages: Offers no chance of organ preservation, highly morbid surgery with poor QOL, intraoperative and post operative risks and complications, despite intensity of intervention may not be curative.

ChemoRT (not the role of induction chemo, e.g. VCF is controversial in terms of benefit vs toxicity, may consider for bulky disease where definitive chemoRT is treatment of choice).
1) Definitive For Stage III - 70Gy/35# with cisplatin (eg. 100mg/msqr)
2) as for 1, potential proceeded by induction chemo.

Palliative:
1) Palliative radiation for local control and symptom MX (e.g. 20/5 or 30/10): Not curative (main disadvantage), can offer good durable local control and symptom relief.
2) In severe case IV - Best supportive care, with medical Mx of Sx, and social emotional and family support.

213
Q

What are the clinical and radiological features that would suggest a diagnosis of HPV-associated oropharyngeal cancer?

A

Clinical – Hx: High risk factors: oral sex, multiple partners, younger age (<60), higher-SES. Low risk factors for non-HPV related (e.g. minimal EtOH, non-smoker, younger age).
Ex: Lesions originate from BOT or lymphoid tissue, whereas non-HPV related can occur anywhere.
Common presentation of HPV related is with a (cyctic) neck mass
Radiologic – Neck nodes tend to be cystic (e.g. Oropharynx 1st eschelon nodes (level 2) involved on CT PET). MRI demonstaring soft tissue mass or suspicious change in oropharynx at BOT or tonsils, cystic neck mass.

214
Q

List the cytological features that are seen with HPV-associated oropharyngeal squamous cell carcinoma.

A

HPV related oropharynx SCC – tend to be poorly differentiated, non-keratinised (the opposite of non HPV)
Strong and diffuse expression of p16

215
Q

What are the diagnostic criteria for p16 positivity?

A

> 70% cells demonstrate STRONG and DIFFUSE p16 staining

216
Q

Describe the key steps leading to HPV carcinogenesis. (2)

A

HPV infection (e.g. 16 and 18 are more oncogenic) -> Persistent infection of basal epithelia with viral host genome integration of viral oncogenes (in particular E6 and E7). These oncogenes promote checkpoint evasion at the G1/S checkpoint of mitosis. Oncogene E6 binds to tumour suppressor protein p53 and promotes it degradation. Normally p53 is upregulated in response to DNA damage (or other factors e.g oxidative stress) in turn activating p21 and promoting cell cycle arrest by Cyclin D-CDK inhibition. Oncogene E7 phosphorylates Rb protein preventing binding to E2F. Free E2F promotes progression through G1/S, increasing likliehood of mitosis when unrepaired mutations.

217
Q

Describe the implications of EGFR over-expression in head and neck mucosal squamous cell carcinoma. (2)

A

80-90% of H&N SCC have EGFR over-expression.
Prognostic: Poor prognostic sign, associated with more aggressive tumour. Due to increased growth signaling leading to increased proliferation and cell death evasion.
Predictive: Predicts an increased response to EGFR targeted therapy (e.g. Cetuxumab), though trials demonstrate only mild benefit of these therapies

218
Q

A well 50 year old man undergoes resection of a desmoplastic cutaneous melanoma medial to his right scapula. There is no disease beyond the primary site. Histology shows perineural invasion of multiple nerves up to 0.4mm diameter.
a.
i. What other features would you like to review on the pathology report when deciding on further management? (0.5)

A

Critical for marks:
- Depth of invasion (Breslow),
- Ulceration (always forget to fucking say ulceration),

Margin status (ideally >5mm radial),
LVI,
mitotic rate,
Tumour infiltrating lymphocytes.

219
Q

A well 50 year old man undergoes resection of a desmoplastic cutaneous melanoma medial to his right scapula. There is no disease beyond the primary site. Histology shows perineural invasion of multiple nerves up to 0.4mm diameter.

ii. If no other high risk features are present, what would be your recommended management and justify your answer? (2)

A

Desmoplastic melanoma is rare, and unlike most fully excised melanomas is associated with 50% local recurrence risk. PNI is also common in this sub-type. Observational data and a small phase II trial suggest significant LC benefit (10%) compared to historic controls. In the setting of immunotherapy this benefit is less certain.
Dose 30Gy/5#s, 2#/week.

220
Q

Axillary dissection reveals complete resection of melanoma in the affected lymph nodes.
b. What features in the pathology specimen could prompt consideration of adjuvant radiation therapy? (1.5)

A

As per the Burmeister study I would consider adjuvant nodal radiation if: ECE, >3 nodes or presence of a large node (>4cm) to improve local control from 60-80%.

NB: H&N 50/25 Parotid node = 1 then do, cervical node >=2 or >3cm then do.

221
Q

Melanoma patient. Adjuvant radiation therapy is recommended. Describe a suitable radiation therapy technique and dose fractionation schedule. Include expected side effects in your answer. (3)

A

Adjuvant EBRT (for local control) to the axilla 48Gy/20#, 5#s/week, VMAT technique, 6MV photons, prescribed to D50.
Pre-sim: assess for lymphoedema, consent to include discussion of increased risk.
Sim: Supine, arms above head secures/immobilised with arm board. Immobilisation with vac bag. Wire scar.
2mm Slice CT + contrast, C5 – L1. Bolus if skin if tumour spill, skin involved.
Volumes:
CTV48= Preoperative disease + surgical bed + 3cm sup-in expansion I would treat SCF nodes if +ve nodes in level III.
PTV = CTV48+10mm.
Verification: Daily CBCT

222
Q

Single 1.5cm melanoma lesion in the motor cortex, what treatment would you recommend and why? (1)

A

I would recommend SRT to brain 27Gy/3#.
Reasoning:

Eloquent brain - Morbidity associated with resection.

Tumour biology: melanoma radioresistant, prefer ablative approach.

Spares neurotoxicity of WBRT

Compared to WBRT allows further treatment if new lesions arise.

223
Q

A 40 year old male presents with a mass in the left testis. The ultrasound shows a 4cm malignant mass.
b. Outline the WHO classification of (2)

i. Germ Cell Tumours

ii. Sex Cord and Stromal Tumours of the testis.

A

i. Germ Cell Tumours
WHO classifies GCTs based on their origin – either from:
1) In-situ germ cell neoplasm: a. Single histology type = Seminomas (but not spermatocytic)
b. Non-semanomatous GCTs: Embryonal, yolk sac (post pubertal), Trophobalstic/choriocarcinoma, teratoma (post pubertal).
c. NSGCT with more than 1 histo (mixed)
d. Germ cell tumour, unknown type.
2) Unrelated to GCNIS:
Pre-pubertal seminoma
Pre-pubertal yolk sac

ii. Sex Cord and Stromal Tumours of the testis.
Into “pure tumour” - Leydig cell and Sertoli cell tumors. And mixed and unclassified sex cord tumour.

224
Q

Define a tumour marker.

A

Broadly, a tumour marker is any substance present in or produced by a tumour which may indicate its presence, activity or volume/burden or potential susceptibility to treatment. Though not by themselves diagnostic, tumour markers may be used in cancer screening (e.g. PSA for prostate cancer), to help guide diagnosis (e.g. beta-HCG for germ cell tumour), as part of risk stratification (beta 2 microglobin level for follicular lymphoma), or to monitor disease response following treatment (e.g thyroglobulin and RAI).

225
Q

What are the risk factors for the development of testicular germ cell tumours? (2)

A

Previous GCT (testicular or extragonadal),
family history,
cryptorchidism (with degree of descent correlated with risk – the less decent the higher the risk),
Dysgenesis syndrome (cryporchidism, hypospadias, poor sperm quality).
testicular development disorders (e.g. Kleinfelters/XXY and Down syndrome),
HIV,
white race

226
Q
  • macroscopic features
  • microscopic features for
    i. seminoma
A

Macro: “Cut potato” appearance – Solid white diffusely infiltrated/replaced and enlarged testicle. No significant necrosis.
Micro: “Fried egg” cells with round nuclei surrounded by clear cytoplasm. Cells in lobular configuration with fibrous septa.

227
Q

Ball cancer
* macroscopic features
* microscopic features for
i. embryonal carcinoma

Also give IHC

A

Macro: Irregular and invasive, poorly circumscribed. Does not replace the ball. Necrosis and haemorrhage.
Micro: Anaplastic large primitive cells, with very large nucleoli and overlapping nuclei
IHC: PLAP and Sall-4 +ve, AFP (70%) Beta-HCG (60%)

228
Q

A dude has a huge ball lump and elevated AFP.

What ball tumour does this exclude?
What if Beta HCG also raised

What if only Beta HCG was raised - and it was by a lot?

A

Not gonna be a pure seminoma (but 15% make BetaHCG).

Elevated AFP: Embryonal, and yolk sac

AFP+Beta HCG: most likely embryonal (70% AFP and 60% B-HCG) - Also consider mixed.

Very high Beta HCG - Choricarcinoma.

229
Q

What is the typical immunohistochemistry profile of a seminoma? (1)

A

PLAP+ve, SAL4+, CD117 (c-kit) +, AFP –ve (always), 15% may stain for B-HCG.

230
Q

Pathogenesis of hypercalcaemia of malignancy?

A

Due to the marked increase in osteoclastic bone resorption and release of calcium from bone also increased renal retention and GI uptake (PTHrP and calcitrol).
3 main mechanisms:
1) Ososteolytic mets -> local release of cytokines including osteoclast activating factors (commonly seen in multiple myeloma)
2) Tumour secretion of parathyroid hormone-related protein (PTHrP) = increased bone resorption and enhances renal retention of Ca (commonly breast, H&N and kidney ca, Lung SCC)
3) tumour production of Calcitrol leading to increased bone resorption and enhanced intestinal calcium absorption as noted in some lymphomas.

231
Q

A 70 year old man with metastatic lung cancer presents with confusion, nausea and constipation. He is found to have a corrected serum calcium of 3.2 mmol/L (normal range 2.2 - 2.6 mmol/L).
a. How would you manage this patient’s hypercalcaemia? (3)

A

Acute: IV fluids for dilution (e.g. bolus 1L, the 1L q4h, plan 4-6L/24hrs monitor for fluid overload), antimetics, plan for Mx of confusion (e.g. nurse special, private room, falls risk), ECG.

Admit to ward. Repeat bloods in 2 hours – assume with this presentation that hyperCa (moderate) will not be sufficiently addressed with fluid therapy.
Plan/Give bisphosphante infusion (zolodronic acid IV over 15 minutes), check renal function prior. Monitor Ca levels post – full effect typically 2-4 days.

Address cause of Hyper Ca where possible – i.e tumour debulking with RT and or chemo.

232
Q

What are the indications for the use of bisphosphonates in the treatment of the following primary cancers: (4)
i. Breast carcinoma

A
  • Metastatic breast cancer: monthly infusions in 1st year, then 3monthly thereafter. RCT demonstrates a benefit in terms of decrease skeletal events (compared to placebo) for patients in this group. More recent phase III RCT data suggests denosubab (Rank-ligand inhibitor) may be superior in terms of delaying time to 1st skeletal event.
  • Hypercalcaemia - acute 8mg IV/15mins - full effect 2-4 days.
233
Q

What are the indications for the use of bisphosphonates in the treatment of the following primary cancers: (4)

Prostate carcinoma Include in your answer the evidence supporting your recommendations.

A
  • Pts on long-term androgen suppression with decreased bone density on routine bone density scan (increased fracture risk). In this setting bisphosphonates (e.g. oral ridondrenate) or denosumab significantly decrease fracture risk.
  • Older trials (pre androgen deprivation) demonstrated a decreased rate of skeletal events with bisphosphonate therapy.
  • Hypercalcaemia as above.
  • Other potential indications – Intractable bone pain (despite multi-modal analgesia),
234
Q

What is considered very low risk PSA density?

A

PSA density<0.15ng/mL/gram

235
Q

Dose and technique for node + prostate cancer
Give the Benefit

A

Treating the nodes in this setting increases FFS to 85% in 3years compared to 55%. A 30%FFS benefit as per Stampede.

Curative intent EBRT with ADT (2-3 years) to maximum 60Gy/20#, 3Gy/#, 5#s/week. VMAT SIB technique, >=6MV photons, prescribed to D50. Aim PGTV D95>98% prescribed dose, avoid plan hotspots over OARS (especially bowel), Dmax<107%.
Pre sim: LUTSxt assessment and Mx as appropriate/if needed. Pre sim and each # - bowels empty (enema) and comfortably full bladder.
Sim: Supine, hands on chest, knee/ankle supports, immob w/vac bag.
22mm slice CT+ con(L3 - upper 1/3rd femur), consider fusion with dx MRI/PSMA PET if +ve nodes small/reduced with ADT.

Volumes:
CTV60p = Entire prostate + 1st 1cm of SVs
GTVn = + nodes
CTV60n= GTVn+3mm (depending on prox to bowel may reduce to 55/20).
CTV44= CTV60 + remaining SVs, external iliacs+obturator nodes, internal and common iliac nodes to bifurcation or 2cm above highest positive node (whichever is higher).!!!!!!!!!!!1
PTV= CTV44+7mm.
OARS (in EQD2): Rectum V40Gy<35% (ideal) may accept up to V50Gy<50%

Bladder V40Gy<30%, Small bowel V45<195cc, femoral heads V40Gy<40%. Large bowel - avoid hotspots

236
Q

The investigations confirm the primary tumour is confined to the prostate gland, with no extracapsular extension. There are 2 pelvic lymph node metastases (10 and 12mm respectively). There is no evidence of disease elsewhere.
b. What are the appropriate treatment options for this patient? Justify your answer. (3)

A

This fit 68yro patient has regional node positive (“Stage IV”) prostate cancer. Phase 3 RCT data supports ADT (Initially 24-36 months) with radiation to the prostate and nodes (STAMPEDE).
In this setting STAMPEDE data also demonstrates a PFS benefit to concurrent Abiraterone + ADT (but not Apalutimide in addition), though this is not routine.

Radiotherapy options all with 2-3 years of ADT (as tolerated preferred by Pt):
- Preferred: 60Gy/30# to the prostate, proximal SV and nodes (55/20 if bowel tolerance exceeded), and 44Gy SIB to the pelvis. As above , this approach has clear benefit in PFS compared with systemic Mx alone or observation.
- EBRT 45/20 (55 to +ve nodes) and brachy therapy boost to prostate (e.g. 18Gy/2#s)
Other options:
- No RT with ongoing medical Mx (e.g. ADT+/- Abiraterone +/- Docetaxel). Or observation (poor option given age, fit and aggressive disease).
- Radical prostatectomy and pelvic lymph node dissection - Though likely organ confined disease (<T3), node +ve status makes it very likely primary surgery will be sufficient. May well require adjuvant/salvage treatment.

237
Q

a. A fit 68 year old male presents with a PSA of 18ng/ml. Rectal examination demonstrates possible extracapsular invasion. 30% of biopsy cores are positive for adenocarcinoma, Gleason 4+4=8.
i. What risk group does this patient fall into? Justify your answer. (2)

A

This patient has very high risk prostate cancer (as per NCCN) – Assuming extracapsular extension is not into the rectum (T4a automatically Very High Risk) this is at least T3a, ISUP 4. Both T3a and ISUP4 are high risk features. 2 or more high risk features = Very High risk.

238
Q

Differentiate:
Urothelial carcinoma from
SCC and
prostate adenoCa (or Pr ductal adeno)

A

UC: GATA 3+, HMWCK+, CK7

SCC: CK 5/6, p63+, GATA3-

PrCa: PSMA, AMARC, PROSTEIN. The following are negative: CK 7, GATA, HMWCK, p63.

239
Q

A 60-year-old man presents with lower urinary tract symptoms (LUTS) and haematuria. On abdomino-pelvic ultrasound examination he is found to have a mass lesion arising from the trigone/bladder neck region of the urinary bladder. Urinary cytology is suspicious for high grade urothelial carcinoma.
a. What further investigations and procedures would you undertake in this patient? Justify your answer. (2)

A

Bloods: Hb (anaemia), Pre-treatmemnt baseline FBC U&E LFTs (renal function may indicate post renal failure).
UMCS prior to cystoscopy and Bx.

Cystoscopy + Bx - for definitive Dx and info on extent/location, mutifocal/Ta/Cis, ureteric involvement/risk of obstruction, urethral/bladder neck involvement. Histology to include report on extent muscle involvement - critical to deciding approach.

Imaging:
CT IVP to image entire urinary tract (Don’t forget this!!).
Pelvic MRI - for soft tissue delineation,
staging CT/PET for systemic staging IF muscle invasive.

240
Q

List the prognostic factors for urothelial bladder cancer:

A

Patient: Age, ECOG status, male, Ongoing smoking

Tumour:
Stage (most important factor)
Grade: high grade = worse
Lymph node status
Histological variants: (Carcinoma with rhabdoid features, micropapillary features, plasmacytoid carcinoma, sarcomatoid carcinoma, small cell carcinoma, and undifferentiated carcinoma = worse)
Lesion size: increase size = worse
Carcinoma in situ: presence = increase recurrence = worse Multi-centricity
Interval to recurrence: shorter interval = worse

Tx: Extent of TURBT

241
Q

Describe the two theories for the multifocal nature of urothelial carcinomas of the urinary tract.

A

1) Monoclonal hypothesis- Tumour as descendants of single transformed cell (monoclonal origin), for example do to local irritation (eg Sarcoma) that undergoes further genetic alteration, proliferates and spreads through the urothelial tract via intra-epithelial migration or intra-luminal seeding E.g. from smoking 


2) Field Change (cancerization): Urothelial cells undergo malignant transformation at multiple sites independently from local carcinogen exposure (polyclonal origin) and become the source of multifocal tumour. E.g. e.g. from cyclophosphamide exposure throughout bladder.

242
Q

Briefly describe the epidemiology, risk factors and histopathology and IHC of primary Squamous Cell Carcinoma (SCC) arising in the urinary bladder. (1.5)

A

Epidem: Rare - <5% of primary bladder cancers.
RFs: Chronic irritation/inflammation (e.g. long term IDC, chronic UTIs, caculi, neurogenic bladder), smoking, schistosomias infection (in endemic areas), previous bladder SCC, radiation.
Macro (not asked): Appear white due to keratin, bulky, polypoid, necrotic.
Histo: Must not show component of conventional urothelial carcinoma (if present, tumour should be classified as urothelial carcinoma with squamous differentiation)!!! Keratin and intracellular bridging.
IHC: CK 5/6, p53, GATA and HMWCK -ve

243
Q

Epi/Biobeh of ductal adenocarcinoma of the prostate and PSA

Histology:

A

3% of prostate cancer
Periuthral disease may cause haematuria.
More likely to have distant disease at Dx
Mets including to unusual sites (e.g. the dick).
Similar mortality rate to Gleason score 8 - 10

PSA can be highly variable, but on average lower than acinar.
More likely to have a PSA < 4.0 ng/mL;

Histo:
Frequently mixed with acinar adenocarcinoma
Tall columnar cells with pseudostratified nuclei.
Patterns include cribriform, papillary, solid and prostatic intraepithelial neoplasia-like pattern

244
Q

A 62 year old man presents with right upper quadrant pain and weight loss on the background of a resected sigmoid colon carcinoma 5 years ago. Abdominal ultrasound shows a liver lesion.
a. How would you further assess this patient? Justify your answer. (2)

A

Hx: Functional status and co-morbidities (e.g. cirrhosis) limiting treatment options. Signs/Symptoms of liver failure. Sx of choletstasis due to e.g. common bile duct obstruction,
Ex: Features of advanced disease (cachexia, bruising, hepatic flap, muscle wasting, jaundice if obstructive). General systems exam with focus on GIT and lymphadenopathy (e.g. virchows) to define disease extent.

Bloods: CEA (recurrent disease burden), LFTS (for impaired function), coags and protein (synthetic function), Baseline U&E/FBC for treatment planning.

Bx for tissue dx: Depending on other workup location - USS or CT guided, for tissue Dx, in some cases may be open/lap resection (I.e. diagnostic and therapeutic).

Imaging: Complete staging CT CAP/PET for systemic disease. Liver MRI for soft tissue delineation.

245
Q

Investigations confirm a solitary 3cm liver metastasis with no other systemic disease. What are the management options available to this patient? Justify your answer. (3)

A

-My preference: SABR 54Gy/3# (> approx 3cm do 50/5). Non-invasive, performed as outpatient, ablative, multiple retrospective and 2 phase II prospective trials demonstrate comparable control to below options. Preferred over RFA due to size of 3cm (>3cm RFA has progressively less local control). Benefit over thermal ablation where lesion near large vessel (which act as heat sync). There is no direct comparison of RFA to SABR.
-Thermal ablation (Radio-frequency, microwave or irreversible electroporation). Offers good local control, and often most familiar technique.
-TACE/TARE (Trans-arterial chemo/radio embolisation): May be better suited too larger (3-5cm lesions), can have significant toxicity, more invasive.
-Surgical reaction (e.g. local excision or hemi-hepatectomy): May be best choice for superficial lesion especially where Dx is in doubt ant tissue needed. Most invasive, resource intensive option.
-Systemic therapies alone (or in combination with above): chemotherapy/immuno/molecular therapies.
-Best supportive care.

246
Q

Paracentesis is commonly used to drain symptomatic ascites in patients with metastatic liver disease.
i. Describe the technique for this procedure. (2)

A

Consent. Analgesia prior (e.g paracetamol, and short acting option such as several 10mg).
Percus abdomen - identify and mark site. Right or left lower quadrant. Approx 5cm superior and 5cm medial to ASIS. Percus for dullness to confirm, USS is considered best practice. Inject topical anaesthetic (e.g. lidocaine 1%).
Patient supine bed elevated 45degrees, and comfortable.
Gown and glove. Clean and prep (e.g. chlorhexidine), then small procedure drive with central fenestration.

Insert 22G needle at site (perpendicular to skin), feel for pop then fluid release through needle. Advance metal catheter over needle. Remove needle. Insert plastic catheter, then attach drain.

Free drainage, up to 6L, in excess of 5L drained consider IV albumin replacement. Monitor patient for comfort and fluid status.

247
Q

What are the management options available for Arterio-Venous Malformations of the brain? Include in your answer the advantages and disadvantages of each option. (4)

A

Surgery: Advtg: Most immediately definitive option where possible, can improve AVM related seizures. Offered excellent LC >90%. Disadv: Limited option as cannot be performed on large (>2cm), deep drainage, or eloquent brain area (reflected in Spetzler Martin grading). However, can be combined with embolisation (see below) to increase suitability for surgery. Invasive, with intraopertive risks including death. Some patients are not operative candidates.
SRS: Advtg: Can treat larger lesion (up to 3cm), with deeper drainage (higher S-M grade) than surg. Offers excellent LC - 90%. Conformal and can treat lesions within more eloquent areas. Disadv: 3yr delay until effect, during which bleeding risk continues unchanged. Does not reduce AVM related seizures. Late effects including radionecrosis, and 2nd maligancy. Cannot treat >3cm (nor any option alone). SRS frame invasive.
Embolisation: Adv: May allow size reduction so above options can be given. Disadv: Not a definitive option, invasive with risks of vascular injury and emboli.
Symptomatic Mx, and risk reduction: May be only available option for high S-M grade lesions.

248
Q

In the treatment of thyroid eye disease with radiation therapy; what are the: (3)
i. indications
ii. contraindications
iii. target volumes
iv. radiation dose fractionation schedules and v. expected response rates to therapy?

A

i. indications
- Clinical exophthalmus due to thyroid eye disease
- Failed trial of steroids
- Not suited to or declined other approaches (surgical decompression)
ii. contraindications
- Responsive to steroids/other Mx.
- Connective tissue diseases (cautionary)
- Eye diseases: e.g. glaucoma
iii. target volumes
CTV= preorbital tissues, PTV 3mm. E.g. with 5 × 5 cm lateral fields using 6 MV photons and 5° poste- rior tilt or half-beam block
iv. radiation dose fractionation schedules and v. expected response rates to therapy?
20Gy/10#, recent evidence suggests that this dose may be reduced (Don’t ever hypofractionate benign). Local control in approximately 2/3rds.

249
Q

An 80 year old patient has a basal cell carcinoma (BCC) excised from the midline left lower eyelid. The lesion is 5mm in size with a focally positive deep margin and clear peripheral margins of 3mm. There is no perineural or lymphovascular invasion.
a.
i. How would you further assess this patient?

A

Hx: Co-morbidities and functional status limiting treatment options or overall life expectancy. Social history (e.g. difficulty attending planned fractions of treatment). Immunesupression. Patient wishes.

Ex: Wound healing/infection/function (eyelid able to close), Cranial nerve (especially CN VII and V2) for PNI. Dermoscopy over margins, nearby lesions.

Histology: Further information from report - BCC subtype (e.g. nodular slower growing, less aggressive than morphoiec). Depth of invasion, invasion of tarsal plate.

250
Q

An 80 year old patient has a basal cell carcinoma (BCC) excised from the midline left lower eyelid. The lesion is 5mm in size with a focally positive deep margin and clear peripheral margins of 3mm. There is no perineural or lymphovascular invasion.
a.
ii. Discuss the management options for this patient.

A

The patient’s risk of recurrence in the setting of +ve margins and orbital location is in the order of 30%. There is a potential for a recurrence in the deep margin to go undetected until disease more advanced - risking larger volume for surgery (greater defect/eyelid function loss need for reconstruction).
Re-excision - may not be technically feasible due to deep margin, with significant cosmetic and or function defect. May involve reconstruction. Patient may prefer other options.
Adjuvant radiotherapy - Preference would be SXR, with eye shield 45Gy/15#.
Observation - May be entirely appropriate depending on above assessment and discussion of risk.
Topical: Proximity to eye, and deep margin makes superficial therapies (imiquimod or cryotherapy) unsuited.
Systemic: vismodigib in this setting while possible is outside the scope of normal clinical practice.

251
Q

Clinically the patient has a 12mm vertical scar in the midline lower eyelid. There is no obvious residual BCC. A decision is made to offer adjuvant radiation therapy.
b. Describe an appropriate radiation therapy technique and dose fractionation schedule. (3)

A

Adjuvant RT to reduce risk of recurrence. SXR technique, 40Gy/15#, 5#s/week, 100% prescribed dose at skin surface, energy/filter/SSD selected for D90 cover at target depth (e.g. 3mm).

Pre: Explain vision changes associated with topical anaesthetic (may need to arrange transport etc).
Sim: Mark-up scar, make sechoose lead cutout and applicator appropriate to markup + 10mm margin (for CTV + setup error and penumbra). Make setup template from above + setup photos. Eyeshield fitted. Using departmental tables, select Energy/Filter/SSD e.g 20cmSSD 2mmAl, 100Kv for D90 ~3mm (consider contribution of scatter from eye shield to dose). Clinical setup photos.
Volume: as above, Dmax at skin surface aim D90 based on path report.
OARS (in EQD2, note SXR higher BED): Eye: lens DMax<8Gy, Eye <45Gy, lacrimal duct Dmax<30Gy.
Verification via template match and setup photographs.

252
Q

Clinically the patient has a 12mm vertical scar in the midline lower eyelid. There is no obvious residual BCC. A decision is made to offer adjuvant radiation therapy.
c. What are the potential complications of adjuvant radiation therapy in this patient? (2)

A

Acute: Conjunctivitis and corneal abrasions from eyeshield, radiation dermatitis, wound breakdown. Sinusitis

Late: Fibrosis of eyelid (function impairment - I.e impaired closing leading to ectropion-like Sx), dryers and impaired/blocked lacrimal ducts. Chronic sinusitis. Chronic skin changes (depigmentations, telangetasia).

253
Q

Following surgery for cutaneous Squamous Cell Carcinoma (SCC) in the Head and Neck area, what high-risk factors would you consider when recommending adjuvant radiation therapy. (2)

A

Absolute indications =
T4,
+ve margin where further resection not available.
PNI in large nerve.
Recurrence where no previous RT given.

Relative (if 2 or more then do):
T21, 2 or T31.
≥6 mm depth of invasion.3, 4
Close margins < 2 mm.
Poorly differentiated.
Pathological microscopic perineural invasion (PNI).1
Lymphovascular invasion.
Immunosuppression.
Higher risk sites, e.g. non-hair bearing lip, chin, ear, preauricular, periorbital, temple.

254
Q

A 70 year old woman presents with several lytic lesions on her spine. You suspect this is myeloma. (2.5)
i. What investigations would you request?

A

Remember BLAC CRAB R-ISSole (yum)

Urine: Urine electrophoresis for M-Protein, also microscopy (rouleaux, protein urea)

Bloods: Calcium, renal fuction, electrolytes, Hb for anaemia and FBC. LDH, Beta2 microglobulin, albumin (for R-ISS). Plasma electrophoresis (I.e >3ng/L M-protein)

Tissue: BMAT for bone marrow involvement >10%, Biopsy of lesion.
Cytogenetics - high risk translocations t(4,14), t(14,16).
**Flow Cytology: show kappa/lambda restriction

Imaging: XR Skeletal survey

255
Q

Describe the pathogenesis of lytic metastases in multiple myeloma. (2)

A

Lytic mets due to increased osteoclast activity (de-regulated bone remodelling in favour of resorption) promoted by monoclonal plasma cell secretion of a number of factors including tumour growth factors (TGFs), cytokines, some factors may promote RANK ligand expression on osteoclasts increasing bone resportption activity.

256
Q

c. With regards to solitary extramedullary plasmacytoma, describe the: (3)
i. epidemiology

ii. most common location in which it occurs

iii. immunostaining pattern

iv. prognosis after curative radiotherapy and how this differs from solitary plasmacytoma of bone.

A

i. epidemiology
Rare malignancy of late adulthood mean age 55 M>F. 2/3 medullary, 1/3 extra medullary.

ii. most common location in which it occurs
H&N neck locations most common site (e.g. nasal cavity, nano or oropharynx), other sites include GI bladder, CN GU.

iii. immunostaining pattern
CD 38, CD 138 and CD 70 +ve. Almost never b cell markers CD19 or CD20.

iv. prognosis
Extramedullary have better prognosis in terms of reduced chances of transformation to MM. Only 35% in 10years transform (versus 90% of medullary type).

257
Q

For monoclonal gammopathy of unknown significance: (2.5)
i. provide the definition

ii. describe how it differs to smouldering myeloma

iii. list the risk factors for progression to multiple myeloma.

A

Most common plasma disorder (5% of ppl age>70) Increased M-protein protein spike on PEP/UPEP (but <3ng/L). NO bone marrow involvement or CRAB.

ii. How it differs to smouldering myeloma
Lacks significant bone marrow involvement (I.e. <10%). M-protein spike <3g/L.

iii. Risk factors for progression to multiple myeloma.
MGUS - 1% transformation per year. Average 10.5 years to transformation.
Patient: Age (>70), immunosuppression, immunestimulation/chronic inflammation, African race, obesity. diabetes.
Tumour: SPEP/UPEP M-protein levels >1.5 ng/L, IgA or IgG proteins.

258
Q

A 3 year old girl presents with left proptosis. She has a biopsy confirmed embryonal rhabdomyosarcoma arising from ethmoid sinus, with para-meningeal and orbital extension. Staging investigations show no evidence of metastatic spread (Stage 2, Clinical Group III disease).
a. Discuss the principles of management in regard to this patient (3m)

A

Acute symptom - Analgesia for pain management management - Dexamethasone to reduce swelling/ proptosis

Ix: - Baseline ophthalmologic assessment
- MRI base of skull to better evaluation of local disease
- FDG PET scan to evaluate distant metastases
- Bone marrow aspirate and trephine (to exclude marrow involvement). ALWAYS FOR RHABDO!!!!
- Lumbar puncture to exclude CSF involvement

On Mx:
- Given the location of the tumour, it would be deemed unresectable
- Her cancer care will likely involve chemotherapy (VAC – vincristine, actinomycin-D,
cyclophosphamide), followed by definitive radiotherapy, commencing ~ week 13 of chemotherapy

259
Q

A 3 year old girl presents with left proptosis. She has a biopsy confirmed embryonal rhabdomyosarcoma arising from ethmoid sinus, with para-meningeal and orbital extension. Staging investigations show no evidence of metastatic spread (Stage 2, Clinical Group III disease).

b. Describe in detail a suitable radiation technique and dose prescription for treating this girl (3m)

A

50.4Gy in 28 fractions, 1.8Gy per fractions, 5 fraction per week, to the ethmoid sinus (+ para-meningeal + orbital extension) with concurrent VAC (vincristine + cyclophosphamide; omit actinomycin-D)
Pre-SIM
- Repeat post- chemotherapy MRI
- Consider GA if patient is not able to remain still for RT SIM
- Position: supine, arms down, head towards gantry
- Immobilisation: thermoplastic mask, vac bag
- Planning CT: 2mm slice CT from vertex to lower cervical spine Fusion: pre-chemotherapy imaging
Target volume
- GTV(post-chemo) = post-chemo residual disease
- GTV(pre-chemo) = pre-chemo gross disease (including extension into para-meningeal and orbit)
- CTV50.4 = GTV(post-chemo) + 5mm
- CTV41.4 = GTV(pre-chemo) + 1cm
- PTV = CTV + 5mm expansion
RT technique – VMAT technique with 6MV photon Plan evaluation
- Ensure PTV coverage, PTV D98>95%
- Minimise hotspot, PTV D2<107%, ensure no hotspot outside of PTV
- Review OAR DVH
o Optic nerve/ chiasm/ brain stem: Dmax<TD o Pituitary gland/ temporal lobe: ALARA
o Lacrimal gland <41.4Gy
o Lens: Dmax <10Gy
Treatment verification
- Daily kV imaging, matched to bone, with 3mm tolerance

260
Q

A 4-year-old girl presents with proptosis. A right orbital mass is identified on MRI. Biopsy confirms an embryonal rhabdomyosarcoma. Further staging investigations show no metastatic disease
a. What is your overall management plan for this patient? (3m)

A

This girl has right orbital rhabdomyosarcoma, embryonal (=favourable histology), orbit (=favourable site), Stage 1, Clinical group 3 (biopsy); low risk category
Acute: anagesisa and dex for swelling
Ix: Bone marrow for involvement. PET Baseline opthal

Onc Mx:
MDM - Generally as inoperable - VAC with RT to commence week 13 concurrent with chemo.

261
Q

Epidemiology of rhabdomyosarcoma:

Associated syndromes:

IHC:

Broadly, what is the benefit of RT in rhabdomyosarcoma

A

Bimodal peak age 4, and 18 (average 4)
slight M>F
Most common site= H&N
Syndromes: Li-Fraumeni, Gorlins, BWS, NF-1, hereditary Rb
IHC= MAD - Myogenin, Actin, desmin. This is a small round blue cell tumour w/3 types (embryonal = good, alveolar = bad, undiff = worse).

We know from the European (SIOP as opposed to COG) pathway that omitting RT is associated w/50% recurrence.

262
Q

A 4-year-old girl presents with proptosis. A right orbital mass is identified on MRI. Biopsy confirms an embryonal rhabdomyosarcoma. Further staging investigations show no metastatic disease

Give dose and technique.

A

Definitive EBRT 50.4Gy/28#, 1.8Gy/#, 5#s/week to the right orbital mass with concurrent chemotherapy (vincristine and cyclophosphamide; and omitting actinomycin-D during RT)
Pre-SIM:
* Repeat MRI post chemotherapy
* Consider GA if patient not able to remain still on RT SIM
* Position: supine, arms down, head towards gantry, under GA
* Immobilisation: thermoplastic mask, vac bag, neck support,
* Marker: no specific
* Planning CT: 2mm slice CT from vertex to lower cervical spine Fusion: pre-chemo and post-chemo MRI
Targe volume:
* GTV = pre-chemo gross disease
* CTV = GTV + 1cm clipped at anatomical (i.e. bone) boundaries (no need to cover entire orbit)
* PTV=CTV+5mm
Technique: partial arc unilateral VMAT technique with 6MV photon Plan evaluation:
* Tumour coverage PTV D98>95%
* Minimise hotspot PTV D2<107%, ensure no hotspot outside of PTV/ over critical OAR
* Ensure low dose (10% and 50%) reasonably distributed around PTV, with contralateral sparing
* Review OAR DVH
o Pituitary and temporal lobe ALARA
o Lacrimal gland Dmax< 41.4Gy
o Optic nerve/ chiasm/ brain stem: Dmax< 50.4Gy o Lens Dmax< 14Gy
Verification: Daily kV, matched to bone, with 3mm tolerance

263
Q

A wide local excision, sentinel node biopsy and axillary dissection are done for a cutaneous melanoma. a. Discuss the prognostic factors that you would look for in:
i. The pathology report (3m)

A

Primary:
- Size,
- Breslow thickness,
- Margin
- Ulceration
- Mitosis
- Satellite
- Clarke level
- LVSI
- TIL
- Regression
- Desmoplastic component
- Neurotropism

The Nodes:
Think Burmeister criteria: ECE, number, and size.

264
Q

Describe the incidence, epidemiology and histological features of the following cutaneous malignant melanoma (4m)
i. Lentigo maligna melanoma

A

10% of melanoma
Chronic sun exposure, older patients, a/w freckles, usually head and neck sites
%5 transform into invasive disease.

Histo: Non-invasive lentiginous growth (continuous single cells at base of epidermis), often in atrophic epidermis, ill-defined borders.

265
Q

Describe the incidence, epidemiology and histological features of the following cutaneous malignant melanoma (4m)

ii. Acral lentiginous melanoma

A

Rare < 4%
No associated with UV exposure
More common in Asian Hispanic/ African (darker skin)
Commonly in palm, sole, subungal, muco- cutaneous oral and nasal cavity, anus

Histo: Can have pattern of lentiginous/ nodular/ superficial spreading pattern

266
Q

Describe the incidence, epidemiology and histological features of the following cutaneous malignant melanoma (4m)

iii. Nodular melanoma

A

20% of melanomas (Superficial spreading are 75%)
Can arise in UV exposed/ non-UV exposed area

Histo: Proceed to vertical growth phase directly (invasive without radial growth phase,
i.e. no intraepidermal component peripheral to the invasive component)

Common site: widespread distribution

267
Q

Describe the incidence, epidemiology and histological features of the following cutaneous malignant melanoma (4m)

iv. Desmoplastic melanoma - also Biobeh

A

Slightly older age group, usually amelanotic, more commonly in H&N

Micro: poorly circumscribed, pleomorphic spindle cells with prominent desmoplastic stroma. PNI (“neurotropism”) common.

Up to 50% local recurrence, with PNI common - some low level data suggests RT reduces risk to 10%.

268
Q

Two years later the man develops in-transit subcutaneous metastatic melanoma nodules affecting the left leg. There is no systemic metastatic disease.
b. What are the options for his management? (2m)

A

What are the options for his management? (2m)
- Local treatment:
o Surgical resection of local disease
o Limb perfusion with Melphalan
o Radiotherapy if not surgical candidate (e.g. 30/5)

  • Adjuvant immunotherapy after local treatment. BRAF+ve then Debraf+Trametinib (DeTram)
269
Q

Immunotherapies (name targets and mechanism) for BRAF negative melanoma:

What is the benefit?

A

If no BRAF than target T-Cell mediated cell death evasion:

CTLA4 - Ipilumimab. Normal role is to downregulate T-Cell activation. Upregulated in cancer leading to immuncheckpoint evasion. Ipilumimab binds CTLA4 and inactivates it.
By itself adds 3 months PFS.

PD-1 - E.g. Nivolumab. Binds PD1 receptor on T-Cell preventing PDL-1 from binding and inactivating T-Cell,
By itself adds 6Months

Ipi + Nivo = 12months PFS

OS for metastatic BRAF negative is 45%

270
Q

Immunotherapies (name targets and mechanism) for BRAF positive melanoma:

What is the benefit?

A

Best results combine BRAF and Mek targeted therapies:
TKI signal transduction pathway RAS RAF MEK ERK

Braf: Dabrafinib

MEK is molecular inhibition Trametinib.

Get on the Dab Tram for 1yr OS 70%

271
Q

b. Describe the microscopic features of a Gleason Grade 3 prostate adenocarcinoma (2m)

c. List the immunohistochemical stains that may help differentiate between benign and malignant tissues in the prostate biopsy (1m)

A

Grade 3:
- Small glands, discrete/ separate (i.e. not fused), no cribriform glands, no comedonecrosis

IHC benign v.s Malig:
1) Malignant loose basal cells and become HMWCK -ve

2)p63 Expressed in basal cell nuclei
- malignant = loss of basal cell → p63 IHC negative

3) AMACR - +ve in prostate cancer. Expressed on Apical portion

272
Q

Briefly describe the clinical impact and pathophysiological mechanisms of sexual dysfunction of the doodle that may follow radiation therapy

A

1) Erectile dysfunction (nerve/vasc/fibrosis)
- Neuronal damage: RT-induced damage to cavernous nerve, and reduction in signalling molecule for vasodilation
- Vascular damage: RT-induced fibrotic changes to vessels resulting in reduced blood flow into penis
- RT-induced fibrosis of smooth muscle of penile bulb and atrophy of corpus cavernosum over time due to infrequent erection

2) Reduced ejaculate - the gland has been cooked.

3) Retrograde ejaculation
Sphincteric dysfunction following RT

273
Q

Metastatic cascade

A
  1. Local invasion
    - Adhesion to and invasion of basement membrane
    - Passage through the extracellular matrix
  2. Intravasation
    - Intravasation into vasculature/ lymphatic
  3. Circulation/ dissemination
    - Evasion of immune system in the circulation (either by homotypic adhesion/ heterotypic adhesion with platelet forming tumour-platelet aggregate)
  4. Extravasation
    - Non-random homing of neoplastic cells to specific metastatic sites (e.g. bone)
    - Adhesion to endothelium and passthrough through basement membrane at distant site
  5. Metastatic deposit
    - Remain dormant as micro-metastasis or
    - Start angiogenesis and proliferation at metastatic site
274
Q

A 72 year old male presents with widespread metastases from a primary cancer originating in the prostate gland.

a. Describe the molecular and cellular events involved in:
i. The development of the primary tumour (2m)

A

Cellular (molecular in brackets):
Normal prostate → proliferative inflammatory atrophy (germline mutations)→ (chromosome 8p loss 8q gain) prostatic intra-epithelial neoplasia (PIN) → Loss of basal cells (gene fusions) → invasive prostate cancer with progressive loss of glandular architecture (PTEN reduction and p53 loss) → Metastatic cascade.

Molecular:
Germline mutations -> Chromosome 8 - 8p loss, 8qgain -> gene fusions -> p53 loss and decreased PTEN.

275
Q

Describe the Gleason Scoring system for prostate cancer. Include mention of any differences between
how the pathologists assigns the Gleason score on prostate biopsy specimens and a radical prostatectomy specimen (2m)

A

For both Bx and RRP specimens a primary (most prevalent) and secondary score (most aggressive or 2nd most common - see below) are reported. For RRP specimens a tertiary score is also reported - namely the 3rd most prevelent score. constituting <5% (if present).
Grading is based on the extent and quality of glandular fromation and the presence of necrosis as follows:

Gleason 1-2 - Non maligant glandular tissue
Gleason 3: Small glands, with no comedo necrosis or solid component.
4: Slit like glandular lumens, fused glands, cribiform architecture
5: Solid architecture, comedo necrosis

276
Q

ISUP grades

A
  • ISUP GG1 = Gleason 3+3
  • ISUP GG2 = Gleason 3+4
  • ISUP GG3 = Gleason 4+3
  • ISUP GG4 = Gleason 8 (4+4, 3+5, 5+3) - anything that adds to 8
  • ISUP GG5 = Gleason 9 (4+5/ 5+4/5+5) - anything>=9
277
Q

What is the optimal duration of hormone therapy for HR prostate cancer?

A

OS benefit has been demonstrated with long-term ADT (28–36 months) compared to short-term regimens (4–6 months), even in the dose-escalated era (DART trial).

EORTC trial 3yrs ADT improved 10yr OS from 40%-58%

One trial (PCS IV) found similar oncologic outcomes between 18 months and 36 months.

The RADAR trial found that 18 months of ADT was superior to 6 months prCa spec OS 13vs10%

278
Q

List the adverse prognostic factors for Neuroblastoma (2m)

A

Prognostic factors = Factors in COGS staging (‘SAND-S’)
- Stage: higher stage = worse
- Age (>18mo = worse)
- N-MYC protein amplified = worse
- DNA ploidy (diploid = worse; hypo/ hyperploid = better)
- Shimada classification (SAD-MiNd)

SHIMIDA:
o Stroma pattern (rich = better; poor = worse)
o Age
o Differentiation (differentiated = better; undifferentiated = worse)
o Mitotic=karyorrhexis index (low = better; high = worse)
o Nodularity (diffuse = better; nodular = worse)
- Cytogenetic abnormalities
o Favourable – hyperploidy DNA, TrK-A amplification
o Unfavourable – N-Myc protein amplification, diploid DNA, deletion of 1p chromosome, del 11q, TERT
rearrangement, ALK amplification

279
Q

In general, what is the role of radiation therapy in the management of neuroblastoma? (2m)

A

1) Consolidative - For high risk as part of multimodal therapy (after induction chemo/ surgery/ stem cell RT transplant) to the surgical bed + gross residual disease to improve local control

2) Emergency palliative RT
a) for Pepper syndrome (respiratory distress from liver mets causing respiratory compression) liver RT 4.5Gy/3#
b) Emergency palliative RT to paraspinal disease causing cord compression

280
Q

For Neuroblastoma: List the factors used by the Children’s Oncology Group or International Neuroblastoma Group to stratify patients into various risk groups (2m)

A

‘SANDS’
- INSS Surgical Stage
- Age
- N-MYC amplification
- DNA ploidy
- Shimada histology (SADMiNd) (Favourable histology vs. Unfavourable histology)
o Stroma (rich = FH; poor = UH)
o Differentiation (differentiated = FH; undifferentiated = UH) o Mitotic-karyorrhexis index (low = FH; high = UH)
o Nodularity (diffuse = FH; nodular = UH)

281
Q

A 3 year-old girl presents with a biopsy confirmed primary neuroblastoma of the right adrenal gland. After the completion of all relevant staging investigations, she is found to have a poorly differentiated, MYCN amplified, INSS Stage IV disease
a. Briefly outline the overall management for this patient (2m)

A

This patient has high risk neuroblastoma. This patients should be treated with multimodality therapy
- induction chemotherapy → surgery → conditioning chemotherapy + haematopoietic stem cell transplant
(HSCT) → radiotherapy to the primary site/ residual disease → maintenance differentiation therapy (retinoic acid) and immunotherapy

282
Q

A 3 year-old girl presents with a biopsy confirmed primary neuroblastoma of the right adrenal gland. After the completion of all relevant staging investigations, she is found to have a poorly differentiated, MYCN amplified, INSS Stage IV disease

Expected OS with this treatment?

A

Consolidative radiotherapy to a total dose of 36Gy/20: pre-op GTV 21.6Gy/12# (Phase 1), with additional boost (Phase 2) TO surgical bed to total overall treatment dose of 36Gy/#, 1.8Gy/#, 5/#s/wk to reduce LRR
Pre-SIM
- Consider GA
Position: supine, arms up, head towards gantry- Immobilisation: vac bag
- Planning CT: 2mm slice CT from thoracic inlet to femoral head (overing entire lung, kidney for OAR DVH) Fusion: pre-op and post-op MRI
Target volume
- GTV(pre-op) = post-chemo and pre-op right adrenal tumour
- GTV(boost) = post-op gross residual disease
- CTV21.6 = GTV(pre-op) + 1.5cm, clipped at anatomical boundaries
- CTV14.4 = GTV(boost) + 1.5cm, clipped at anatomical boundaries
- PTV=CTV+5mm

5yr OS 5-10%!!!

283
Q

Neuroblastoma:
Following completion of all treatment, this patient has a bone marrow relapse. The decision is made to treat her with palliative intent using I131-metaiobenzulguanidine (I131-MIBG). With a knowledge of the physical and biological properties of this radio-pharmaceutical: (2m)
i. What is the rationale for its use in this setting?
ii. What are the principal side effects?

A

Rationale
- Neuroblastoma cells take up nor-epinephrine due to expression of nor-epinephrine transporter on tumour cell membrane
- MIBG is taken up in the same way as nor-epinephrine
- I131 attached to MIBG – deliver -ray (short range) of ablative dose, allow sparing of OAR

SEs:
Marrow supression
- GI – nausea/ vomiting, diarrhea, deranged liver function (esp. ALT)
- Hypothyroidism (10-20%)

284
Q

Define the following:
Absolute Risk Reduction (ARR)
Number needed to treat (NNT)
Relative Risk (RR)
Relative Risk Reduction (RRR)

A

ARR = I Experimental event rate - Control event rate I
NNT = 1/ARR
RR = Experimental Event rate/Control Event Rate
RRR = ARR/Control event rate = 1-RR

285
Q

He has a 2cm scar 3cm below the lateral infraorbital rim with no signs of local recurrence. There is reduced sensation in the infraorbital region only. There is no lymphadenopathy.
Histology shows a 1cm x1cm x 6.5mm thick poorly differentiated SCC. Peripheral margin are well clear, deep margin is 0.3mm. There is extensive perineural spread involving nerves up to 0.2mm in diameter. There is no other sign of disease
b. A decision is made to proceed with adjuvant radiation therapy. Describe a suitable radiation therapy technique and dose fractionation schedule. Your answer should include a detailed description and justification for your choice of your target volume (4m)

A

Multiple HR factors: extensive PNI in nerves up to .22 mm, >6mm deep, +ve deep margin (0.3mm)

PORT for LC: EBRT to total of 63Gy/30# 5#/wk. VMAT SIB technique, >=6MV photons, prescribed to D50.

Pre: dental assess.
Sim: the usual. Bolus. 1mm slice CT with contrast fused with skull bass MRI for nerves.

CTV63=Tumour bed, scar, because of PNI then 2cm expansion.
CTV 60 = CTV63+ a strip (from medial edge of field to infra orbital foramen to encompass HR PNI).
CTV54= CTV60+ infra-orbital nerve to foramen, entire paritid for next escelanon node and to cover potential CNVII invasion.
PTV=CTV54+5mm.

286
Q

After PORT for a skin lesion the patient develops osteoradionecrosis of the mandible
i. What factors increase the risk of this complication (2m)

A

Patient - Continued ETOH and tobacco use - Poor oral hygiene. Advanced age.

Tumour - Location of the primary tumour (tongue, FOM, tonsil, retromolar trigone) - Advanced stage of tumour - invasion. Vascular compromise.

Treatment - Dose approx 5% risk >70Gy. Volume. Plan hot spots (e.g a parotid tumour decreases w/size during treatment, therby decreasing attenuation leading to a greater dose to mandible).

287
Q

How would you management osteoradionecrosis (1m)

A

PENTOCLO. Pentoxifylline, tocopherol (vitamin E) and clodronate (PENTOCLO) - Multiple trials (and systematic review) support potential for improvement and local control.
- Good oral hygiene
- Analgesia/ anti-inflammatory for pain
- Antibiotics if infection

Referral to maxillofacial surgeon – consider removal of devitalised tissues (sequestrectomy)
- Consider hyperbaric oxygen in combination with surgery/ dental extraction

288
Q

Describe each of the skin SCC precursor lesions. The give macro and micro. Same for SCC

A

Actinic keratosis (solar keratosis)
o pre-malignant dysplastic lesion, a/w build-up of excess keratin o 1% progress to SCC, but 80% of SCC arise from them
o Macro: brown rough sand-paper from excess keratin, ‘cutaneous horn’
o Micro: cytologic atypia of basal cells with hyperplasia, intracellular bridges, thickened stratum corneum,
due to excess keratin production

Bowen’s disease (SCC in situ) o 3% progress to SCC
o Macro: sharply defined red macule/ papule/ plaque
o Micro: atypical cells in all layers of epidermis but does not invade basement membrane (cf: actinic
keratosis where dysplasia limited to basal layer of epidermis)

SCC
o Macro: nodular, ulcerative, varying amount of hyperkeratosis
o Micro: atypical cells (variable cellular differentiation, descriptors of anaplasia), intracellular bridges,
infiltrates dermis (WD – keratin pearls; PD – focal necrosis, no keratin)
IHC: CK5/6+ve, AE1/3+ve, p63+ve, EMA +ve (cf: EMA -ve in BCC)

DDx Keratokanthoma.

289
Q

Types of BCC:

general micro/macro:

A

Most common skin Ca
Nodular most common (60%),
Multifocal Superficial BCC (30%)
Morphoiec BCC (5%) - high risk of recurrence. Higher risk PNI
Infiltrative BCC (2%) - Indication for Moh’s, high recurrence and PNI risk

Basosquarmous - very bad - Have port and some chemo.

Macro: papule with pearly translucent appearance, overlying telangiectasia, central ulceration (rodent ulcer)

Micro:
Monotonous basoloid cells (large nuclei, scant cytoplasm), no intercellular bridge
Architecture = Palisading peripheral cells, peripheral clefting, fibroblastic stroma

IHC: BCL2+ve, p63+ve, EMA-ve (cf EMA+ve in SCC), CK20-ve

290
Q

Key Guideline for deciding whether appropriate for partial breast? What are the indications?

A

Tumour bed must be clearly demarcated AND NO NeoAdj Chemo.

Astro - identifies, appropriate, cautionary, inappropriate.

Appropriate:
Age > 50 (unsuited <40)
Size <2cm (unsuited >3cm)
Unifocal (unsuited multifocal)
ER pos (caution neg)
Margins <2mm
Node neg (cution pNmi)
No LVSI (caution focal LVSI)

IF EIC>3cm do not do!

291
Q

When delivering post mastectomy radiation to the chest wall:
i. What is the rationale for the use of bolus? (1)

Ii. Which patients require bolus? (2)

A

i. What is the rationale for the use of bolus? (1)
- to reduce dose to underlying lung
- Increase skin and subcutaneous dose (most common site of recurrence post mastectomy) and smooth some post mastectomy tissue irregularities.
Ii. Which patients require bolus? (2)
- Pts with close/involved superficial/skin margin
- Pts with highly irregular surface contour
- Pts with shallow skin to lung interface depth,

292
Q
A
293
Q

Macro and Micro description of PTC (papillary thyroid cancer):

A

Solid, grey white tumour, firm, invasive with ill-defined margins (<10% surrounded by complete capsule).

Dx based heavily on distinct nuclear features:
Changes in nuclear size and shape: large ovoid nuclei, nuclear elongation
Irregularities of nuclear membranes: abundant
nuclear grooves (from infolding of nuclear membrane), highly irregular nuclear contour
*Chromatin pattern: empty appearance of nucleoplasm, ground-glass nuclei (Orphan-Annie nuclei. i.e. empty looking)

o Papillary Architecture

o Presence of psammoma bodies (rounded, concentrically laminated calcification, from necrosis) in 50% of cases

294
Q

Broad aetiology of thyroid malignancies:

Relate genetics to specific subtypes

A
  • Sporadic
  • Ionising radiation (H&N RT, atomic bomb survivors)
  • Low iodine (FTC and ATC only)
  • Family history of thyroid cancer
  • Genetic syndrome
    o PTC/ FTC: Cowden (PTEN gene), Gardners (APC gene)

o MTC a/w MEN 2A and MEN 2B (RET proto-oncogene mutation)

295
Q

For medullary carcinoma of the thyroid gland, briefly describe the: (3m)
i. Epidemiology
ii. Cell of origin, and
iii. Microscopic and immunohistochemical features

A

Epidemiology
5% of thyroid cancer
70% sporadic, 30% familial (a/w germline mutation in RET proto-oncogene)

Cell of origin
Para-follicular C-cell, which secretes calcitonin

Microscopic
Round, oval, spindled to plasmacytoid cells
Amyloid accumulation in 70% cases
IHC Positive: Calcitonin, CEA
Negative: Thyroglobulin

296
Q

A 60 year old man presents with moderately firm mass in his right anterior neck, suspected to be malignancy in the
thyroid gland.
a. Briefly describe the investigation you would recommend. Provide a justification for each (2m)

A

Thyroid function (TSH, T3/T4)
- PTH and calcium = to rule out hyperparathyroidism
- Calcitonin and CEA, urine/serum catecholamine ( MTC)
- RET proto-oncogene germline mutation (in MTC)
- LDH/ ESR – lymphoma
- Beta-HCG/ AFP – germ cell

Imaging
- Thyroid ultrasound
- CT neck
- Radioactive iodine scan
o if hot = functional = likely non-malignant = no FNA
o if cold = need FNA? Malignant)
- FDG PET scan

Tissue
- U/S guided FNA for tissue diagnosis
- Mutation testing
- BRAF/ TERT mutation testing (prognostic for papillary ca)
- IHC as needed: TTF-1, PAX 8, thyroglobulin, calcitonin

297
Q

Describe the biological behaviour of the
following
i. Papillary carcinoma
ii. Medullary carcinoma
iii. Undifferentiated (anaplastic) carcinoma

A

Pap:
Generally good prognosis
5-20% local recurrence
30% nodal involvement
5% distant mets

Med:
75% nodal mets
10% distant mets
5-yr OS 60-90%

Ana:
Aggressive
Median OS 3 months
90% have nodal/ distant
mets at diagnosis

298
Q

Papillary thyroid carcinomas commonly have genetic alterations of the MAPK growth factor receptor signalling pathway.
e. Name the 2 different genetic alterations that can occur. (1)

A

Name the 2 different genetic alterations that can occur.

1) The most common mutation is BRAF V600E (occurs in approx 50%). Is particularly associated with hobnail and tall cell variants.

2) TERT mutations occur in approx. 20%. Both mutations are associated with a poorer prognosis.

299
Q

IHC and molecular genetics for follicular thyroid cancer:

A

IHC:
TTF1+, thyroglobulin+, PAX8+

Molecular genetics
o NRAS and HRAS mutation in 50%
o PAX8-PPAR gamma rearrangement in 33%

300
Q

IHC and molecular genetics of Medullary thyroid cancer:

A

IHC
o Calcitonin+, CEA+, TTF1 (weak-moderate), PAX8 (variables to weak), Congo red-stain for amyloid
o CEA+, chromogranin A+, synaptophysin+
o Thyroglobulin -
o Para-ganglioma-like variant: melanin+, calcitonin+, S100+
o Small cell variant: calcitonin+
o Tubular variant: calcitonin+
o Usually negative for: thyroglobulin-, ER-,

Molecular genetic:
1) RET proto-oncogene gain in function mutation (exon 6 M918T mutation is most common), in majority of familial MTC, and 50% of sporadic MTC
2) NRAS - 20%

301
Q

Compare the epidemiology, natural history and microscopic features of Papillary Thyroid Carcinoma and Anaplastic Thyroid Carcinoma. (4)

A

Epi:
- Papillarly: Most common thyroid cancer (80%), 2.5xF:Male, Average age 50.
- Ana: Rare (<5% of thyroid cancers), more common in elderly >70, often with a prior (or current) Hx of thyroid cancer (40%) or Goitre (20%)
Nat Hx:
- Pap: Tend to be slow growing, with spread to nodes more likely with more advanced local disease. 2/3 present with thyroid confined disease, 1/3 with nodes. Systemic spread especially to lung is common in node+/locally advanced disease. Typically responsive to RAI.
- Ana: Rapidly growing, locally aggressive with early nodal and systemic spread. Often metastatic at diagnosis with median OS 3 months (10% at 1 year). Not responsive to RAI.

302
Q

What are the barriers to enrolment in clinical trials?

A

Four sources of barriers have been identified:
- Patient (beliefs or trust, distance to trial sites, health insurance coverage, language, and immigration status),
- Clinician (limited awareness of trial, time constraint, and non-cooperation from colleagues),
- Clinical (eligibility criteria and clinical design), and
- Institutional (policy and limited logistic support).

303
Q

Define and outline the advantages and limitations of the following commonly used endpoints in oncology clinical trials: (4)
i. Overall Survival (OS)

ii. Progression Free Survival (PFS)

iii. Disease Free Survival (DFS)

iv. Time to Treatment Failure (TTF)

A

i. Overall Survival (OS)
Is generally taken as the average length of time participants are alive from the commencement of a trial (time of randomization). Considered a gold-standard clinical endpoint, but in many situations the ability to measure it is limited – E.g. due to a trial timeframe (e.g. 3 yrs) being shorter than the average OS of the population being studied (e.g low risk prostate cancer). Issue 2 - For longer study times (e.g. 10yrs), drop out rates can be high and prone to bias, thereby limiting an accurate measurement (loss of power). For these reasons surrogate measures (e.g. PFS) may be preferred, but are indirect at best and come with their own limitations.

ii. Progression Free Survival (PFS)
Best where clinical evidence of disease is not eradicated by the treatment (e.g. primary RT to a meningioma) - Defined as the average (or median) time from study commencement (randomization) to progression. Advantages: Often occur within a realistic trial time-frame (e.g.2-5 yrs), denotes a significant clinical event from a treatment and patient perspective. Disadvantages: requires a defined/agreed upon clinical threshold for progression, may be a very indirect measure of OS (especially where many lines of treatment remain).

iii. Disease Free Survival (DFS)
In contrast to PFS, DFS is best suited where a treatment has left patient with initially no signs of disease (e.g. treatment to solitary plasmacytoma). The same advantages and disadvantages otherwise apply.

iv. Time to Treatment Failure (TTF): The duration from randomization to discontinuation for any reason (patient drop out, toxicity, treatment failure, death). Advantage – may be readily calculated from the data, but lacks specificity and depending on the situation may not be clinically useful. Not typically a primary end-point.

304
Q

Write brief notes on the following para-neoplastic syndromes. Include in your answer for each the commonly associated malignancies and causal mechanisms to support the diagnosis (where applicable). (4.5)
Carcinoid syndrome

A

Is associated with neuroendocrine tumours (e.g. Neuroendocrine pancreatic cancer) and is a combination of: Flushing, Diarrhoea, Bronchoconstriction, Cardiac disease.

Mediated by a variety of tumour secreteing substances in particular serotonin - a main metabolite of which if elevated in urine can point to a diagnosis of this syndrome.

Treatment - debulk, somatostatin analogs - particularly octreotide.

305
Q

Main cancers and MOA for paraneoplastic SIADH:

Mx:

A

By far the most common maligant cause is SCLC, which produces ADH causing aquaporins in the collecting ducts to increase resoption of H20 -> increased volume in plasma and diluting [Na]p. Other cancers that rarely may cause - bladder, prostate, CNS tumours.

Mx:
If chronic, limit rate of correction
Fluid restriction should generally be first-line therapy
Consider pharmacologic therapies if serum Na + is not corrected after 24-48 hr of fluid restriction or if patient has a low urinary electrolye free water excretion.

Second-line treatments include increasing solute intake with 0.25–0.50 g/kg per day of urea or a combination of low-dose loop diuretics and oral sodium chloride

306
Q

For each of the following simply give doses, the study they cam from, a guideline supporting their use (and the evidence of grade assigned by that guideline):

Prostate-only RT:
* EBRT only
* EBRT + Brachy
Stereotactic radiotherapy (SBRT):

Pelvic nodal RT (node -):
*The current gold standard.
*Your approach

Postoperative RT:

A

All the below doses are within UK-RCR
Prostate-only RT:
* 60 Gy/20 over 4 weeks CHIiP Trial (Grade A)

EBRT+Brachy
* 46 Gy in 23 fractions (prostate and pelvic nodes) over 4.5 weeks followed by 15 Gy HDR or 115 Gy LDR brachytherapy boost (Grade A)

SBRT:
* 36.25Gy/5(Grade A) - 2 trials: Hypo RT (42/7), PACE-B 36.25/5 (USE). Same outcomes as conventional, but possibly higher late grd II GU.

Pelvic nodal RT node -ve:
* 50Gy/25 over 5 weeks or equivalent (Grade A) - POP-RT current only trial for node-ve to show benefit - therfore this is current standard.
*44-47/20 are in common use. 46/20 (2.3Gy/#) = 50Gy BED (Grade D)

Node +: STAMPEDE - PFS benefit. 60/20 to Pr, 44/20 nodes, 55-60Gy to +ve nodes (OARS permitting).

Postoperative RT:
* I favour early salvage and treat nodes (SPORT Trial 64.8/36, 45/25 nodes), but dose I prefer 55/20 (60/20 is PSMA residual) and 44/20 to the nodes

307
Q

A decision is made to administer TBI using an opposed lateral technique. Describe this radiation technique and include an appropriate dose fractionation schedule. (5)

A

TBI delivered to a total of 12Gy/6#, 2#s/day treat over 3 days, opposed lateral photons (SSD 4m), aim energies below 6Mv, prescribed to midpoint (typically umbilicus). Alternating between lateral beams at each fraction. Aim for 7 days prior to transplant.

Pre sim: Anti-nausea medication, other meds to consider are dexamethasone and anti-anxiety meds (e.g. Lorazepam). Fertility preservation (though they’re pretty fucked if not done by this point).
Sim: Supine, small under head, knee fix, Upper arm resting on 4cm polystyrene (to reduce lung dose), Hand resting on abdomen.
Compensator/ bolus (‘beam spoiler’) – (because of skin-sparing effect of photon):
Bolus (E.g. flab) – on lateral and anterior surface of neck + chest (to reduce dose to lung)
Perspex – as head frame compensator, and from mid-thigh inferiorly (thicker from mid knee
inferiorly)Linac field typically at most open position/largest practical field size.
Volume: Whole Body – Covered by light field. Aim homogenous dose +/-5% of PD.
OAR: Lungs – Aim <12.6Gy, consider Lung shields.

Verification:
Patient position confirmed with laser matching. Whole body within light field, in vivo dosimetry (e.g. TLDs on skin surface) not routine.

308
Q

Which other treatment approaches may be used to achieve the same benefit as conventionally fractionated TBI? What are the advantages and disadvantages of these approaches? (3.5)

A

1) Systemic therapy/additional chemo (e.g cyclclophosphamide or mephalin for Myeloma). Advantages: More available than TBI, avoid acute and late toxicities of radiation. Treat pts with contraindications to RT (e.g. connective tissue disorders). Disadvantages: Increased side effects (including death) from further cytotoxic therapy, systemic therapy may not penetrate sanctuary sites as well as XRT and may not overcome chemo resistant cells.
2) Lower dose TBI – e.g. 2Gy/1#, 4Gy/2 – Advantage: May achieve acceptable immune suppression/suff while sparing significant toxicity. Disadvantages: May not achieve tumour cell elimination.

309
Q

a.What are the two hypothetical mechanisms of radiation that can be exploited for the treatment of benign disease? Give two examples of each. (2)

A

Anti-inflammatory effect (usually in the range 2-6Gy): Low dose radiation induces changes in immune cell rolling, adhesion and migration (e.g.reducing NO mediated pro-inflammatory states and production of adhesion molecules). E.g Hydraenitis supperativa and synovitis.

Antiproliferative (around 10Gy): Radiation can interrupt the cell cycle and critical growth phases - e.g. post operatively during profileration in Heterotrophic Ossification, or in the proliferation phase of early Duyptren’s.

310
Q

Discuss the role of radiation for gynaecomastia caused by androgen deprivation therapy.
Include in your answer a suitable technique, dose fractionation schedule and response rate. What other alternatives to radiation exist? (3)

The Alternative…

A

Pooled analaysis also suggests tamoxifen 10-20mg (in a prophylaxis setting) equally or more effective.

Radiation (e.g. 20Gy/5#) is more effective if given prophylactically - 3/4 prevention rate, than after gynaecomastia has occurred - 1/3 response rate and 1/3 pts reporting improved pain.
Technique:
Palliative intent EBRT, MeV electrons (energy selected for deepest extent of breast tissue covered by D90, e.g 18mm choose,6Mev). 15Gy/3#, 1#/day.
Sim: supine, head towards gantry, arms above head, no bolus (to reduce skin dose). Immobilise with vac bag.
CTV = Breast tissue on planning 2mm slice CT.

311
Q

Regarding Pleomorphic Adenoma vs. Warthin’s Tumour
In a table format, compare and the contrast the following:
i. Clinical features. (1)

ii. Biological behaviour. (1)

A

i. Clinical features. (1)
Pleo: Most common benign (and overall) parotid tumour (80% benign), slow growing, firm painless round lump. Wide age range, more common in females.
WT: Second most common benign parotid tumour, soft round slow growing lump. More common in older men and smokers.

ii. Biological behaviour. (1)
Pleo: Typically indolent and cured with surgery, Recurrence is common! (50%), can transform into Carcinoma Ex pleomorphic adenoma (5%)
WT: Highly PET avid and is often incidental finding on PET, Cured by surgery, malignant transformation vary rare (<1%)

312
Q

Describe the microscopic features of Pleomorphic Adenoma. (1)

A

Triphasic tumour with central ductal Ck7+ve, myoepithelial (S100, SOX100 +ve) and stromal components (often hyalinised and or sclerotic).

313
Q

Regarding Mucoepidermoid Carcinoma vs. Adenoidcystic Carcinoma:
Compare and the contrast the following:
Epidemiological and clinical features. (1)

Pathological features (Macroscopic and Microscopic). (2)

A

Epidemiological and clinical features. (1)
Muco: Most common malignant parotid tumour, Most common radiation therapy induced tumour. M=F, occurs over a wide range
Adenoid: Most common malignant salivary gland tumour. More common in older patients (>60), May present with clinical Sx of PNI>

Pathological features (Macroscopic and Microscopic). (2)
Muco: Macro: Mucin filled cysts, solid, well demarcated but no capsule. Micro: MIX OF mucin producing columnar epithelia, and solid epidermoid cells. Grade based on solid extent component.
Adenoid: Macro = solid, well demarcated, no capsule. Micro= BIPHASIC TUMOUR – Ductal and myoepithelial components. PNI may be evident.

314
Q

An otherwise well 58-year-old man presents with chest pain. CT demonstrates a mass at the apex of the left lung, invading the first and second ribs anteriorly.
Core biopsy confirms squamous cell carcinoma (SCC).
What specific aspects of clinical examination and investigations would allow you to determine the best management plan for this patient? (2)
What Stage/T score is this (if N0/M0)

A

Stage T3 (chest wall, unifocal) = stg II

Exam: Signs of brachial plexus or nerve involvement. Signs of subclavian vessel involvement. Full systems exam with focus on respiratory system - including dyspnoea at rest and physical effort.
Ix: PET scan for complete staging - determine suitable for curative intent.
MRI Brain – for stage II and above (as per NCCN)
Tissue - grade.
Respiratory function test, FBC, U&E, LFT, Coags.

315
Q

Contraindications to bladder preservation approach:

A

Assess for relative contraindications: Tumour>5cm, multifocal/in-situ, incomplete TURBT, irreversible hydronephrosis.

316
Q

Indications of breast boost - name who recommends?

A

ASTRO (2018) Guidelines:

50 years or younger with any grade tumor, or in patients aged 51 to 70 years with high-grade tumors or positive margins.

***Consistent w/subgroup analysis of Bartelink (now 20yrs) - suggests greatest benefit age<50. Also those w/adjacent DCIS (at long term follow up) and grdIII

317
Q

In general, what are the principles of systemic treatment of breast cancer in pregnant woman (2m)

A

Chemo:
- Generally safe in 2nd and 3rd trimester
- Stop chemotherapy 3 weeks prior to delivery to avoid complication during delivery
(neutropenia, thrombocytopenia)

Her 2 therapies:
- Contraindicated
- Trastuzumab increased risk of oligohydramnios/anhydramnios, and may cause foetal
respiratory and heart failure

Tamoxifen:
- Contraindicated
- a/w high risk of birth defect, spontaneous abortion, foetal demise

318
Q

In general, what are the options for the further management of the axilla where one sentinel node is positive?
Justify your answer (2m)

A

1) Observation (i.e. no further axillary treatment)
Z0011 study, patients with 1-2 SLN +
- Randomised to ALND vs. observation
- no differences in nodal recurrence/ OS with between arms
- higher risk of lymphoedema in patients who had ALND (13% vs. 2%) Also, given that she has endocrine therapy and targeted therapy options
2) Axillary RT
AMAROS study – patients with SLN+
- randomised to level 1-3 ALND vs. axillary RT 50Gy/25#
- no differences in axillary recurrence (primary endpoint) or OS between arms
- less lymphoedema with axillary RT (23% vs. 11%)
3) ALND In AMAROS, ALND arm
- 33% had further LN+, and 8% had 4 LN+
4)Clinical trial POST-NOC (1-2 SLN+, macro mets >2mm)
- randomised to observation vs. axillary treatment (surgery or RT)

319
Q

32F completely excised left breast IDC tripple pos, grd II T2.
She undergoes a left level II axillary dissection and is found to have a total of 4 macro-metastases from 15 nodes. She is referred for adjuvant radiation therapy.
Provide justification for your choice of nodal volume (1m)

A

This pt with N2 disease following ALND would benefit from RNI to levels 3, SCF and IMN. I would boost the surgical bed for improved local control given age <50 (Barlelink 20yr data, Astro guidelines).

As per MA.20 and EBCTG meta-analysis this is likely to improve DFS (in the order of 5% or more). MA.20 included IMN, and specific benefit is noted in prospective data (Thorsten) when N2 disease.

320
Q
A
321
Q

Describe the basal like subtype of breast cancer (2m)

A

When a RANZCR path question says “describe” they want epi, macro, micro, bio beh.

Epi
- 15-20% of breast cancer - Young patients,
- a/w BRCA1 mutation

Micro:
High proliferation rate
Usually grade 3, high mitotic indices, with necrosis, pushing borders, conspicuous LVI

Molecular/IHN
Tripple negative
p53 often mutated
HMWCK +
Biological behaviour

322
Q

List the risk factors for triple negative breast cancer

A
  • Younger patients
  • p53 mutation
  • BRCA mutation carriers
  • Gynae: early menarche, early age at first pregnancy
  • More commonly a/w pregnancy-related breast cancer, inflammatory breast cancer
323
Q

Contribution of genetics to BrCa risk:

Describe the BRCA 1 cancer risks:

Describe the BRCA 2 cancer risks:

A

BRCA1—AD (17q21), 70% lifetime risk of breast cancer, 30% to 50% lifetime risk of ovarian cancer, higher risk of triple negative (ER−/PR−/HER2−);

BRCA2—AD (13q12), 50% lifetime risk of breast cancer

10% to 20% lifetime risk of ovarian cancer, male breast cancer, prostate, bladder, endometrial, and pancreatic ca.

324
Q

Key pathological features of Invasive lobular carcinoma:

Include key mutational risks

A

10 to 15% of cases overall, more common in women 45-55, estrogen exposure appears a greater risk factor (e.g. HRT),

Macro: Mass with ill-defined margins (often no mass because diffuse growth pattern, often Multifocal/ multicentric

CDH1 mutation increase risk of lobular but not ductal cas,

BRAC2 increases risk

Poorly visible on mammogram (better w/ MRI).

Monotonous small non-cohesive infiltrative cells arranged in
single file/ Indian file linear pattern
- Cells can be seen encircling normal duct (onion skin pattern)
- Classic ILC will not show tubule formation, and typically
not mitotically active
- No desmoplastic stroma
>80% ER+

Spreads to unusual locations such as meninges, serosal surfaces, BM, ovary, and RP - ?WHY????

325
Q

Key pathological features of Invasive ductal carcinoma:

Incidence, imaging
Macro
Micro
IHC
Bio beh

A

80% cases. Often well seen on mammogram.

Macro:
Firm mass (cartilagenous feeling) irregular mass with desmoplastic reaction, advanced can show teathering to sking (dimpling of the skin surface).

Micro:
Invasive malignant epithelial cells with degree of tubule formation suggestive of grade (more = lower) along with nuclear pleomorphism and mitotic rate. Surrounded by desmoplastic stroma. Lack characteristic feature suggestive of special type.

IHC:
Ck7, E-cadherin +, 60% ER+, 60%PR+, 30%HER2

326
Q

The TROG EXPERT breast cancer trial uses the PAM50 risk of recurrence score. Describe this tool and its clinical utility (2m)

A

Tool - Micro-array test using quantitative PCR on RNA extracted from FFPE tissue specimen
- Test for 50 genes; generate risk of recurrence (ROR) score: low risk 0-40; intermediate risk 41-60; high risk 61-100)

Utility - Predict risk of distant mets in post-menopausal women with ER+ N0 disease who had 5-years of
endocrine therapy
- Can also identify intrinsic subtypes (i.e. Luminal type)
- Further stratify women with low risk of recurrence

327
Q

A patient gets a DXA scan, how would you relate that to # risk:

A

T-score (number of standard deviations that BMD is above or below the mean of healthy young adults) or a Z-score (number of standard deviations that BMD is above or below the mean of normal controls matched for age).

Fracture risk doubles for every standard deviation below the mean for a young adult.

Combine that with other risks - in particular falls, or high risk activities (e.g break dancing)

328
Q

Give an example of toxicity scoring system for radiation dermatitis and provide clinical description of each grade (3m)
RTOG (Radiation Therapy and Oncology Group) scoring

A
  • G0: No Sx
  • G1: follicular/ faint, or dull erythema, epilation, dry desquamation
  • G2: tender, or bright erythema, with patchy moist desquamation/ moderate oedema
  • G3: confluent moist desquamation
  • G4: ulceration, haemorrhage, necrosis
  • G5: Death
329
Q

A fuck up in MU calculation leads to a patient getting 1.5x the intended dose. What is the procedure?

A

Full disclosure to patient: dose to OARS, quantify risks as much as possible inform on the process (see below):

1) Departmental procedures: Notify radiation safety officer (member of physics team in most deparments). Radiation safety offer to notify (see step 2). Initate hospital incident report and internal departmental inicidentr review mechanisms (e.g. with M&M meeting discussion)
2) Office of radiation safety notified - initial notification form. Depending on incident may require additional information. Performs review,
3) Results of review shared with relevant parties.

330
Q

Doses and volumes for penile cancers:

And benefit for each.

Also, indications for concurrent chemo

A

Use 2gy/# cisplatin 40mg.
1) Definitive local:
- Primary <4cm and does not involve cavernosum local +/- cisplatin - 60Gy/30#s CTV = GTV+2cm (alternative is brachy 60/5). Surgery seems about equivalent for local - 5yr OS 75%, 80% LCR.

2) Definitive advanced (surg is the standard approach):
>4cm, or >=T3 (meaning cavernosus invasion or worse): 50/25 whole penis and inguinal/pelvis, boost primary (+ve nodes, or ece and dissection) to 66/33Gy. This supported by UK RCR.
(PORT is the same but adds 60/30 to micro)

> 4cm or >T2 should always get concurrent chemo:
E.g. cisplatin or cisplain + MMC

3)PORT: 66/33 to residual, 60/30 to R1and ECE, then 50/25 to everything else. Chemo is R1, ECE or very advanced. Benefit extrapolated from anal/cervix. RT added to CT may increase PFS to 30% from 15% (multi-institutional anaylsis).

4) NeoAdj is investigational InPACT (45/25).

331
Q

A previously well 63 year old man presents with a hard, ulcerated 3cm mass on his glans penis. A biopsy confirms poorly differentiated squamous cell carcinoma of the penis.
a. How would your further stage this patient? Justify your answer (2m)

A

Primary
- Urethroscopy/ cystoscopy to assess urethra involvement
- MRI with artificial erection to assess the degree of local invasion (i.e. corpus spongiosum/
corpus cavernosum/ urethra)

Node:
- CT pelvis and FDG PET scan for radiological evaluation of nodal disease
- If node negative of clinical examination/ staging imaging, I will organize pathological evaluation
with either sentinel lymph node biopsy or bilateral inguinal lymph node dissection for nodal
staging
- If clinical node positive on imaging, CT-guided biopsy of involved node

332
Q

a. In general, what are the indications for adjuvant radiation therapy in the management of thymoma? (3m)

c. What para-neoplastic syndromes are associated with thymoma? (2m)

A

a. In general, what are the indications for adjuvant radiation therapy in the management of thymoma? (3m)
- Stage II-IV A disease
- Residual disease (R1-R2)
- Thymic cancer (i.e. WHO Grade C)

  • Myasthenia gravis (most common ~ 50% of thymoma patients have myasthenia gravis)
  • Red cell aplasia
  • Hypo-gamma-globulinaemia
  • Cushing’s
  • Addison
333
Q

a. In general, what are the indications for adjuvant radiation therapy in the management of thymoma? (3m)

A

a. In general, what are the indications for adjuvant radiation therapy in the management of thymoma? (3m)
- Stage II-IV A disease
- Residual disease (R1-R2)
- Thymic cancer (i.e. WHO Grade C)

334
Q

Key cytogenetics for:
Liposarcoma

Leiomyosarcoma

Synovial

Rhabdo

Fibrosarcoma

A

Lipo - MDM2 overexpressed especially in de-differentiated lipo
do (12q13) FISH test

Leio - Most common Li-Fraumini = TP53

Synovial t(X:18)(p11:q11) involving genes SS18, and either SSX1, SSX2:
o SS18-SSX1 inhibit Snail gene,
o SS18-SSX2 inhibit Slug gene
(X,18)

Rhabdo:
FOXO1 translocation for alveolar rhabdomyosarcoma

Fibrosarcoma = Aneuploidy.

335
Q

For each of the following subtypes of soft tissue sarcoma, write brief notes on the key histological features and any critical chromosomal abnormalities
i. Fibrosarcoma (1m)
ii. Well-differentiated liposarcoma (1m)
iii. Synovial sarcoma (1m)
iv. Leiomyosarcoma (1m)

A

Fibrosarcoma:
Highly cellular fibroblast proliferation in herringbone pattern,
cyto = anueploidy

WD Lipo:
Mature adipocytes with variable size - Bizarre and hyperchromatic stromal cells within fibrous stroma

Synovial:
2 subtypes: spindle cell only (monophasic) or biphasic, spindle-epithelial. Chromosomal translocation t(X:18)(p11:q11), involving SS18, and SSX1/ SSX2

Leimyosarcoma:
intersecting fascicles of spindle cells
- Elongated ‘cigar-shaped nuclei
- Necrosis common in large tumour
- Mitotically active

No key cytogentics, but most common Li-Fraumini (p53) associated.

336
Q

ii. Compare and contrast two predictive tools that are commonly used in assessing patients with DCIS?

A

Van Nuys prognostic index – Older (than IBTR), though updated, has been retrospectively validated. Based on margins, size, grd, age, does not include hormone therapy.

MSKCC (employs more variables than Van Nuys) – Retrospective prognostic and predictive validation: Commonly used and easily accessible on internet. Based on age, size, grade, necrosis, number of re-excisions, family Hx, presentation and hormone therapy and radiation therapy.

337
Q

iii. In general, what is the goal and magnitude of the benefit from adjuvant radiation therapy for DCIS.

A

The goal of therapy is to reduce ipsilateral breast tumour recurrence (approximately 50% of recurrences are invasive disease). Broadly (depending on prognostic factors) recurrence at 10 years ranges between 10-30% (very significantly less if hormone therapy part of treatment), 50% or recurrences are with invasive disease. With adjuvant RT reducing by more than half (~60%) this risk.

338
Q

List the adverse prognostic factors for Neuroblastoma (2m)

A

Prognostic factors = Factors in COGS staging (‘SAND-S’)
- Stage: higher stage = worse
- Age (>18mo = worse)
- N-MYC protein amplified = worse
- DNA ploidy (diploid = worse; hypo/ hyperploid = better)
- Shimada classification (SAD-MiNd).

SHIMADA (SAD-MiNd):
o Stroma pattern (rich = better; poor = worse)
o Age
o Differentiation of neuroblast (differentiated = better; undifferentiated = worse) o Mitotic=karyorrhexis index (low = better; high = worse)
o Nodularity (diffuse = better; nodular = worse)
- Cytogenetic abnormalities
o Favourable – hyperploidy DNA, TrK-A amplification
o Unfavourable – N-Myc protein amplification, diploid DNA, deletion of 1p chromosome, del 11q, TERT
rearrangement, ALK amplification

339
Q

List the factors used by the Children’s Oncology Group or International Neuroblastoma Group to stratify patients into various risk groups (2m)

Yeah this question is asked tiwce (most common neuroblastoma exam question)

A

‘SANDS’
- INSS Surgical Stage
- Age
- N-MYC amplification
- DNA ploidy
- Shimada histology (SADMiNd) (Favourable histology vs. Unfavourable histology)
o Stroma (rich = FH; poor = UH)
o Differentiation (differentiated = FH; undifferentiated = UH) o Mitotic-karyorrhexis index (low = FH; high = UH)
o Nodularity (diffuse = FH; nodular = UH)

340
Q

Roa NOA Nordic Roa

Explain

A

Key RCT trials for older people w/GBM

Roa = phase III RCT - 40/15 = 60/30. There is an OS benefit to the addition of TMZ to short course.

NOA & NORDIC = TMZ = RT if methylated

Roa = Phase III - 25/5 = 40/15
IF
- age >64
Or
- age >50 and poor performance status (KPS 50-70)

341
Q

Discuss the use of chemotherapy in uterine cancer including in your answer the rationale for its use. (2)

A

RCT Data (PORTEC3) demonstrates at 5years concurrent chemo (weekly cisplatin) followed by adjuvant x4 cycles (Carboplatin Paclitaxel) can provide LC, FFS and OS benefit when concurrent with radiotherapy. It may be considered from grd 3 stg1C.
Sub-analysis of PORTEC3 shows the benefit of chemo is primarily in any of the following: Stage III (serosa or nodes involved), Papillary Serous or p53abn.

342
Q

Discuss the role of systemic therapy in the management of metastatic prostate cancer. Include in your answer the class of agents used, duration of therapy and justification for use. 5 marks

A
  • Anti-androgen medications - Typically non steroidal anti-androgens (e.g. Bicalutamide): In hormone naive pts: part of initial therapy + GnRH agonist +/- abiraterone (STAMPEDE ADT+enzalutamide + abiraterone no benefit). Alternatively can be used for 3-4 weeks as part of commencement of GnRH agonist - to prevent testosterone spike from initial LH surge. Can also be used when progression despite GnRH agonist (“total androgen blockade) to prolong PFS. ADT+ Apalutamide superior DMFS (SPARTAN). Used life-long or until significant progression.
  • GnRH agonist (e.g Goserelin 10.8mg s/c q3monthly), provides a PFS, OS and symptomatic benefit in patients with newly Dx metastatic prostate cancer, and is part of multimodal therapy as above. Duration life-long, or until progression, or in the setting of stable disease intermittent (restart when >10ng/ml).

-Androgen synthesis inhibitor - Abiraterone: Improves OS and PFS (STOPCAT meta-analysis)- used in the following contexts - as initial therapy for newly Dx metastatic disease (in combination with ADT but not ADT+Enzalutimide STAMPEDE), or when progression on ADT/disease progression.

-Chemo - Typically Taxane chemotherapy - Docetaxel, improves PFS and OS (CHAARTED phase III RCT OS increase from 48 to 58months) when part of initial Mx of metastatic disease. Also increases OS for N+ and locally advanced (STAMPEDE). 75mg/m.sqr (65mg for Asians) until response (up to 10 cycles).